Gulfie Dentists Students
Evergreen Performance Test
Exams in alternate weeks to test your level of preparation.

Test of the Week
Scheduled for Sunday
Test of the Week
PERFORMANCE TEST
Quiz-summary
0 of 150 questions completed
Questions:
- 1
- 2
- 3
- 4
- 5
- 6
- 7
- 8
- 9
- 10
- 11
- 12
- 13
- 14
- 15
- 16
- 17
- 18
- 19
- 20
- 21
- 22
- 23
- 24
- 25
- 26
- 27
- 28
- 29
- 30
- 31
- 32
- 33
- 34
- 35
- 36
- 37
- 38
- 39
- 40
- 41
- 42
- 43
- 44
- 45
- 46
- 47
- 48
- 49
- 50
- 51
- 52
- 53
- 54
- 55
- 56
- 57
- 58
- 59
- 60
- 61
- 62
- 63
- 64
- 65
- 66
- 67
- 68
- 69
- 70
- 71
- 72
- 73
- 74
- 75
- 76
- 77
- 78
- 79
- 80
- 81
- 82
- 83
- 84
- 85
- 86
- 87
- 88
- 89
- 90
- 91
- 92
- 93
- 94
- 95
- 96
- 97
- 98
- 99
- 100
- 101
- 102
- 103
- 104
- 105
- 106
- 107
- 108
- 109
- 110
- 111
- 112
- 113
- 114
- 115
- 116
- 117
- 118
- 119
- 120
- 121
- 122
- 123
- 124
- 125
- 126
- 127
- 128
- 129
- 130
- 131
- 132
- 133
- 134
- 135
- 136
- 137
- 138
- 139
- 140
- 141
- 142
- 143
- 144
- 145
- 146
- 147
- 148
- 149
- 150
Information
Click start to practice this Evergreen Performance Test.
You have already completed the quiz before. Hence you can not start it again.
Quiz is loading...
You must sign in or sign up to start the quiz.
You have to finish following quiz, to start this quiz:
Results
0 of 150 questions answered correctly
Time has elapsed
You have reached 0 of 0 points, (0)
Average score |
|
Your score |
|
Categories
- DHA 0%
Pos. | Name | Entered on | Points | Result |
---|---|---|---|---|
Table is loading | ||||
No data available | ||||
- 1
- 2
- 3
- 4
- 5
- 6
- 7
- 8
- 9
- 10
- 11
- 12
- 13
- 14
- 15
- 16
- 17
- 18
- 19
- 20
- 21
- 22
- 23
- 24
- 25
- 26
- 27
- 28
- 29
- 30
- 31
- 32
- 33
- 34
- 35
- 36
- 37
- 38
- 39
- 40
- 41
- 42
- 43
- 44
- 45
- 46
- 47
- 48
- 49
- 50
- 51
- 52
- 53
- 54
- 55
- 56
- 57
- 58
- 59
- 60
- 61
- 62
- 63
- 64
- 65
- 66
- 67
- 68
- 69
- 70
- 71
- 72
- 73
- 74
- 75
- 76
- 77
- 78
- 79
- 80
- 81
- 82
- 83
- 84
- 85
- 86
- 87
- 88
- 89
- 90
- 91
- 92
- 93
- 94
- 95
- 96
- 97
- 98
- 99
- 100
- 101
- 102
- 103
- 104
- 105
- 106
- 107
- 108
- 109
- 110
- 111
- 112
- 113
- 114
- 115
- 116
- 117
- 118
- 119
- 120
- 121
- 122
- 123
- 124
- 125
- 126
- 127
- 128
- 129
- 130
- 131
- 132
- 133
- 134
- 135
- 136
- 137
- 138
- 139
- 140
- 141
- 142
- 143
- 144
- 145
- 146
- 147
- 148
- 149
- 150
- Answered
- Review
-
Question 1 of 150
1. Question
A patient came for regular dental check up. Dentist noticed some lesion on patient’s tongue as seen in the picture. Patient was unaware of this lesion on tongue. What is your diagnosis?
Correct
ANSWER
Benign migratory glossitisOTHER OPTIONS
⢠Lichen planus – Lichen planus is seen usually as greyish white striae on skin and oral cavity.
⢠Fissured tongue – Fissured tongue shows small furrows or fissures across the surface, causing the tongue to have a wrinkled appearance.
⢠Median rhomboid glossitis – Median rhomboid glossitis presents in posterior midline of dorsum of tongue, just anterior to the V-shaped grouping of circumvallate papillae.SYNOPSIS
⢠Benign migratory glossitis or geographic tongue is a psoriasiform mucositis of dorsum of tongue.
⢠Characteristic pattern is constantly changing pattern of serpiginous white lines surrounding areas of smooth, depapillated mucosa.
⢠Etiology is unknown and usually patient does not have any symptoms.
⢠May be seen associated with psoriatic skin.
⢠No treatment is necessary.REFERENCE
Shafer’s Textbook of Oral Pathology – 8th Edition Page No 31.Incorrect
ANSWER
Benign migratory glossitisOTHER OPTIONS
⢠Lichen planus – Lichen planus is seen usually as greyish white striae on skin and oral cavity.
⢠Fissured tongue – Fissured tongue shows small furrows or fissures across the surface, causing the tongue to have a wrinkled appearance.
⢠Median rhomboid glossitis – Median rhomboid glossitis presents in posterior midline of dorsum of tongue, just anterior to the V-shaped grouping of circumvallate papillae.SYNOPSIS
⢠Benign migratory glossitis or geographic tongue is a psoriasiform mucositis of dorsum of tongue.
⢠Characteristic pattern is constantly changing pattern of serpiginous white lines surrounding areas of smooth, depapillated mucosa.
⢠Etiology is unknown and usually patient does not have any symptoms.
⢠May be seen associated with psoriatic skin.
⢠No treatment is necessary.REFERENCE
Shafer’s Textbook of Oral Pathology – 8th Edition Page No 31. -
Question 2 of 150
2. Question
A 45 year old male patient came to your clinic. He has severe periodontitis. He is a heavy smoker with halitosis and poor oral hygiene. What would be the main cause for progression of periodontitis?
Correct
ANSWER
Heavy smoking habitOTHER OPTIONS
Not applicableSYNOPSIS
⢠Smoking results in increased periodontal destruction.
⢠It alters neutrophil chemotaxis, phagocytosis and oxidative burst.
⢠It increases TNF alpha and prostaglandin E2 in GCF.
⢠Immunoglobulin G2 level is reduced suggesting reduced protection against periodontal infection.
⢠Nicotine suppresses osteoblast production and stimulates alkaline phosphatase activity.
⢠Smoking in turn can also result in poorer oral hygiene.REFERENCE
Essentials of Clinical Periodontology and Periodontics – 5th Edition Page No 151.Incorrect
ANSWER
Heavy smoking habitOTHER OPTIONS
Not applicableSYNOPSIS
⢠Smoking results in increased periodontal destruction.
⢠It alters neutrophil chemotaxis, phagocytosis and oxidative burst.
⢠It increases TNF alpha and prostaglandin E2 in GCF.
⢠Immunoglobulin G2 level is reduced suggesting reduced protection against periodontal infection.
⢠Nicotine suppresses osteoblast production and stimulates alkaline phosphatase activity.
⢠Smoking in turn can also result in poorer oral hygiene.REFERENCE
Essentials of Clinical Periodontology and Periodontics – 5th Edition Page No 151. -
Question 3 of 150
3. Question
A 27-year-old patient presents with localized aggressive periodontitis. Subgingival plaque samples reveal the presence of Aggregatibacter actinomycetemcomitans. The clinician considers adjunctive systemic antibiotic therapy. Which of the following is the only antibiotic in periodontal therapy to which all strains of A. actinomycetemcomitans are susceptible?
Correct
ANSWER
CiprofloxacinOTHER OPTIONS
⢠Erythromycin – Many strains show resistance.
⢠Amoxicillin – Effective against some oral pathogens but not reliably against A. actinomycetemcomitans alone.SYNOPSIS
⢠Aggregatibacter actinomycetemcomitans is a key pathogen in aggressive periodontitis.
⢠Ciprofloxacin is unique because all strains of A. actinomycetemcomitans are susceptible to it.REFERENCE
Carranzaās Clinical Periodontology, 13th EditionIncorrect
ANSWER
CiprofloxacinOTHER OPTIONS
⢠Erythromycin – Many strains show resistance.
⢠Amoxicillin – Effective against some oral pathogens but not reliably against A. actinomycetemcomitans alone.SYNOPSIS
⢠Aggregatibacter actinomycetemcomitans is a key pathogen in aggressive periodontitis.
⢠Ciprofloxacin is unique because all strains of A. actinomycetemcomitans are susceptible to it.REFERENCE
Carranzaās Clinical Periodontology, 13th Edition -
Question 4 of 150
4. Question
An 8 year old child comes to the clinic with 1 year old fractured central incisor. On examination tooth is found to be non vital. How will you manage this case?
Correct
ANSWER
ApexificationOTHER OPTIONS
⢠RCT – Later required following apexification once the root end is closed.
⢠DPC – done in cases with minute pulp exposure within short period of time.
⢠IPC – done in cases with deep dentinal caries that doesn’t reach pulp.SYNOPSIS
⢠Apexification is the treatment of choice for non vital teeth with open apex and pulp exposure.
⢠Apexification is a method of treatment for immature permanent teeth in which root growth and development cease due to pulp necrosis.
⢠Its purpose is to induce root end closure with no canal wall thickening or continuous root lengthening.
⢠It can be achieved in two ways
1) As a long-term procedure use calcium hydroxide dressing to allow the formation of a biological hard tissue barrier.
2) As a short-term, more recent procedure, creating an artificial apical plug of MTA or other bioceramic material.
⢠Apexification is most often performed in incisors that lost vitality because of traumatic injury, after carious exposures, and in teeth with anatomic variations such as dens invaginatus with an immature root.REFERENCE
Pediatric Dentistry Arthur.J.Nowak Page 489Incorrect
ANSWER
ApexificationOTHER OPTIONS
⢠RCT – Later required following apexification once the root end is closed.
⢠DPC – done in cases with minute pulp exposure within short period of time.
⢠IPC – done in cases with deep dentinal caries that doesn’t reach pulp.SYNOPSIS
⢠Apexification is the treatment of choice for non vital teeth with open apex and pulp exposure.
⢠Apexification is a method of treatment for immature permanent teeth in which root growth and development cease due to pulp necrosis.
⢠Its purpose is to induce root end closure with no canal wall thickening or continuous root lengthening.
⢠It can be achieved in two ways
1) As a long-term procedure use calcium hydroxide dressing to allow the formation of a biological hard tissue barrier.
2) As a short-term, more recent procedure, creating an artificial apical plug of MTA or other bioceramic material.
⢠Apexification is most often performed in incisors that lost vitality because of traumatic injury, after carious exposures, and in teeth with anatomic variations such as dens invaginatus with an immature root.REFERENCE
Pediatric Dentistry Arthur.J.Nowak Page 489 -
Question 5 of 150
5. Question
Which is the space maintainer of choice for missing single 2nd mandibular molar tooth in a 5 year old?
Correct
ANSWER
Distal shoeOTHER OPTIONS
⢠Lingual arch – It is indicated in case of bilateral loss of primary molars when no permanent anteriors are erupted.
⢠Band and loop – Can be used to replace E only after eruption of permanent first molar.
⢠Nance palatal arch – For missing bilateral maxillary molars.SYNOPSIS
⢠The distal shoe appliance is used to maintain the space of a primary second molar that has been lost before the eruption of the permanent first molar.
⢠An unerupted permanent first molar drifts mesial within the alveolar bone if the primary second molar is lost prematurely.
⢠The distal shoe appliance is used to maintain the space of a primary second molar that has been lost prematurely before the eruption of the permanent first molar.
⢠A stainless steel extension is soldered to the distal end of the band and loop, this extension is positioned 1 mm below the mesial marginal ridge of the unerupted permanent first molar.
⢠The extension serves to guide the eruption of the permanent first molar.
⢠The distal shoe can have modifications.
⢠It can also be fabricated with the crown and distal shoe modification in design.
⢠Although feasible, like the crown and loop, it is difficult to modify and repair.REFERENCE
Pediatric dentistry Arthur.J.Nowak Page 381Incorrect
ANSWER
Distal shoeOTHER OPTIONS
⢠Lingual arch – It is indicated in case of bilateral loss of primary molars when no permanent anteriors are erupted.
⢠Band and loop – Can be used to replace E only after eruption of permanent first molar.
⢠Nance palatal arch – For missing bilateral maxillary molars.SYNOPSIS
⢠The distal shoe appliance is used to maintain the space of a primary second molar that has been lost before the eruption of the permanent first molar.
⢠An unerupted permanent first molar drifts mesial within the alveolar bone if the primary second molar is lost prematurely.
⢠The distal shoe appliance is used to maintain the space of a primary second molar that has been lost prematurely before the eruption of the permanent first molar.
⢠A stainless steel extension is soldered to the distal end of the band and loop, this extension is positioned 1 mm below the mesial marginal ridge of the unerupted permanent first molar.
⢠The extension serves to guide the eruption of the permanent first molar.
⢠The distal shoe can have modifications.
⢠It can also be fabricated with the crown and distal shoe modification in design.
⢠Although feasible, like the crown and loop, it is difficult to modify and repair.REFERENCE
Pediatric dentistry Arthur.J.Nowak Page 381 -
Question 6 of 150
6. Question
A 10-year-old patient presents with an anterior crossbite involving the incisors. Clinical examination reveals that the patient has a pseudo-Class III malocclusion. The patient exhibits a forward mandibular posture upon closure, but no skeletal discrepancies are observed on cephalometric analysis. The crossbite is believed to be caused by occlusal prematurities that are forcing the mandible forward into a Class III position. What is the most appropriate management for this patient?
Correct
ANSWER
Removal of occlusal prematuritiesOTHER OPTIONS
⢠Full orthodontic treatment with braces – It might be necessary if additional alignment or correction is needed after removing the occlusal prematurities, but it is not the first-line treatment.
⢠Using a chin cup to restrict mandibular growth – It is inappropriate for pseudo-Class III malocclusion since the issue is dental rather than skeletal.
⢠Orthognathic surgery – It is typically reserved for true skeletal Class III malocclusions and is unnecessary for a dental-based problem like pseudo-Class III.SYNOPSIS
⢠Pseudo-Class III malocclusion is often due to occlusal prematurities (interferences) that cause the mandible to shift forward during closure, creating an anterior crossbite. In such cases, the primary issue is dental, not skeletal.
⢠The most appropriate initial treatment is the removal of occlusal prematurities to eliminate the forward mandibular posture and correct the anterior crossbite. This conservative approach can often resolve malocclusion without the need for more invasive interventions.REFERENCE
Pseudo Class III malocclusion – Saudi Medical Journal 2016 Apr – 37(4), 450-456Incorrect
ANSWER
Removal of occlusal prematuritiesOTHER OPTIONS
⢠Full orthodontic treatment with braces – It might be necessary if additional alignment or correction is needed after removing the occlusal prematurities, but it is not the first-line treatment.
⢠Using a chin cup to restrict mandibular growth – It is inappropriate for pseudo-Class III malocclusion since the issue is dental rather than skeletal.
⢠Orthognathic surgery – It is typically reserved for true skeletal Class III malocclusions and is unnecessary for a dental-based problem like pseudo-Class III.SYNOPSIS
⢠Pseudo-Class III malocclusion is often due to occlusal prematurities (interferences) that cause the mandible to shift forward during closure, creating an anterior crossbite. In such cases, the primary issue is dental, not skeletal.
⢠The most appropriate initial treatment is the removal of occlusal prematurities to eliminate the forward mandibular posture and correct the anterior crossbite. This conservative approach can often resolve malocclusion without the need for more invasive interventions.REFERENCE
Pseudo Class III malocclusion – Saudi Medical Journal 2016 Apr – 37(4), 450-456 -
Question 7 of 150
7. Question
A patient with a history of severe penicillin allergy requires antibiotic prophylaxis before a dental procedure. Which of the following is the correct antibiotic and dosage to prescribe?
Correct
ANSWER
Clindamycin 600mgOTHER OPTIONS
⢠Amoxicillin 2g – It is the standard dosage for patients who are not allergic to penicillin, so it is not appropriate for this patient.
⢠Clindamycin 300mg – It is below the recommended dosage for prophylaxis; 600mg is the correct dose.
⢠Azithromycin 250mg – It can be used as an alternative in penicillin-allergic patients, but the standard dosage for prophylaxis is 500mg, not 250mg.SYNOPSIS
⢠For patients who are allergic to penicillin and require antibiotic prophylaxis before a dental procedure, the recommended alternative is Clindamycin 600mg taken orally 30 to 60 minutes before the procedure.
⢠This dosage is effective in preventing bacterial endocarditis and other infections in patients who are at risk.REFERENCE
Preoperative Antibiotic Prophylaxis – Stat pearlsIncorrect
ANSWER
Clindamycin 600mgOTHER OPTIONS
⢠Amoxicillin 2g – It is the standard dosage for patients who are not allergic to penicillin, so it is not appropriate for this patient.
⢠Clindamycin 300mg – It is below the recommended dosage for prophylaxis; 600mg is the correct dose.
⢠Azithromycin 250mg – It can be used as an alternative in penicillin-allergic patients, but the standard dosage for prophylaxis is 500mg, not 250mg.SYNOPSIS
⢠For patients who are allergic to penicillin and require antibiotic prophylaxis before a dental procedure, the recommended alternative is Clindamycin 600mg taken orally 30 to 60 minutes before the procedure.
⢠This dosage is effective in preventing bacterial endocarditis and other infections in patients who are at risk.REFERENCE
Preoperative Antibiotic Prophylaxis – Stat pearls -
Question 8 of 150
8. Question
A 14-year-old patient presents with symptoms of fever, sore throat, and swollen lymph nodes. A Paul-Bunnell test returns positive. Which of the following is the most likely causative organism for this condition?
Correct
ANSWER
Epstein-Barr virusOTHER OPTIONS
⢠Not applicableSYNOPSIS
⢠The Paul-Bunnell test, also known as the heterophile antibody test, is used to diagnose infectious mononucleosis, which is commonly caused by the Epstein-Barr virus (EBV).
⢠A positive result on this test typically indicates the presence of heterophile antibodies, which are associated with EBV infection.REFERENCE
Journal of Clinical Pathology – Paul-Bunnell test in infectious mononucleosis and other diseasesIncorrect
ANSWER
Epstein-Barr virusOTHER OPTIONS
⢠Not applicableSYNOPSIS
⢠The Paul-Bunnell test, also known as the heterophile antibody test, is used to diagnose infectious mononucleosis, which is commonly caused by the Epstein-Barr virus (EBV).
⢠A positive result on this test typically indicates the presence of heterophile antibodies, which are associated with EBV infection.REFERENCE
Journal of Clinical Pathology – Paul-Bunnell test in infectious mononucleosis and other diseases -
Question 9 of 150
9. Question
To minimize the risk of black triangles between teeth, the distance from the contact point to the alveolar bone should be
Correct
ANSWER
Less than 5mmOTHER OPTIONS
⢠Refer SynopsisSYNOPSIS
⢠To minimize the risk of black triangles between teeth, the distance from the contact point to the alveolar bone should beĀ 5 mm or less.Ā
⢠The iconic study by Tarnow et al. who produced the 5 mm rule, states that when the distance from the contact point to the interproximal osseous crest is 5 mm or less, there is complete fill of the gingival embrasures with an interdental papilla.
⢠For every 1 mm above 5 mm, the chance of complete fill is progressively reduced by 50 percent.
⢠For square-shaped teeth with wide contact points, the chances of black triangles is minimal compared with triangular teeth having narrow, more incisally positioned contact points.REFERENCE
Black triangle dilemma and its management in esthetic dentistry – Dental Research Journal 2013 JuneIncorrect
ANSWER
Less than 5mmOTHER OPTIONS
⢠Refer SynopsisSYNOPSIS
⢠To minimize the risk of black triangles between teeth, the distance from the contact point to the alveolar bone should beĀ 5 mm or less.Ā
⢠The iconic study by Tarnow et al. who produced the 5 mm rule, states that when the distance from the contact point to the interproximal osseous crest is 5 mm or less, there is complete fill of the gingival embrasures with an interdental papilla.
⢠For every 1 mm above 5 mm, the chance of complete fill is progressively reduced by 50 percent.
⢠For square-shaped teeth with wide contact points, the chances of black triangles is minimal compared with triangular teeth having narrow, more incisally positioned contact points.REFERENCE
Black triangle dilemma and its management in esthetic dentistry – Dental Research Journal 2013 June -
Question 10 of 150
10. Question
A 21 year old male patient came to your clinic with extrusion of 11 followed by a trauma happened 30 minutes ago. What treatment will you recommended?
Correct
ANSWER
Repositioned into socket with gentle pressure and splintOTHER OPTIONS
Not applicableSYNOPSIS
⢠Extrusion is partial displacement of tooth from its alveolar socket.
⢠Extruded teeth should be forced back into socket as soon as possible by means of gentle pressure, after accident and preferably done after anesthetizing the region.
⢠When extruded teeth are repositioned they are quite likely to be retained for a lifetime.
⢠Then the tooth should be splinted with a flexible splint for a period of 2 weeks.
⢠Tooth should be tested for vitality once a month and if non vital, perform RCT.REFERENCE
Grossman’s Endodontic Practise – 13th Edition Page No 434.Incorrect
ANSWER
Repositioned into socket with gentle pressure and splintOTHER OPTIONS
Not applicableSYNOPSIS
⢠Extrusion is partial displacement of tooth from its alveolar socket.
⢠Extruded teeth should be forced back into socket as soon as possible by means of gentle pressure, after accident and preferably done after anesthetizing the region.
⢠When extruded teeth are repositioned they are quite likely to be retained for a lifetime.
⢠Then the tooth should be splinted with a flexible splint for a period of 2 weeks.
⢠Tooth should be tested for vitality once a month and if non vital, perform RCT.REFERENCE
Grossman’s Endodontic Practise – 13th Edition Page No 434. -
Question 11 of 150
11. Question
A 48-year-old patient reports with pain on chewing in the lower right back tooth (46). The tooth has a history of root canal treatment 2 years ago. On clinical examination, the tooth is tender on percussion, and a narrow, isolated periodontal pocket is present on the distal root. Radiograph reveals a J-shaped radiolucency extending along the root surface.
Correct
ANSWER
Vertical Root FractureOTHER OPTIONS
⢠NilSYNOPSIS
⢠According to the American Association of Endodontists, A vertical root fracture is a longitudinally oriented fracture of the root that originates from the apex and propagates to the coronal part
⢠Possible symptoms include
– General discomfort or a sharp twinge of pain when biting or chewing,
– A visible crack during a dental examination,
– Swelling, and
– Abscess, or boil near the tooth indicating an infection,
– An isolated pocket in the gum next to the tooth in question.
⢠The prognosis of the root with VRF is poor therefore tooth extraction and root amputation are usually the only treatment options.REFERENCE
Treatment of a vertical root fracture using dual-curing resin cement- a case report-Iranian Dental JournalIncorrect
ANSWER
Vertical Root FractureOTHER OPTIONS
⢠NilSYNOPSIS
⢠According to the American Association of Endodontists, A vertical root fracture is a longitudinally oriented fracture of the root that originates from the apex and propagates to the coronal part
⢠Possible symptoms include
– General discomfort or a sharp twinge of pain when biting or chewing,
– A visible crack during a dental examination,
– Swelling, and
– Abscess, or boil near the tooth indicating an infection,
– An isolated pocket in the gum next to the tooth in question.
⢠The prognosis of the root with VRF is poor therefore tooth extraction and root amputation are usually the only treatment options.REFERENCE
Treatment of a vertical root fracture using dual-curing resin cement- a case report-Iranian Dental Journal -
Question 12 of 150
12. Question
A patient with abrasion on the teeth and gingivitis complains of bleeding on brushing. On examination, the patient’s oral hygiene measures show brushing twice and using floss too. What change has to be advised for him for better periodontal health?
Correct
ANSWER
Change the brushing techniqueOTHER OPTIONS
⢠Not applicableSYNOPSIS
⢠Changing the brushing technique can improve the condition of this patient
⢠Bass Method or Sulcular cleaning method is the most accepted and effective method for the removal of dental plaque present adjacent to and underneath the gingival margin
⢠Indications
– Open interproximal areas
– Cervical areas beneath the height of the contour of enamel
– Exposed root surface
– Recommended for patients with or without periodontal involvement
⢠Technique
– The bristles are held at a 45-degree angle toward the gum line.
– Very slight pressure and vibratory motions are made so that the bristles go slightly beneath the gum line.
– Only small groups of teeth can be done at a time.
– Once an area is complete, move on to the next set of teeth.REFERENCE
Carranza’s Clinical PeriodontologyIncorrect
ANSWER
Change the brushing techniqueOTHER OPTIONS
⢠Not applicableSYNOPSIS
⢠Changing the brushing technique can improve the condition of this patient
⢠Bass Method or Sulcular cleaning method is the most accepted and effective method for the removal of dental plaque present adjacent to and underneath the gingival margin
⢠Indications
– Open interproximal areas
– Cervical areas beneath the height of the contour of enamel
– Exposed root surface
– Recommended for patients with or without periodontal involvement
⢠Technique
– The bristles are held at a 45-degree angle toward the gum line.
– Very slight pressure and vibratory motions are made so that the bristles go slightly beneath the gum line.
– Only small groups of teeth can be done at a time.
– Once an area is complete, move on to the next set of teeth.REFERENCE
Carranza’s Clinical Periodontology -
Question 13 of 150
13. Question
A 25-year-old techie presents with tooth sensitivity and enamel loss on the palatal surfaces of maxillary anterior teeth. The patient admits to frequent consumption of acidic beverages. What type of tooth wear is most likely responsible for these clinical findings?
Correct
ANSWER
ErosionOTHER OPTIONS
⢠Attrition – Dental attrition isĀ a type of tooth wear caused by tooth-to-tooth contact, resulting in loss of tooth tissue, usually starting at the incisal or occlusal surfaces.Ā
⢠Abrasion – Abrasion isĀ the wearing away of tooth surface caused by friction or a mechanical process
⢠Abfraction – Abfraction is the loss of tooth structure from flexural forces.SYNOPSIS
⢠Erosion is the wearing away of the tooth surface by an acid, which dissolves the enamel and the dentine.
⢠The causes may include dietary, acid regurgitation, environmental, etc
⢠The erosion is seen as a light yellow patch on the tooth. This indicates that the enamel has been dissolved and the sensitive dentine under the enamel has been exposed.
⢠This exposure of the dentine will result in pain from sweet, hot, and cold food and drinks. If the erosion penetrates deeper and reaches the nerves and blood vessels, the pain can become severe.
⢠Treatment includes
– Treating the underlying medical disorder or disease.
– Changing the patientās lifestyle to avoid the food or beverage.
– Applying fluoride gels or varnishes to the teeth.
– Drinking milk or using other dairy products.
– Using a neutralizing agent such as antacid tablets.
– Applying dentine bonding agents to areas of exposed dentin.REFERENCE
Shafer’s textbook of Oral PathologyIncorrect
ANSWER
ErosionOTHER OPTIONS
⢠Attrition – Dental attrition isĀ a type of tooth wear caused by tooth-to-tooth contact, resulting in loss of tooth tissue, usually starting at the incisal or occlusal surfaces.Ā
⢠Abrasion – Abrasion isĀ the wearing away of tooth surface caused by friction or a mechanical process
⢠Abfraction – Abfraction is the loss of tooth structure from flexural forces.SYNOPSIS
⢠Erosion is the wearing away of the tooth surface by an acid, which dissolves the enamel and the dentine.
⢠The causes may include dietary, acid regurgitation, environmental, etc
⢠The erosion is seen as a light yellow patch on the tooth. This indicates that the enamel has been dissolved and the sensitive dentine under the enamel has been exposed.
⢠This exposure of the dentine will result in pain from sweet, hot, and cold food and drinks. If the erosion penetrates deeper and reaches the nerves and blood vessels, the pain can become severe.
⢠Treatment includes
– Treating the underlying medical disorder or disease.
– Changing the patientās lifestyle to avoid the food or beverage.
– Applying fluoride gels or varnishes to the teeth.
– Drinking milk or using other dairy products.
– Using a neutralizing agent such as antacid tablets.
– Applying dentine bonding agents to areas of exposed dentin.REFERENCE
Shafer’s textbook of Oral Pathology -
Question 14 of 150
14. Question
A 40-year-old patient with a history of rheumatic heart disease presents with fever, malaise, and a new heart murmur. Blood cultures reveal bacteria commonly found in the oral cavity. Which of the following organisms is most commonly associated with subacute infective endocarditis?
Correct
ANSWER
Streptococcus viridansOTHER OPTIONS
⢠Not applicableSYNOPSIS
⢠Viridans streptococci (especially Streptococcus sanguis, S. mitis, S. mutans) are the most common cause of subacute bacterial endocarditis, often following dental procedures in susceptible patients.
⢠Staphylococcus aureus – causes acute bacterial endocarditis, particularly in IV drug users. It is the reason for acute infective endocarditis.REFERENCE
Carranzaās Clinical Periodontology, 13th EditionIncorrect
ANSWER
Streptococcus viridansOTHER OPTIONS
⢠Not applicableSYNOPSIS
⢠Viridans streptococci (especially Streptococcus sanguis, S. mitis, S. mutans) are the most common cause of subacute bacterial endocarditis, often following dental procedures in susceptible patients.
⢠Staphylococcus aureus – causes acute bacterial endocarditis, particularly in IV drug users. It is the reason for acute infective endocarditis.REFERENCE
Carranzaās Clinical Periodontology, 13th Edition -
Question 15 of 150
15. Question
60-year-old female, presents with dry mouth and dry eyes. Lab tests reveal positive anti-SSA (Ro) and anti-SSB (La) antibodies, and salivary gland biopsy shows lymphocytic infiltration. Which additional diagnostic test should be considered for confirming Sjƶgren syndrome?
Correct
ANSWER
Minor salivary gland biopsyOTHER OPTIONS
⢠Not applicableSYNOPSIS
⢠Sjögren syndrome diagnosis is confirmed with a positive salivary gland biopsy showing focal lymphocytic sialadenitis.
⢠Positive anti-SSA and anti-SSB antibodies further support the diagnosis.REFERENCE
Incorrect
ANSWER
Minor salivary gland biopsyOTHER OPTIONS
⢠Not applicableSYNOPSIS
⢠Sjögren syndrome diagnosis is confirmed with a positive salivary gland biopsy showing focal lymphocytic sialadenitis.
⢠Positive anti-SSA and anti-SSB antibodies further support the diagnosis.REFERENCE
-
Question 16 of 150
16. Question
Primary teeth which had trauma shows the change in color from white to yellow. What should you tell the parents?
Correct
ANSWER
Calcification of pulpOTHER OPTIONS
⢠Refer SynopsisSYNOPSIS
⢠The more the tooth moves out of the alveolar socket, the higher the chances of disruption of the blood supply and development of pulp necrosis.
⢠Two main mechanisms can explain how the pulp of injured primary teeth becomes necrotic.
1) Infection of the pulp in cases of untreated crown fracture with pulp exposure and
2) Interrupted blood supply to the pulp through the apex in cases of luxation injury leading to ischemia.
⢠Crown discoloration is the external expression of changes in the pulp-dentin complex that become visible through the almost transparent enamel.
⢠Crown discoloration of primary incisors is a common posttraumatic finding and is often the only evidence of trauma to the tooth.
⢠Pulp canal obliteration (PCO) presents itself as a frequent pulp complication in traumatized primary teeth, characterized as a progressive deposition of hard tissue in the root canal.
⢠Yellow discoloration of primary incisors can be seen when the dentin is thick and the pulp chamber narrower than usual.
⢠This condition is termed pulp canal obliteration- PCO.
⢠Although PCO is a pathologic process, it has no known deleterious effects and therefore does not necessitate any treatment except follow-up.REFERENCE
Pediatric Dentistry Arthur.J.Nowak Page 238Incorrect
ANSWER
Calcification of pulpOTHER OPTIONS
⢠Refer SynopsisSYNOPSIS
⢠The more the tooth moves out of the alveolar socket, the higher the chances of disruption of the blood supply and development of pulp necrosis.
⢠Two main mechanisms can explain how the pulp of injured primary teeth becomes necrotic.
1) Infection of the pulp in cases of untreated crown fracture with pulp exposure and
2) Interrupted blood supply to the pulp through the apex in cases of luxation injury leading to ischemia.
⢠Crown discoloration is the external expression of changes in the pulp-dentin complex that become visible through the almost transparent enamel.
⢠Crown discoloration of primary incisors is a common posttraumatic finding and is often the only evidence of trauma to the tooth.
⢠Pulp canal obliteration (PCO) presents itself as a frequent pulp complication in traumatized primary teeth, characterized as a progressive deposition of hard tissue in the root canal.
⢠Yellow discoloration of primary incisors can be seen when the dentin is thick and the pulp chamber narrower than usual.
⢠This condition is termed pulp canal obliteration- PCO.
⢠Although PCO is a pathologic process, it has no known deleterious effects and therefore does not necessitate any treatment except follow-up.REFERENCE
Pediatric Dentistry Arthur.J.Nowak Page 238 -
Question 17 of 150
17. Question
What will you do for an 8 year old child patient with obliteration in the central permanent incisor?
Correct
ANSWER
Careful monitoringOTHER OPTIONS
⢠No need for any particular treatment until symptomatic.SYNOPSIS
⢠Occurrence of obliteration in permanent teeth increased over time, with most cases diagnosed more than 3 years following the trauma.
⢠Although pulp canal obliteration is a pathologic process, it has no known deleterious effects and therefore does not necessitate any treatment in primary or permanent teeth except follow-up.
⢠In teeth with an open apex, there is a greater possibility of maintaining pulp vitality or revascularization of the neurovascular supply compared to mature teeth, due to the intense cell activity capable of promoting the defense and regeneration of affected tissues.REFERENCE
Pulp canal obliteration to primary and permanent teeth following trauma-Bianca SantosIncorrect
ANSWER
Careful monitoringOTHER OPTIONS
⢠No need for any particular treatment until symptomatic.SYNOPSIS
⢠Occurrence of obliteration in permanent teeth increased over time, with most cases diagnosed more than 3 years following the trauma.
⢠Although pulp canal obliteration is a pathologic process, it has no known deleterious effects and therefore does not necessitate any treatment in primary or permanent teeth except follow-up.
⢠In teeth with an open apex, there is a greater possibility of maintaining pulp vitality or revascularization of the neurovascular supply compared to mature teeth, due to the intense cell activity capable of promoting the defense and regeneration of affected tissues.REFERENCE
Pulp canal obliteration to primary and permanent teeth following trauma-Bianca Santos -
Question 18 of 150
18. Question
A patient with a history of trauma on tooth 11, half an hour ago showed slight pulpal exposure on periapical x-ray examination with open apex. What is the treatment of choice?
Correct
ANSWER
DPCOTHER OPTIONS
⢠Pulpotomy – is carried out when there is a large pulp exposure.
⢠Indirect pulp capping – is done with deep dentinal caries that don’t reach the pulp.
⢠Apexification – Done in immature nonvital tooth with pulp exposure.SYNOPSIS
⢠The direct pulp cap is only indicated in small exposures that can be treated within a few hours of the injury.
⢠The chances for pulp healing decrease if the tissue is inflamed, has formed a clot, or is contaminated with foreign materials.
⢠The objective, then, is to preserve vital pulp tissue that is free of inflammation and physiologically walled off by a calcific barrier.
⢠MTA, is the currently recommended capping agent.
⢠MTA is less toxic and produces less pulpal inflammation compared to Calcium Hydroxide. MTA is superior in dentinogenic processes and more predictable in hard tissue barrier formation.REFERENCE
Pediatric Dentistry Arthur.J.Nowak Page 499Incorrect
ANSWER
DPCOTHER OPTIONS
⢠Pulpotomy – is carried out when there is a large pulp exposure.
⢠Indirect pulp capping – is done with deep dentinal caries that don’t reach the pulp.
⢠Apexification – Done in immature nonvital tooth with pulp exposure.SYNOPSIS
⢠The direct pulp cap is only indicated in small exposures that can be treated within a few hours of the injury.
⢠The chances for pulp healing decrease if the tissue is inflamed, has formed a clot, or is contaminated with foreign materials.
⢠The objective, then, is to preserve vital pulp tissue that is free of inflammation and physiologically walled off by a calcific barrier.
⢠MTA, is the currently recommended capping agent.
⢠MTA is less toxic and produces less pulpal inflammation compared to Calcium Hydroxide. MTA is superior in dentinogenic processes and more predictable in hard tissue barrier formation.REFERENCE
Pediatric Dentistry Arthur.J.Nowak Page 499 -
Question 19 of 150
19. Question
What is the most prevalent bacteria found in saliva?
Correct
ANSWER
StreptococcusOTHER OPTIONS
⢠Actinomyces – Present in oral biofilm and gingival crevices, but not the most prevalent in saliva.
⢠Staphylococcus – can be found, especially S. aureus and S. epidermidis, but in much lower proportions.SYNOPSIS
⢠Saliva harbors a wide range of microorganisms, but the predominant bacteria are Streptococcus species (especially Streptococcus salivarius, Streptococcus mitis, Streptococcus mutans).REFERENCE
Carranzaās Clinical Periodontology, 13th EditionIncorrect
ANSWER
StreptococcusOTHER OPTIONS
⢠Actinomyces – Present in oral biofilm and gingival crevices, but not the most prevalent in saliva.
⢠Staphylococcus – can be found, especially S. aureus and S. epidermidis, but in much lower proportions.SYNOPSIS
⢠Saliva harbors a wide range of microorganisms, but the predominant bacteria are Streptococcus species (especially Streptococcus salivarius, Streptococcus mitis, Streptococcus mutans).REFERENCE
Carranzaās Clinical Periodontology, 13th Edition -
Question 20 of 150
20. Question
A 30 year old female patient came with skeletal class III and a concave profile. What treatment will you suggest?
Correct
ANSWER
Maxillary advancement and mandible setbackOTHER OPTIONS
Not applicableSYNOPSIS
⢠Severe class III after growth completion is treated by surgical corrective procedures.
⢠Class III due to maxillary deficiency is treated by maxillary advancement procedures like LeFort I osteotomy.
⢠Class III due to mandibular excess is treated by mandibular setback procedures.
⢠A combination of both is also indicated.REFERENCE
Orthodontics The Art and Science – 6th Edition Page No 518.Incorrect
ANSWER
Maxillary advancement and mandible setbackOTHER OPTIONS
Not applicableSYNOPSIS
⢠Severe class III after growth completion is treated by surgical corrective procedures.
⢠Class III due to maxillary deficiency is treated by maxillary advancement procedures like LeFort I osteotomy.
⢠Class III due to mandibular excess is treated by mandibular setback procedures.
⢠A combination of both is also indicated.REFERENCE
Orthodontics The Art and Science – 6th Edition Page No 518. -
Question 21 of 150
21. Question
A child patient came to you with gray discoloration of the deciduous incisor shows dilation of follicle of the permanent successor on radiographic examination. What will be your treatment plan?
Correct
ANSWER
Extract the deciduous toothOTHER OPTIONS
⢠Observe over time – If a tooth is asymptomatic and does not pose any threat to permanent tooth bud.
⢠Pulpotomy – Done in cases where the coronal pulp is exposed.SYNOPSIS
⢠Due to the close relationship between the apex of primary teeth and the bud of permanent teeth, any lesion to the primary dentition may influence the eruption of the permanent teeth.
⢠The severity of sequelae depends on the patientās age, the degree of root reabsorption, the type and extent of the trauma, and the degree of development of the permanent successor at the time of trauma.
⢠Intrusion and avulsion of primary teeth are considered the types of traumas that produce the greatest number of alterations in the development of permanent teeth, according to various studies.
⢠Here the permanent tooth follicle shows dilation, so its better to extract the deciduous tooth.
⢠The main consequences of primary tooth trauma in the development of permanent teeth are enamel discoloration, enamel hypoplasia, coronal dilaceration, root dilaceration, odontoma-like malformations, and alterations in eruption.
⢠In cases of asymptomatic tooth, constant follow-up, with complementary tests, such as radiographs, and appropriate clinical interventions, can minimize or even prevent damage to the successor tooth.REFERENCE
Pediatric Dentistry Arthur.J.Nowak Page 238Incorrect
ANSWER
Extract the deciduous toothOTHER OPTIONS
⢠Observe over time – If a tooth is asymptomatic and does not pose any threat to permanent tooth bud.
⢠Pulpotomy – Done in cases where the coronal pulp is exposed.SYNOPSIS
⢠Due to the close relationship between the apex of primary teeth and the bud of permanent teeth, any lesion to the primary dentition may influence the eruption of the permanent teeth.
⢠The severity of sequelae depends on the patientās age, the degree of root reabsorption, the type and extent of the trauma, and the degree of development of the permanent successor at the time of trauma.
⢠Intrusion and avulsion of primary teeth are considered the types of traumas that produce the greatest number of alterations in the development of permanent teeth, according to various studies.
⢠Here the permanent tooth follicle shows dilation, so its better to extract the deciduous tooth.
⢠The main consequences of primary tooth trauma in the development of permanent teeth are enamel discoloration, enamel hypoplasia, coronal dilaceration, root dilaceration, odontoma-like malformations, and alterations in eruption.
⢠In cases of asymptomatic tooth, constant follow-up, with complementary tests, such as radiographs, and appropriate clinical interventions, can minimize or even prevent damage to the successor tooth.REFERENCE
Pediatric Dentistry Arthur.J.Nowak Page 238 -
Question 22 of 150
22. Question
Direct pulp capping is done in which of the following
Correct
ANSWER
Permanent molarĀOTHER OPTIONS
⢠Direct pulp capping is contraindicated in primary teeth because of the risk of developing internal root resorption.SYNOPSIS
⢠Direct Pulp Capping is the placement of a medicament or non-medicated material on an exposed pulp that is occurred in course of excavating the last portions of deep dentinal caries or because of trauma.
⢠Indications of Direct Pulp Capping
Pulp-capping procedures should be limited to asymptomatic (absence of pain)permanent teeth, vital young permanent teeth, with the possible exception of discomfort caused by the intake of food, the teeth should have
– True pinpoint exposure (small mechanical exposure less than 1 mm) surrounded by sound dentin produced accidentally by trauma during cavity preparation.
– Mechanical or carious exposures less than 1 mm, in an asymptomatic vital young permanent tooth.
– The exposure site should have bright red hemorrhage easily controlled by dry cotton pellet with minimal pressure.
– Lack of bleeding at the exposure site (the amount of bleeding considered normal in the absence of a hyperemic or an inflamed pulp.
⢠Contraindications
1) Curiously exposed deciduous teeth
2) Spontaneous pain or severe toothache at night
3) Swelling
4) Fistula
5) Tenderness to percussion
6) Pathologic mobility
7) Root resorption-external or internal
8) Periapical or interradicular radiolucency
9) Pulp calcifications
10) Profuse hemorrhage from the exposure site
11) Pus or exudate from exposure site.
⢠Presently, the materials most commonly used are calcium hydroxide, emdogain, and mineral trioxide aggregate – MTA.REFERENCE
Pediatric Dentistry Arthur.J.Nowak Page 337, 338Incorrect
ANSWER
Permanent molarĀOTHER OPTIONS
⢠Direct pulp capping is contraindicated in primary teeth because of the risk of developing internal root resorption.SYNOPSIS
⢠Direct Pulp Capping is the placement of a medicament or non-medicated material on an exposed pulp that is occurred in course of excavating the last portions of deep dentinal caries or because of trauma.
⢠Indications of Direct Pulp Capping
Pulp-capping procedures should be limited to asymptomatic (absence of pain)permanent teeth, vital young permanent teeth, with the possible exception of discomfort caused by the intake of food, the teeth should have
– True pinpoint exposure (small mechanical exposure less than 1 mm) surrounded by sound dentin produced accidentally by trauma during cavity preparation.
– Mechanical or carious exposures less than 1 mm, in an asymptomatic vital young permanent tooth.
– The exposure site should have bright red hemorrhage easily controlled by dry cotton pellet with minimal pressure.
– Lack of bleeding at the exposure site (the amount of bleeding considered normal in the absence of a hyperemic or an inflamed pulp.
⢠Contraindications
1) Curiously exposed deciduous teeth
2) Spontaneous pain or severe toothache at night
3) Swelling
4) Fistula
5) Tenderness to percussion
6) Pathologic mobility
7) Root resorption-external or internal
8) Periapical or interradicular radiolucency
9) Pulp calcifications
10) Profuse hemorrhage from the exposure site
11) Pus or exudate from exposure site.
⢠Presently, the materials most commonly used are calcium hydroxide, emdogain, and mineral trioxide aggregate – MTA.REFERENCE
Pediatric Dentistry Arthur.J.Nowak Page 337, 338 -
Question 23 of 150
23. Question
Which is the best treatment of choice for carious exposure of a primary molar in a 3 years old child who complaints of toothache during and after food taking?
Correct
ANSWER
Formocresol pulpotomyOTHER OPTIONS
⢠DPC – not indicated in carious exposures of primary teeth.
⢠CaOH pulpotomy – induces internal root resorption.SYNOPSIS
⢠When the carious process has reached the pulp or in incidences of direct pulpal exposure during excavation of a carious lesion, the pulpotomy procedure is indicated and is the treatment of choice.
⢠Pulpotomy techniques using formocresol on primary teeth have been a standard in pediatric dentistry for decades.
⢠The indication was primarily the presence of a carious exposure and a desire to retain the primary tooth until exfoliation.
⢠Contraindications for the formocresol pulpotomy are
(1) teeth with history of spontaneous pain
(2) pain percussion due to periapical involvement,
(3) suppuration in the canals,
(4) the presence of a parulis or sinus tract,
(5) hemorrhage which cannot be controlled in five minutes after extirpation of the coronal pulp,
(6) evidence of periapical or furcal pathology, and
(7) presence of necrotic pulp in the chamber.REFERENCE
Pediatric Dentistry Arthur.J.Nowak Page 340Incorrect
ANSWER
Formocresol pulpotomyOTHER OPTIONS
⢠DPC – not indicated in carious exposures of primary teeth.
⢠CaOH pulpotomy – induces internal root resorption.SYNOPSIS
⢠When the carious process has reached the pulp or in incidences of direct pulpal exposure during excavation of a carious lesion, the pulpotomy procedure is indicated and is the treatment of choice.
⢠Pulpotomy techniques using formocresol on primary teeth have been a standard in pediatric dentistry for decades.
⢠The indication was primarily the presence of a carious exposure and a desire to retain the primary tooth until exfoliation.
⢠Contraindications for the formocresol pulpotomy are
(1) teeth with history of spontaneous pain
(2) pain percussion due to periapical involvement,
(3) suppuration in the canals,
(4) the presence of a parulis or sinus tract,
(5) hemorrhage which cannot be controlled in five minutes after extirpation of the coronal pulp,
(6) evidence of periapical or furcal pathology, and
(7) presence of necrotic pulp in the chamber.REFERENCE
Pediatric Dentistry Arthur.J.Nowak Page 340 -
Question 24 of 150
24. Question
Pulpitis in deciduous teeth on radiographic examination is seen in relation to?
Correct
ANSWER
Furcation areaOTHER OPTIONS
⢠Lateral to root – periodontal infections are seen as radiolucent areas lateral to rootSYNOPSIS
⢠Infected primary molar teeth display radiographic bony changes that are different from those of permanent molars.
⢠Periapical radiographs are generally used to detect the effects that necrotic infected pulpal tissue have on the lamina dura and bone surrounding the root apices of permanent teeth and on the furcal area between the roots of primary molar teeth.
⢠The posterior bitewing radiographs’ greatest use is in the detection of furcal radiolucencies caused by pulpally infected primary molar teeth.
⢠It is particularly useful in viewing the root areas of maxillary primary molars where developing premolars obscure the furcation in the periapical radiograph.REFERENCE
Radiographic Interpretation of Infected Primary Molar Teeth-KeithIncorrect
ANSWER
Furcation areaOTHER OPTIONS
⢠Lateral to root – periodontal infections are seen as radiolucent areas lateral to rootSYNOPSIS
⢠Infected primary molar teeth display radiographic bony changes that are different from those of permanent molars.
⢠Periapical radiographs are generally used to detect the effects that necrotic infected pulpal tissue have on the lamina dura and bone surrounding the root apices of permanent teeth and on the furcal area between the roots of primary molar teeth.
⢠The posterior bitewing radiographs’ greatest use is in the detection of furcal radiolucencies caused by pulpally infected primary molar teeth.
⢠It is particularly useful in viewing the root areas of maxillary primary molars where developing premolars obscure the furcation in the periapical radiograph.REFERENCE
Radiographic Interpretation of Infected Primary Molar Teeth-Keith -
Question 25 of 150
25. Question
Which of the following is the only antibiotic in periodontal therapy to which all strains of actinomycetemcomitans are susceptible?
Correct
ANSWER
CiprofloxacinOTHER OPTIONS
⢠High dose penicillin – Not reliably effective due to β-lactamase producing strains.
⢠Tetracycline – Useful in periodontal therapy but not effective against all strains.
⢠Macrolides – E.g., erythromycin, azithromycin) ā resistance is common among A. actinomycetemcomitans.SYNOPSIS
⢠Aggregatibacter actinomycetemcomitans is an important pathogen in aggressive periodontitis.
⢠Studies have shown that all strains of A. actinomycetemcomitans are susceptible to ciprofloxacin, making it unique among antibiotics used in periodontal therapy.REFERENCE
Carranzaās Clinical Periodontology, 13th EditionIncorrect
ANSWER
CiprofloxacinOTHER OPTIONS
⢠High dose penicillin – Not reliably effective due to β-lactamase producing strains.
⢠Tetracycline – Useful in periodontal therapy but not effective against all strains.
⢠Macrolides – E.g., erythromycin, azithromycin) ā resistance is common among A. actinomycetemcomitans.SYNOPSIS
⢠Aggregatibacter actinomycetemcomitans is an important pathogen in aggressive periodontitis.
⢠Studies have shown that all strains of A. actinomycetemcomitans are susceptible to ciprofloxacin, making it unique among antibiotics used in periodontal therapy.REFERENCE
Carranzaās Clinical Periodontology, 13th Edition -
Question 26 of 150
26. Question
A patient came with nasopharyngeal carcinoma reported for dental check up before treatment of carcinoma. When will you extract a teeth in this case?
Correct
ANSWER
Grade III mobilityOTHER OPTIONS
Not applicableSYNOPSIS
⢠Among the various periodontal prognosis systems, the one most widely used was proposed by McGuire and Nunn in 1996.
⢠It consists of five categories that include good, fair, poor, questionable and hopeless.
⢠In poor prognosis, it includes 50 percent attachment loss and grade II furcation.
⢠In questionable prognosis, more than 50 percent attachment loss and grade III furcation, poor crown root ratio, class II or more mobility.
⢠Hopeless includes severe attachment loss.
⢠Extraction may be preferred in this case.REFERENCE
Strategies for Periodontal Risk Assessment and Prognosis by Decisions in Dentistry.Incorrect
ANSWER
Grade III mobilityOTHER OPTIONS
Not applicableSYNOPSIS
⢠Among the various periodontal prognosis systems, the one most widely used was proposed by McGuire and Nunn in 1996.
⢠It consists of five categories that include good, fair, poor, questionable and hopeless.
⢠In poor prognosis, it includes 50 percent attachment loss and grade II furcation.
⢠In questionable prognosis, more than 50 percent attachment loss and grade III furcation, poor crown root ratio, class II or more mobility.
⢠Hopeless includes severe attachment loss.
⢠Extraction may be preferred in this case.REFERENCE
Strategies for Periodontal Risk Assessment and Prognosis by Decisions in Dentistry. -
Question 27 of 150
27. Question
A 32-year-old patient presents with a deep periodontal pocket associated with a non-vital mandibular molar. Radiograph reveals periapical radiolucency with periodontal bone loss extending to the apex. This is diagnosed as an endo-perio lesion. Which type of bacteria is most commonly involved?
Correct
ANSWER
Gram-negative anaerobic rodsOTHER OPTIONS
⢠Gram-positive facultative cocci – (e.g., streptococci, enterococci) are common in early caries or pulpal infections, but not predominant in endo-perio.
⢠Spirochetes – Spirochetes are seen in necrotizing periodontal diseases, not the main flora in endo-perio lesions.
⢠Viridans streptococci – Viridans streptococci are mainly early colonizers in dental plaque, not responsible for advanced endo-perio pathology.SYNOPSIS
⢠Endo-perio lesions involve infection spreading between the pulpal and periodontal tissues.
⢠The microbiota is typically mixed, but Gram-negative anaerobic rods predominate (e.g., Porphyromonas gingivalis, Prevotella intermedia, Fusobacterium nucleatum).
⢠These organisms produce endotoxins and enzymes that contribute to tissue destruction.REFERENCE
Carranzaās Clinical Periodontology, 13th EditionIncorrect
ANSWER
Gram-negative anaerobic rodsOTHER OPTIONS
⢠Gram-positive facultative cocci – (e.g., streptococci, enterococci) are common in early caries or pulpal infections, but not predominant in endo-perio.
⢠Spirochetes – Spirochetes are seen in necrotizing periodontal diseases, not the main flora in endo-perio lesions.
⢠Viridans streptococci – Viridans streptococci are mainly early colonizers in dental plaque, not responsible for advanced endo-perio pathology.SYNOPSIS
⢠Endo-perio lesions involve infection spreading between the pulpal and periodontal tissues.
⢠The microbiota is typically mixed, but Gram-negative anaerobic rods predominate (e.g., Porphyromonas gingivalis, Prevotella intermedia, Fusobacterium nucleatum).
⢠These organisms produce endotoxins and enzymes that contribute to tissue destruction.REFERENCE
Carranzaās Clinical Periodontology, 13th Edition -
Question 28 of 150
28. Question
What is the typical minimum bone height required for successful dental implant placement in the posterior mandible?
Correct
ANSWER
8-10 mmOTHER OPTIONS
⢠Not applicableSYNOPSIS
⢠In the posterior mandible, a minimum bone height of 8-10 mm is typically required for successful dental implant placement. This ensures that the implant has sufficient bone support for stability.REFERENCE
Contemporary Implant Dentistry – Carl E MischIncorrect
ANSWER
8-10 mmOTHER OPTIONS
⢠Not applicableSYNOPSIS
⢠In the posterior mandible, a minimum bone height of 8-10 mm is typically required for successful dental implant placement. This ensures that the implant has sufficient bone support for stability.REFERENCE
Contemporary Implant Dentistry – Carl E Misch -
Question 29 of 150
29. Question
Which of the following disinfectants is most commonly used to sterilize dental impressions, such as alginate, before sending them to the lab?
Correct
ANSWER
IodophorOTHER OPTIONS
⢠Glutaraldehyde – Preferred for elastomeric impression materials.
⢠Alcohol-based disinfectants – They are not recommended, as they cause rapid surface drying and poor disinfection.SYNOPSIS
⢠Alginate impressions are hydrophilic and dimensionally unstable, hence, the disinfectant must be effective yet gentle.
⢠Glutaraldehyde, while effective, can be too aggressive for certain impression materials and may cause deformation.
⢠For effective disinfection, alginate impressions should typically be immersed in an iodophor solution for around 10 minutes.
⢠This time frame is sufficient to kill most microorganisms without compromising the quality or accuracy of the impression.REFERENCE
Evaluation of the Effectiveness of Disinfectants on Impression Materials CureusIncorrect
ANSWER
IodophorOTHER OPTIONS
⢠Glutaraldehyde – Preferred for elastomeric impression materials.
⢠Alcohol-based disinfectants – They are not recommended, as they cause rapid surface drying and poor disinfection.SYNOPSIS
⢠Alginate impressions are hydrophilic and dimensionally unstable, hence, the disinfectant must be effective yet gentle.
⢠Glutaraldehyde, while effective, can be too aggressive for certain impression materials and may cause deformation.
⢠For effective disinfection, alginate impressions should typically be immersed in an iodophor solution for around 10 minutes.
⢠This time frame is sufficient to kill most microorganisms without compromising the quality or accuracy of the impression.REFERENCE
Evaluation of the Effectiveness of Disinfectants on Impression Materials Cureus -
Question 30 of 150
30. Question
Which is considered the best method for disinfecting dental impressions to ensure effective microbial reduction?
Correct
ANSWER
ImmersionOTHER OPTIONS
⢠Refer SynopsisSYNOPSIS
⢠Immersion is considered the best method for disinfecting dental impressions, as it allows for complete contact with the disinfectant, ensuring thorough penetration and effective microbial reduction.
⢠Spraying may lead to incomplete coverage and may not be as effective while spraying twice still doesnāt guarantee the same level of disinfection as immersion.REFERENCE
Disinfection of impressions. Methods and effects on accuracy – State Dental JournalIncorrect
ANSWER
ImmersionOTHER OPTIONS
⢠Refer SynopsisSYNOPSIS
⢠Immersion is considered the best method for disinfecting dental impressions, as it allows for complete contact with the disinfectant, ensuring thorough penetration and effective microbial reduction.
⢠Spraying may lead to incomplete coverage and may not be as effective while spraying twice still doesnāt guarantee the same level of disinfection as immersion.REFERENCE
Disinfection of impressions. Methods and effects on accuracy – State Dental Journal -
Question 31 of 150
31. Question
A 30-year-old pregnant woman in her second trimester visits the dental clinic for a dental extraction due to severe caries in tooth 16. She has no significant medical history and is currently taking prenatal vitamins. During the consultation, the dentist discusses the need for anesthesia during the procedure. The dentist is considering using a local anesthetic classified as Category C according to the FDA pregnancy categories. Which of the following statements is true regarding the use of Category C anesthesia in this pregnant patient?
Correct
ANSWER
Category C anesthetics can be used if the potential benefits to the mother outweigh the risks to the fetus.OTHER OPTIONS
⢠Not applicableSYNOPSIS
⢠Category C anesthetics are those for which animal reproduction studies have shown an adverse effect on the fetus, but there are no adequate studies in humans.
⢠They may be used if the benefits to the mother justify the potential risks to the fetus.
⢠It is important to evaluate each situation individually, considering the necessity of the procedure and the specific anesthetic involved.
⢠Of the five categories (A, B, C, D, and X) lidocaine is considered in Category B, the safest of the local anesthetics.Ā
⢠Mepivacaine and bupivacaine are Category C and may be used with caution.REFERENCE
Local Anesthesia and Pregnancy – Dental Care.comIncorrect
ANSWER
Category C anesthetics can be used if the potential benefits to the mother outweigh the risks to the fetus.OTHER OPTIONS
⢠Not applicableSYNOPSIS
⢠Category C anesthetics are those for which animal reproduction studies have shown an adverse effect on the fetus, but there are no adequate studies in humans.
⢠They may be used if the benefits to the mother justify the potential risks to the fetus.
⢠It is important to evaluate each situation individually, considering the necessity of the procedure and the specific anesthetic involved.
⢠Of the five categories (A, B, C, D, and X) lidocaine is considered in Category B, the safest of the local anesthetics.Ā
⢠Mepivacaine and bupivacaine are Category C and may be used with caution.REFERENCE
Local Anesthesia and Pregnancy – Dental Care.com -
Question 32 of 150
32. Question
In a patient diagnosed with Class III malocclusion exhibiting a 5 mm reverse overjet, which of the following statements is true?
Correct
ANSWER
Chin is protrudedOTHER OPTIONS
⢠Not applicableSYNOPSIS
⢠In Class III malocclusion, characterized by a reverse overjet, the mandible is positioned forward relative to the maxilla.
⢠This positioning often results in a protruded appearance of the chin.
⢠The upper soft tissues may also appear retruded relative to the lower, contributing to the overall profile typical of Class III relationships.REFERENCE
Evolution of Class III treatment in orthodontics – American Journal of Orthodontics and Dentofacial OrthopedicsIncorrect
ANSWER
Chin is protrudedOTHER OPTIONS
⢠Not applicableSYNOPSIS
⢠In Class III malocclusion, characterized by a reverse overjet, the mandible is positioned forward relative to the maxilla.
⢠This positioning often results in a protruded appearance of the chin.
⢠The upper soft tissues may also appear retruded relative to the lower, contributing to the overall profile typical of Class III relationships.REFERENCE
Evolution of Class III treatment in orthodontics – American Journal of Orthodontics and Dentofacial Orthopedics -
Question 33 of 150
33. Question
Which among the following cells are responsible for the change in mucosal color because of gingivitis?
Correct
ANSWER
Plasma cellsOTHER OPTIONS
⢠Refer SynopsisSYNOPSIS
⢠In gingivitis, plasma cells produce antibodies in response to the inflammation, contributing to changes in the mucosal color.
⢠T lymphocytes and B lymphocytes also play roles in the immune response, but plasma cells are specifically responsible for the production of antibodies that can affect tissue appearance.
⢠Neutrophils are involved in the initial inflammatory response but do not directly cause changes in mucosal color related to chronic inflammation like gingivitis.REFERENCE
Carranza’s Clinical PeriodontologyIncorrect
ANSWER
Plasma cellsOTHER OPTIONS
⢠Refer SynopsisSYNOPSIS
⢠In gingivitis, plasma cells produce antibodies in response to the inflammation, contributing to changes in the mucosal color.
⢠T lymphocytes and B lymphocytes also play roles in the immune response, but plasma cells are specifically responsible for the production of antibodies that can affect tissue appearance.
⢠Neutrophils are involved in the initial inflammatory response but do not directly cause changes in mucosal color related to chronic inflammation like gingivitis.REFERENCE
Carranza’s Clinical Periodontology -
Question 34 of 150
34. Question
12-year-old female child reported to the Department of Pedodontics and Preventive Dentistry, with a painless swelling on the right side of the hard palate and displaced tooth on the same side. It had become noticeable for the past 6 years which gradually increased over a period of time. There was a positive family history and no relevant medical history. On physical examination, several cafĆ© au lait macules and freckling spots on the patient’s legs and palms were evident. (See Image) What is your diagnosis?
Correct
ANSWER
Neurofibromatosis Type 1OTHER OPTIONS
⢠Not applicableSYNOPSIS
⢠Neurofibromatosis-1 (NF-1), or Von Recklinghausen disease, is one of the inheritable neurocutaneous disorders manifested by developmental changes in the nervous system, bones, and skin. It is an autosomal dominant disorder
⢠Patients with NF-1 may present with the following
– Cafe-au-lait macules
– Neurofibromas
– Freckling
– Lisch nodules
– Skeletal abnormalities
– Neurological abnormalities
– Other malignancies
⢠Café au lait spots are commonly associated with neurofibromatosis type 1 (NF1).
⢠The presence of multiple spots, especially when there are six or more, is a key diagnostic criterion for NF1.
⢠Other conditions, such as tuberous sclerosis and McCune-Albright syndrome, can also present with skin findings but have additional characteristic features that are not present in this case.
⢠Freckling syndrome is not typically associated with café au lait spots.REFERENCE
Shafers Textbook of Oral PathologyIncorrect
ANSWER
Neurofibromatosis Type 1OTHER OPTIONS
⢠Not applicableSYNOPSIS
⢠Neurofibromatosis-1 (NF-1), or Von Recklinghausen disease, is one of the inheritable neurocutaneous disorders manifested by developmental changes in the nervous system, bones, and skin. It is an autosomal dominant disorder
⢠Patients with NF-1 may present with the following
– Cafe-au-lait macules
– Neurofibromas
– Freckling
– Lisch nodules
– Skeletal abnormalities
– Neurological abnormalities
– Other malignancies
⢠Café au lait spots are commonly associated with neurofibromatosis type 1 (NF1).
⢠The presence of multiple spots, especially when there are six or more, is a key diagnostic criterion for NF1.
⢠Other conditions, such as tuberous sclerosis and McCune-Albright syndrome, can also present with skin findings but have additional characteristic features that are not present in this case.
⢠Freckling syndrome is not typically associated with café au lait spots.REFERENCE
Shafers Textbook of Oral Pathology -
Question 35 of 150
35. Question
A patient comes with pain in the upper left premolar region, which is radiating to the ear and eye. On examination, there is no caries or periapical pathology. Radiographically no abnormalities found. But when you press the premolar area it feels severe pain. What is the cause?
Correct
ANSWER
Maxillary sinusitisOTHER OPTIONS
⢠Acute apical periodontitis – In acute apical periodontitis, patient will complaints of deep dental caries, periapical pathology and associated tenderness on percussion.
⢠Reversible pulpitis – In reversible pulpitis, patient will have pain or sensitivity that relieves after removal of stimulus.SYNOPSIS
⢠Maxillary sinusitis is an inflammation of the paranasal sinuses caused by bacteria, viruses, fungi or allergic reactions.
⢠It is clinically presented as headache, maxillary dental pain, cough, halitosis, facial pressure, nasal obstruction, nasal blockage etc.
⢠Usually pain radiates to ear, eyes and forehead similar to that of dental pain.
⢠Pain can maxillary sinusitis can be confused with dental pain most of the time.REFERENCE
Peterson’s Principles of Oral and Maxillofacial Surgery – 2nd Edition Pge No 295.Incorrect
ANSWER
Maxillary sinusitisOTHER OPTIONS
⢠Acute apical periodontitis – In acute apical periodontitis, patient will complaints of deep dental caries, periapical pathology and associated tenderness on percussion.
⢠Reversible pulpitis – In reversible pulpitis, patient will have pain or sensitivity that relieves after removal of stimulus.SYNOPSIS
⢠Maxillary sinusitis is an inflammation of the paranasal sinuses caused by bacteria, viruses, fungi or allergic reactions.
⢠It is clinically presented as headache, maxillary dental pain, cough, halitosis, facial pressure, nasal obstruction, nasal blockage etc.
⢠Usually pain radiates to ear, eyes and forehead similar to that of dental pain.
⢠Pain can maxillary sinusitis can be confused with dental pain most of the time.REFERENCE
Peterson’s Principles of Oral and Maxillofacial Surgery – 2nd Edition Pge No 295. -
Question 36 of 150
36. Question
A healthy 18 years old patient, the bacterial complex present in his mouth is?
Correct
ANSWER
Purple complexOTHER OPTIONS
⢠Refer SynopsisSYNOPSIS
⢠In a young healthy patient (like 18 years old) with no periodontal breakdown, the microflora is dominated by early colonizers.
⢠Socransky (1998) grouped subgingival plaque bacteria into microbial complexes based on their association with disease.
⢠Purple complex (early colonizers) – Streptococcus sanguinis, Actinomyces species) are the first colonizers on the tooth surface. They pave the way for later colonizers.
⢠Green complex (Capnocytophaga, Eikenella corrodens, Aggregatibacter actinomycetemcomitans) is also involved in early colonization.
⢠Orange complex (Prevotella intermedia, Fusobacterium nucleatum, Campylobacter rectus) is associated with gingivitis and precedes the red complex.
⢠Red complex (Porphyromonas gingivalis, Tannerella forsythia, Treponema denticola) is strongly associated with chronic periodontitis and deep pockets.REFERENCE
Carranzaās Clinical Periodontology, 13th EditionIncorrect
ANSWER
Purple complexOTHER OPTIONS
⢠Refer SynopsisSYNOPSIS
⢠In a young healthy patient (like 18 years old) with no periodontal breakdown, the microflora is dominated by early colonizers.
⢠Socransky (1998) grouped subgingival plaque bacteria into microbial complexes based on their association with disease.
⢠Purple complex (early colonizers) – Streptococcus sanguinis, Actinomyces species) are the first colonizers on the tooth surface. They pave the way for later colonizers.
⢠Green complex (Capnocytophaga, Eikenella corrodens, Aggregatibacter actinomycetemcomitans) is also involved in early colonization.
⢠Orange complex (Prevotella intermedia, Fusobacterium nucleatum, Campylobacter rectus) is associated with gingivitis and precedes the red complex.
⢠Red complex (Porphyromonas gingivalis, Tannerella forsythia, Treponema denticola) is strongly associated with chronic periodontitis and deep pockets.REFERENCE
Carranzaās Clinical Periodontology, 13th Edition -
Question 37 of 150
37. Question
Patient comes to your clinic for review of CD has no complaints during speech or swallowing. But lips look elongated with narrow vermillion border. What could be the reason?
Correct
ANSWER
Malpositioned upper teethOTHER OPTIONS
Not applicableSYNOPSIS
⢠The most reliable guides to aid the dentist in the final setting of the maxillary anterior teeth are the nasolabial sulcus, the philtrum, and the commissure (corner) of the mouth.
⢠When proper lip support has been developed, by the denture base and anterior teeth arrangement these anatomic landmarks will exhibit a distinct, normal appearance.
⢠Without proper support, there is a noticeable deepening of the nasolabial, loss of definition in the philtrum and a lowering or drooping of the corners of the mouth.
⢠If denture teeth are set too far facially, it produces a stretched look. The nasolabial and the philtrum become very shallow and appear somewhat smooth, while the corners of the mouth are stretched laterally.REFERENCE
Winkler S, Essentials of complete denture Prosthodontics, pg 203Incorrect
ANSWER
Malpositioned upper teethOTHER OPTIONS
Not applicableSYNOPSIS
⢠The most reliable guides to aid the dentist in the final setting of the maxillary anterior teeth are the nasolabial sulcus, the philtrum, and the commissure (corner) of the mouth.
⢠When proper lip support has been developed, by the denture base and anterior teeth arrangement these anatomic landmarks will exhibit a distinct, normal appearance.
⢠Without proper support, there is a noticeable deepening of the nasolabial, loss of definition in the philtrum and a lowering or drooping of the corners of the mouth.
⢠If denture teeth are set too far facially, it produces a stretched look. The nasolabial and the philtrum become very shallow and appear somewhat smooth, while the corners of the mouth are stretched laterally.REFERENCE
Winkler S, Essentials of complete denture Prosthodontics, pg 203 -
Question 38 of 150
38. Question
You noticed bony undercuts on anterior maxilla and lateral maxilla on intraoral examination of a complete denture patient. What would be the management?
Correct
ANSWER
Remove lateral undercut surgically and leave anterior undercutOTHER OPTIONS
⢠Refer SynopsisSYNOPSIS
⢠The ideal treatment is to remove the lateral undercut surgically and leave the anterior undercut.
⢠The anterior residual ridge must be preserved whenever possible, since this area is crucial for support and stability of the denture and tends to be a region of rapid bone loss.
⢠Undercuts that interfere with the path of placement both in anterior and posterior regions are usually detrimental to accurate tissue adaptation and should be reduced.
⢠If anterior and posterior interferences both are present, reduction of the posterior areas is preferred over reduction of the anterior regions.
⢠With bilateral bony protuberances in the tuberosity area, one side can usually be left intact and the opposite side surgically corrected.REFERENCE
Winkler S, Essentials of complete denture Prosthodontics, pg 71Incorrect
ANSWER
Remove lateral undercut surgically and leave anterior undercutOTHER OPTIONS
⢠Refer SynopsisSYNOPSIS
⢠The ideal treatment is to remove the lateral undercut surgically and leave the anterior undercut.
⢠The anterior residual ridge must be preserved whenever possible, since this area is crucial for support and stability of the denture and tends to be a region of rapid bone loss.
⢠Undercuts that interfere with the path of placement both in anterior and posterior regions are usually detrimental to accurate tissue adaptation and should be reduced.
⢠If anterior and posterior interferences both are present, reduction of the posterior areas is preferred over reduction of the anterior regions.
⢠With bilateral bony protuberances in the tuberosity area, one side can usually be left intact and the opposite side surgically corrected.REFERENCE
Winkler S, Essentials of complete denture Prosthodontics, pg 71 -
Question 39 of 150
39. Question
A 45 year old man has recently came from vacation in Bangkok where he had been provided with bilateral mandibular tooth supported cast metal fixed prostheses. He is having continuous soreness in the teeth. Clinical examination reveals the diagnosis of post restorative hyper occlusion. Which of the following will be most likely happening if necessary management is delayed for a period of one year?
Correct
ANSWER
Prosthesis connector fractureOTHER OPTIONS
Not applicableSYNOPSIS
⢠In all ceramic fixed partial dentures the connector area is a common fracture location.
⢠The occlusal forces applied to a fixed dental prosthesis are transmitted to the supporting structures through the pontics, connectors, and retainers
⢠The connector area is usually narrowly constricted for biological or esthetic reasons, which typically considers stresses relative to the average stress levels in other areas of the prosthesis
⢠Long-term unmanaged hyper occlusion in an FPD can lead to prosthesis connector fractureREFERENCE
Goodacre et al Clinical complications in fixed prosthodontics. Journal 0f Prosthetic Dentistry 2003,90-31-41Incorrect
ANSWER
Prosthesis connector fractureOTHER OPTIONS
Not applicableSYNOPSIS
⢠In all ceramic fixed partial dentures the connector area is a common fracture location.
⢠The occlusal forces applied to a fixed dental prosthesis are transmitted to the supporting structures through the pontics, connectors, and retainers
⢠The connector area is usually narrowly constricted for biological or esthetic reasons, which typically considers stresses relative to the average stress levels in other areas of the prosthesis
⢠Long-term unmanaged hyper occlusion in an FPD can lead to prosthesis connector fractureREFERENCE
Goodacre et al Clinical complications in fixed prosthodontics. Journal 0f Prosthetic Dentistry 2003,90-31-41 -
Question 40 of 150
40. Question
Lateral occlusal forces during mandibular eccentric, functional or para functional movements, exert forces that could lead to non restorable fractures of root canal treated posterior teeth. Which occlusal parameter from the following would expose posterior teeth to more lateral occlusal forces?
Correct
ANSWER
Group functionOTHER OPTIONS
⢠Canine-protected occlusion is otherwise called mutually protected occlusionSYNOPSIS
⢠Unilateral balanced occlusion or group function calls for all teeth on the working side to be in contact during a lateral excursion. On the other hand, teeth on the nonworking side are contoured to be free of any contact.
⢠The group function of the teeth on the working side distributes the occlusal load. The absence of contact on the working side prevents those teeth from being subjected to the destructive, obliquely directed forces found in non-working interferences.
⢠Group function exposes posterior teeth to more lateral occlusal forces.REFERENCE
Shillingburg, Fundamentals of Fixed Prosthodontics,pg 19Incorrect
ANSWER
Group functionOTHER OPTIONS
⢠Canine-protected occlusion is otherwise called mutually protected occlusionSYNOPSIS
⢠Unilateral balanced occlusion or group function calls for all teeth on the working side to be in contact during a lateral excursion. On the other hand, teeth on the nonworking side are contoured to be free of any contact.
⢠The group function of the teeth on the working side distributes the occlusal load. The absence of contact on the working side prevents those teeth from being subjected to the destructive, obliquely directed forces found in non-working interferences.
⢠Group function exposes posterior teeth to more lateral occlusal forces.REFERENCE
Shillingburg, Fundamentals of Fixed Prosthodontics,pg 19 -
Question 41 of 150
41. Question
What is the causative organism most strongly associated with periodontitis?
Correct
ANSWER
Tanerella forsythiaOTHER OPTIONS
⢠Refer SynopsisSYNOPSIS
⢠Periodontitis is a polymicrobial disease – no single pathogen is solely responsible.
– However, certain bacteria are considered key pathogens
⢠Red complex organisms (Porphyromonas gingivalis, Tannerella forsythia, Treponema denticola) are strongly linked with chronic periodontitis.
⢠Aggregatibacter actinomycetemcomitans is associated with localized aggressive periodontitis.
⢠Listeria is not associated with periodontal disease, it is mainly a foodborne pathogen.
⢠Streptococcus are early colonizers in plaque but not primary causes of periodontitis.
⢠Thus, among the given options, Tannerella forsythia is the best answer.REFERENCE
Carranzaās Clinical Periodontology, 13th EditionIncorrect
ANSWER
Tanerella forsythiaOTHER OPTIONS
⢠Refer SynopsisSYNOPSIS
⢠Periodontitis is a polymicrobial disease – no single pathogen is solely responsible.
– However, certain bacteria are considered key pathogens
⢠Red complex organisms (Porphyromonas gingivalis, Tannerella forsythia, Treponema denticola) are strongly linked with chronic periodontitis.
⢠Aggregatibacter actinomycetemcomitans is associated with localized aggressive periodontitis.
⢠Listeria is not associated with periodontal disease, it is mainly a foodborne pathogen.
⢠Streptococcus are early colonizers in plaque but not primary causes of periodontitis.
⢠Thus, among the given options, Tannerella forsythia is the best answer.REFERENCE
Carranzaās Clinical Periodontology, 13th Edition -
Question 42 of 150
42. Question
Which is true about the properties of polyvinyl siloxanes compared with polysulfide?
Correct
ANSWER
A and BOTHER OPTIONS
⢠Less dimensional stability – Dimensional change is greater in magnitude for the polysulfide than for the addition silicone elastomers. This result is expected because polysulfide loses polymerization by-products, i.e.,water.SYNOPSIS
⢠Polyvinyl siloxane impressions can be poured even after seven days or more than once.
⢠Research has shown that a cast produced from an addition silicone impression between 24 hours and 1 week was as accurate as a cast made in the first hour assuming that there was no effect associated with the formation of hydrogen bubbles.
⢠These materials exhibit the least amount of distortion from the loads imposed on the set material.
⢠Thus, multiple pouring of the impressions and removal of the casts will maintain the dimensional stability of the impression even though a substantial force is needed each time a cast is removed from the impression.REFERENCE
Philips Science of Dental materials, 12th Ed, Pg 166Incorrect
ANSWER
A and BOTHER OPTIONS
⢠Less dimensional stability – Dimensional change is greater in magnitude for the polysulfide than for the addition silicone elastomers. This result is expected because polysulfide loses polymerization by-products, i.e.,water.SYNOPSIS
⢠Polyvinyl siloxane impressions can be poured even after seven days or more than once.
⢠Research has shown that a cast produced from an addition silicone impression between 24 hours and 1 week was as accurate as a cast made in the first hour assuming that there was no effect associated with the formation of hydrogen bubbles.
⢠These materials exhibit the least amount of distortion from the loads imposed on the set material.
⢠Thus, multiple pouring of the impressions and removal of the casts will maintain the dimensional stability of the impression even though a substantial force is needed each time a cast is removed from the impression.REFERENCE
Philips Science of Dental materials, 12th Ed, Pg 166 -
Question 43 of 150
43. Question
Which is the principle that embraces respect for person, the ability to be self-governing?
Correct
ANSWER
AutonomyOTHER OPTIONS
⢠Justice – Justice is principle concerned with providing individuals or groups with what is owed, dued and deserved.
⢠Beneficence – Beneficence is principle that actions and practises are right in so far as they produce good consequences.
⢠Veracity – Veracity is principle of being honest and telling truth and is related to autonomy.SYNOPSIS
⢠Autonomy is an ethical principle which includes self governance.
⢠It includes patient’s privacy and safeguarding the patient’s privacy.
⢠It includes involving patient in treatment decisions with due consideration being given to patient needs, desires and abilities.
⢠Informing the patient of disease or defective dental concerns is first step.
⢠A written consent should be signed from patient before any treatment.REFERENCE
Article on Ethics in Dentistry – Principles and Values.Incorrect
ANSWER
AutonomyOTHER OPTIONS
⢠Justice – Justice is principle concerned with providing individuals or groups with what is owed, dued and deserved.
⢠Beneficence – Beneficence is principle that actions and practises are right in so far as they produce good consequences.
⢠Veracity – Veracity is principle of being honest and telling truth and is related to autonomy.SYNOPSIS
⢠Autonomy is an ethical principle which includes self governance.
⢠It includes patient’s privacy and safeguarding the patient’s privacy.
⢠It includes involving patient in treatment decisions with due consideration being given to patient needs, desires and abilities.
⢠Informing the patient of disease or defective dental concerns is first step.
⢠A written consent should be signed from patient before any treatment.REFERENCE
Article on Ethics in Dentistry – Principles and Values. -
Question 44 of 150
44. Question
A 26-year-old patient presents with gingival trauma characterized by erythema and localized abrasions on the marginal gingiva. The history reveals faulty toothbrushing habits (horizontal scrubbing with a medium brush).
What is the best management approach for this patient?Correct
ANSWER
All of the aboveOTHER OPTIONS
⢠NilSYNOPSIS
⢠Management of gingival trauma due to faulty brushing involves
– Correction of faulty technique – Educate patient to adopt an atraumatic method (e.g., Modified Bass or Stillman).
– Reassurance – Gingival lesions usually heal if trauma is removed.
– Soft brush substitution – Replacing medium hard brush with soft bristles prevents recurrence.
⢠Therefore, a combined approach is best.REFERENCE
Carranzaās Clinical Periodontology, 13th EditionIncorrect
ANSWER
All of the aboveOTHER OPTIONS
⢠NilSYNOPSIS
⢠Management of gingival trauma due to faulty brushing involves
– Correction of faulty technique – Educate patient to adopt an atraumatic method (e.g., Modified Bass or Stillman).
– Reassurance – Gingival lesions usually heal if trauma is removed.
– Soft brush substitution – Replacing medium hard brush with soft bristles prevents recurrence.
⢠Therefore, a combined approach is best.REFERENCE
Carranzaās Clinical Periodontology, 13th Edition -
Question 45 of 150
45. Question
An 11 year old child was brought to the clinic with skeletal class II malocclusion with mandibular deficiency. Which among the following is best indicated for its correction?
Correct
ANSWER
Herbst applianceOTHER OPTIONS
⢠Reverse head gear and Chin cup – Reverse head gear and chin cup are used in correction of skeletal class III in growing patients.
⢠Fixed orthodontics – Fixed orthodontics is indicated in non growing patients or to correct dental malocclusions mainly.SYNOPSIS
⢠Class II malocclusion by mandibular deficiency is treated during mixed dentition period by use of myofunctional appliances.
⢠This moves mandible forward and downward direction and therefore stimulates mandibular growth.
⢠They can also cause retrusion of maxillary anteriors and proclination of mandibular anteriors.
⢠Examples of myofunctional appliances are activator and functional regulator.
⢠In case of a patient at end of growing period, fixed functional appliances like herbst appliance or jasper jumper is indicated.REFERENCE
Orthodontics The Art and Science – 6th Edition Page No 497.Incorrect
ANSWER
Herbst applianceOTHER OPTIONS
⢠Reverse head gear and Chin cup – Reverse head gear and chin cup are used in correction of skeletal class III in growing patients.
⢠Fixed orthodontics – Fixed orthodontics is indicated in non growing patients or to correct dental malocclusions mainly.SYNOPSIS
⢠Class II malocclusion by mandibular deficiency is treated during mixed dentition period by use of myofunctional appliances.
⢠This moves mandible forward and downward direction and therefore stimulates mandibular growth.
⢠They can also cause retrusion of maxillary anteriors and proclination of mandibular anteriors.
⢠Examples of myofunctional appliances are activator and functional regulator.
⢠In case of a patient at end of growing period, fixed functional appliances like herbst appliance or jasper jumper is indicated.REFERENCE
Orthodontics The Art and Science – 6th Edition Page No 497. -
Question 46 of 150
46. Question
You are treating a 62-year-old female patient who recently got new dentures. She complains that her dentures tend to displace posteriorly (towards the back of her mouth) when she smiles. The patient has no relevant medical history, and her oral examination shows well-fitting dentures with no apparent issues. You suspect a specific reason for this denture displacement during smiling. What is the most likely reason for the posterior displacement of the dentures while the patient smiles?
Correct
ANSWER
Inadequate vertical dimension of occlusionOTHER OPTIONS
⢠Inadequate denture adhesive – Inadequate denture adhesive is not the most likely reason in this case, as the patient’s dentures have been described as well-fitting with no apparent issues.
⢠Improper denture base material – Improper denture base material is not the most likely reason, as there are no indications of material-related problems in the scenario.
⢠Overextended denture borders – Overextended denture borders may cause discomfort or sore spots but are less likely to be the primary reason for posterior displacement during smiling.SYNOPSIS
⢠The most likely reason for the posterior displacement of the dentures during smiling is an inadequate vertical dimension of occlusion.
⢠The vertical dimension of occlusion (VDO) refers to the space between the upper and lower jaws when the teeth are in contact.
⢠Proper VDO is crucial for maintaining the stability and function of dentures.
⢠If the VDO is too low, it can cause the dentures to displace backward during muscle activity like smiling.
⢠In this case, since the patient has no apparent issues with the denture fit or oral examination, the problem is likely related to the VDO.
To address this issue, the dentist should assess and adjust the VDO to ensure the patient’s dentures have proper stability and do not displace during function.REFERENCE
Textbook of Prosthodontics – Deepak NallaswamyIncorrect
ANSWER
Inadequate vertical dimension of occlusionOTHER OPTIONS
⢠Inadequate denture adhesive – Inadequate denture adhesive is not the most likely reason in this case, as the patient’s dentures have been described as well-fitting with no apparent issues.
⢠Improper denture base material – Improper denture base material is not the most likely reason, as there are no indications of material-related problems in the scenario.
⢠Overextended denture borders – Overextended denture borders may cause discomfort or sore spots but are less likely to be the primary reason for posterior displacement during smiling.SYNOPSIS
⢠The most likely reason for the posterior displacement of the dentures during smiling is an inadequate vertical dimension of occlusion.
⢠The vertical dimension of occlusion (VDO) refers to the space between the upper and lower jaws when the teeth are in contact.
⢠Proper VDO is crucial for maintaining the stability and function of dentures.
⢠If the VDO is too low, it can cause the dentures to displace backward during muscle activity like smiling.
⢠In this case, since the patient has no apparent issues with the denture fit or oral examination, the problem is likely related to the VDO.
To address this issue, the dentist should assess and adjust the VDO to ensure the patient’s dentures have proper stability and do not displace during function.REFERENCE
Textbook of Prosthodontics – Deepak Nallaswamy -
Question 47 of 150
47. Question
You were presented with a case of impacted tooth third molar. The radiographic examination showed an unusually large dentigerous cyst involving the mandibular ramus associated with it. Which is the best way to manage?
Correct
ANSWER
MarsupialisationOTHER OPTIONS
⢠Explained belowSYNOPSIS
⢠Dentigerous cyst is managed by Enucleation, Marsupialisation, or a combination of both
⢠Enucleation – As a general surgical technique, enucleation refers to the surgical removal of a mass without cutting into or dissecting it.
⢠Marsupialization – Marsupialization is the surgical technique of cutting a slit into an abscess or cyst and suturing the edges of the slit to form a continuous surface from the exterior surface to the interior surface of the cyst or abscess. Sutured in this fashion, the site remains open and can drain freely.
⢠Large dentigerous cysts may also be treated by marsupialization followed by enucleation
⢠For small to moderate-sized cysts, enucleation with removal of associated tooth is the treatment of choice.
⢠For very large cysts (as in this case, extending into the ramus), enucleation risks fracture of mandible, damage to inferior alveolar nerve, and removal of large bone volume.
⢠In this case, Marsupialisation (decompression) is preferred initially as it reduces intracystic pressure, promotes bone fill, and decreases cyst size. Later, if required, a second-stage enucleation can be performed more safely.
mREFERENCE
Shafer’s textbook of oral pathologyIncorrect
ANSWER
MarsupialisationOTHER OPTIONS
⢠Explained belowSYNOPSIS
⢠Dentigerous cyst is managed by Enucleation, Marsupialisation, or a combination of both
⢠Enucleation – As a general surgical technique, enucleation refers to the surgical removal of a mass without cutting into or dissecting it.
⢠Marsupialization – Marsupialization is the surgical technique of cutting a slit into an abscess or cyst and suturing the edges of the slit to form a continuous surface from the exterior surface to the interior surface of the cyst or abscess. Sutured in this fashion, the site remains open and can drain freely.
⢠Large dentigerous cysts may also be treated by marsupialization followed by enucleation
⢠For small to moderate-sized cysts, enucleation with removal of associated tooth is the treatment of choice.
⢠For very large cysts (as in this case, extending into the ramus), enucleation risks fracture of mandible, damage to inferior alveolar nerve, and removal of large bone volume.
⢠In this case, Marsupialisation (decompression) is preferred initially as it reduces intracystic pressure, promotes bone fill, and decreases cyst size. Later, if required, a second-stage enucleation can be performed more safely.
mREFERENCE
Shafer’s textbook of oral pathology -
Question 48 of 150
48. Question
You dispose waste materials after doing treatment for a hepatitis B patient carelessly. Your dental clinic cleaning staff was got needle prick injury. Which infection control is failed here?
Correct
ANSWER
Active infection controlOTHER OPTIONS
⢠Not applicableSYNOPSIS
⢠Without proper infection control techniques, many of the things in a dental clinic, including people, instruments, and even computer components can be carriers for cross-contamination and contribute to the spreading of germs and disease.
⢠Active Infection control prevents or stops the spread of infections in healthcare settings.
ā¢.Non-disposable items like dental tools are cleaned and sterilized between patients.
⢠Disposable dental tools and needles are never reused.
⢠Infection control precautions also require all dental staff involved in patient care to use appropriate protective equipment such as gloves, masks, gowns, and eyewear when needed.
⢠Failure to follow any of the above guidelines means failure of active infection control.REFERENCE
Hepatitis B- Penina Haber, MPH and Sarah Schillie, MD, MPH, MBAIncorrect
ANSWER
Active infection controlOTHER OPTIONS
⢠Not applicableSYNOPSIS
⢠Without proper infection control techniques, many of the things in a dental clinic, including people, instruments, and even computer components can be carriers for cross-contamination and contribute to the spreading of germs and disease.
⢠Active Infection control prevents or stops the spread of infections in healthcare settings.
ā¢.Non-disposable items like dental tools are cleaned and sterilized between patients.
⢠Disposable dental tools and needles are never reused.
⢠Infection control precautions also require all dental staff involved in patient care to use appropriate protective equipment such as gloves, masks, gowns, and eyewear when needed.
⢠Failure to follow any of the above guidelines means failure of active infection control.REFERENCE
Hepatitis B- Penina Haber, MPH and Sarah Schillie, MD, MPH, MBA -
Question 49 of 150
49. Question
In Ellis classification of injured teeth, a class V represents?
Correct
ANSWER
Avulsed toothOTHER OPTIONS
⢠Refer SynopsisSYNOPSIS
⢠Class I – Simple crown fracture with little or no dentin affected.
⢠Class II – Extensive crown fracture with considerable loss of dentin, but with the pulp not affected.
⢠Class III – Extensive crown fracture with considerable loss of dentin and pulp exposure.
⢠Class IV – A tooth devitalized by trauma with or without loss of tooth structure.
⢠Class V – Teeth lost as a result of trauma.
⢠Class VI – Root fracture with or without the loss of crown structure.
⢠Class VII – Displacement of the tooth with neither root nor crown fracture
⢠Class VIII – Complete crown fracture and its replacement.
⢠Class IX – Traumatic injuries of primary teeth.REFERENCE
An overview of the classification of dental trauma-Sasikala PagadalaIncorrect
ANSWER
Avulsed toothOTHER OPTIONS
⢠Refer SynopsisSYNOPSIS
⢠Class I – Simple crown fracture with little or no dentin affected.
⢠Class II – Extensive crown fracture with considerable loss of dentin, but with the pulp not affected.
⢠Class III – Extensive crown fracture with considerable loss of dentin and pulp exposure.
⢠Class IV – A tooth devitalized by trauma with or without loss of tooth structure.
⢠Class V – Teeth lost as a result of trauma.
⢠Class VI – Root fracture with or without the loss of crown structure.
⢠Class VII – Displacement of the tooth with neither root nor crown fracture
⢠Class VIII – Complete crown fracture and its replacement.
⢠Class IX – Traumatic injuries of primary teeth.REFERENCE
An overview of the classification of dental trauma-Sasikala Pagadala -
Question 50 of 150
50. Question
Which among the following muscles is not involved in the elevation of the mandible?
Correct
ANSWER
Lateral pterygoidOTHER OPTIONS
⢠Masseter – The large muscle which raises the lower jaw, and assists in mastication.
⢠Temporalis – A broad muscle that fans out to cover much of the temporal bone on the side of the skull that elevates and retracts the jaw.
⢠Medial pterygoid – The medial pterygoid elevates and closes the jaw, contributes to protrusion of the mandible, and assists in mastication.SYNOPSIS
⢠Lateral pterygoid is a muscle of mastication with two heads. It lies superiorly to the medial pterygoid.
⢠The lateral pterygoid is the only muscle of mastication that actively opens the jaw.
⢠Unilateral action of a lateral pterygoid produces lateral movement in the jaw, usually performed in concert with the medial pterygoidsREFERENCE
Muscles of mastication – Pocket DentistryIncorrect
ANSWER
Lateral pterygoidOTHER OPTIONS
⢠Masseter – The large muscle which raises the lower jaw, and assists in mastication.
⢠Temporalis – A broad muscle that fans out to cover much of the temporal bone on the side of the skull that elevates and retracts the jaw.
⢠Medial pterygoid – The medial pterygoid elevates and closes the jaw, contributes to protrusion of the mandible, and assists in mastication.SYNOPSIS
⢠Lateral pterygoid is a muscle of mastication with two heads. It lies superiorly to the medial pterygoid.
⢠The lateral pterygoid is the only muscle of mastication that actively opens the jaw.
⢠Unilateral action of a lateral pterygoid produces lateral movement in the jaw, usually performed in concert with the medial pterygoidsREFERENCE
Muscles of mastication – Pocket Dentistry -
Question 51 of 150
51. Question
Which among the following has a high rate of recurrence?
Correct
ANSWER
OKCOTHER OPTIONS
⢠Not applicableSYNOPSIS
⢠Odontogenic keratocyst (OKC) is the cyst arising from the cell rests of the dental lamina.
⢠It can occur anywhere in the jaw but is commonly seen in the posterior part of the mandible.
⢠Radiographically, most OKCs are unilocular when presented at the periapex and can be mistaken for a radicular or lateral periodontal cyst.
⢠The exact reason for the high recurrence rate of OKC has not been established, it is thought to be due to incomplete removal of the primary lesion with thin epithelial lining, the presence of satellite cysts, and epithelial remnants.REFERENCE
Shafer’s Textbook of Oral PathologyIncorrect
ANSWER
OKCOTHER OPTIONS
⢠Not applicableSYNOPSIS
⢠Odontogenic keratocyst (OKC) is the cyst arising from the cell rests of the dental lamina.
⢠It can occur anywhere in the jaw but is commonly seen in the posterior part of the mandible.
⢠Radiographically, most OKCs are unilocular when presented at the periapex and can be mistaken for a radicular or lateral periodontal cyst.
⢠The exact reason for the high recurrence rate of OKC has not been established, it is thought to be due to incomplete removal of the primary lesion with thin epithelial lining, the presence of satellite cysts, and epithelial remnants.REFERENCE
Shafer’s Textbook of Oral Pathology -
Question 52 of 150
52. Question
A young patient is worried about proclination. On examination you noticed that the patient has class I molar malocclusion. What would be the main reason for this proclination?
Correct
ANSWER
Arch length tooth size discrepancyOTHER OPTIONS
Not applicableSYNOPSIS
⢠Class I malocclusions is a situation where the anteroposterior occlusal
relationship is normal and there is a discrepancy either within the arches or in the transverse or vertical relationship between the arches.
⢠Dental factors are the main aetiological influences in Class I malocclusions.
⢠The most common are tooth or arch size discrepancies, leading to crowding or, less frequently, spacing.
⢠Local factors also include displaced or impacted teeth, and anomalies in the
size, number and form of the teeth, all of which can lead to a localized
malocclusion.
⢠Soft tissue and skeletal factors can cause this malocclusion.REFERENCE
Arch Size and Tooth Size in Class I malocclusion by JDAT.Incorrect
ANSWER
Arch length tooth size discrepancyOTHER OPTIONS
Not applicableSYNOPSIS
⢠Class I malocclusions is a situation where the anteroposterior occlusal
relationship is normal and there is a discrepancy either within the arches or in the transverse or vertical relationship between the arches.
⢠Dental factors are the main aetiological influences in Class I malocclusions.
⢠The most common are tooth or arch size discrepancies, leading to crowding or, less frequently, spacing.
⢠Local factors also include displaced or impacted teeth, and anomalies in the
size, number and form of the teeth, all of which can lead to a localized
malocclusion.
⢠Soft tissue and skeletal factors can cause this malocclusion.REFERENCE
Arch Size and Tooth Size in Class I malocclusion by JDAT. -
Question 53 of 150
53. Question
A child patient with past history of chickenpox presented with brownish stains on teeth. At what stage of development the teeth would have been affected?
Correct
ANSWER
AppositionOTHER OPTIONS
⢠Initiation – Anodontia, hypodontia etc
⢠Histodifferentiation – Cysts or neoplasm
⢠Morphodifferentiation – Anomalies in morphology of teethSYNOPSIS
⢠Systemic postnatal infection (e.g. measles, chicken pox, streptococcal infections, scarlet fever) can cause enamel hypoplasia.
⢠The band-like discoloration on the tooth is visualized where the enamel layer has a variable thickness and becomes extrinsically stained after tooth eruption.
⢠Enamel hypoplasia occurs when the special cells that produce dental enamel are disturbed during a particular stage of enamel formation – theĀ matrix formation stage or apposition stageREFERENCE
Shafer’s Textbook of Oral PathologyIncorrect
ANSWER
AppositionOTHER OPTIONS
⢠Initiation – Anodontia, hypodontia etc
⢠Histodifferentiation – Cysts or neoplasm
⢠Morphodifferentiation – Anomalies in morphology of teethSYNOPSIS
⢠Systemic postnatal infection (e.g. measles, chicken pox, streptococcal infections, scarlet fever) can cause enamel hypoplasia.
⢠The band-like discoloration on the tooth is visualized where the enamel layer has a variable thickness and becomes extrinsically stained after tooth eruption.
⢠Enamel hypoplasia occurs when the special cells that produce dental enamel are disturbed during a particular stage of enamel formation – theĀ matrix formation stage or apposition stageREFERENCE
Shafer’s Textbook of Oral Pathology -
Question 54 of 150
54. Question
A dentist accidentally pushed the mandibular third molar tooth towards the bone while extracting it. It might have fallen into which space?
Correct
ANSWER
Submandibular spaceOTHER OPTIONS
⢠Pterygomandibular space -The pterygomandibular space is a fascial space of the head and neck. It is located between the lateral pterygoid muscle and the medial surface of the ramus of the mandible.
⢠Lateral pharyngeal space -The parapharyngeal space, is a potential space in the head and the neck. It has clinical importance in otolaryngology due to parapharyngeal space tumors and parapharyngeal abscesses developing in this area.
⢠Retropharyngeal space -The retropharyngeal space (RPS) is an anatomical region that spans from the base of the skull to the mediastinum. Its location is anterior to the prevertebral muscles and posterior to the pharynx and esophagus.SYNOPSIS
⢠Usually, the impacted third molar tooth is indicated for extraction.
⢠Sometimes, it may displace into the lingual pouch or submandibular space during extraction particularly when the lingual cortex is extremely thin.
⢠It may further traverse into pharyngeal spaces if not managed by some expert.
⢠The intraoral approach under local anesthesia is the simplest and least invasive technique for the removal of displaced roots from the lingual pouch.REFERENCE
Retrieval of mandibular third molar tooth accidentally displaced in submandibular space – Ravinder SolankiIncorrect
ANSWER
Submandibular spaceOTHER OPTIONS
⢠Pterygomandibular space -The pterygomandibular space is a fascial space of the head and neck. It is located between the lateral pterygoid muscle and the medial surface of the ramus of the mandible.
⢠Lateral pharyngeal space -The parapharyngeal space, is a potential space in the head and the neck. It has clinical importance in otolaryngology due to parapharyngeal space tumors and parapharyngeal abscesses developing in this area.
⢠Retropharyngeal space -The retropharyngeal space (RPS) is an anatomical region that spans from the base of the skull to the mediastinum. Its location is anterior to the prevertebral muscles and posterior to the pharynx and esophagus.SYNOPSIS
⢠Usually, the impacted third molar tooth is indicated for extraction.
⢠Sometimes, it may displace into the lingual pouch or submandibular space during extraction particularly when the lingual cortex is extremely thin.
⢠It may further traverse into pharyngeal spaces if not managed by some expert.
⢠The intraoral approach under local anesthesia is the simplest and least invasive technique for the removal of displaced roots from the lingual pouch.REFERENCE
Retrieval of mandibular third molar tooth accidentally displaced in submandibular space – Ravinder Solanki -
Question 55 of 150
55. Question
The disadvantage of screw-retained Implant prosthesis lies in its
Correct
ANSWER
PassivityOTHER OPTIONS
⢠Explained belowSYNOPSIS
⢠Implant restorations can be screw-retained or cement retained.
⢠In screw-retained restorations, the fastening screw provides a solid joint between the restoration and the implant abutment, while in cement-retained prostheses the restorative screw is eliminated to enhance esthetics, occlusal stability, and passive fit of the restorations.
⢠The factors that influence the type of fixation of the prostheses to the implants are as follows.
1) Passivity of the framework.
2) Ease of fabrication and cost.
3) Occlusion.
4) Complications.
5) Esthetics.
6) Accessibility.
7) Retention.
8) Retrievability.
9) Cementation.
⢠Advantages of Screw-retained implant restorations
– Predictable retention,
– Retrievability and
– Lack of potentially retained sub-gingival cement.
⢠Disadvantages
– Ā Precise placement of the implant for the optimal and esthetic location of the screw access hole and
– Obtaining passive fit.REFERENCE
Factors influencingthe success of cement versus screw-retained implant restorations A clinical review – Journal of OsseointegrationIncorrect
ANSWER
PassivityOTHER OPTIONS
⢠Explained belowSYNOPSIS
⢠Implant restorations can be screw-retained or cement retained.
⢠In screw-retained restorations, the fastening screw provides a solid joint between the restoration and the implant abutment, while in cement-retained prostheses the restorative screw is eliminated to enhance esthetics, occlusal stability, and passive fit of the restorations.
⢠The factors that influence the type of fixation of the prostheses to the implants are as follows.
1) Passivity of the framework.
2) Ease of fabrication and cost.
3) Occlusion.
4) Complications.
5) Esthetics.
6) Accessibility.
7) Retention.
8) Retrievability.
9) Cementation.
⢠Advantages of Screw-retained implant restorations
– Predictable retention,
– Retrievability and
– Lack of potentially retained sub-gingival cement.
⢠Disadvantages
– Ā Precise placement of the implant for the optimal and esthetic location of the screw access hole and
– Obtaining passive fit.REFERENCE
Factors influencingthe success of cement versus screw-retained implant restorations A clinical review – Journal of Osseointegration -
Question 56 of 150
56. Question
A patient presents with persistent discomfort around an endodontically treated tooth 46. Radiographs show a halo-like radiolucency extending from the apical area to the furcation. What is the most likely diagnosis?
Correct
ANSWER
Vertical root fractureOTHER OPTIONS
⢠Not applicableSYNOPSIS
⢠The radiographic halo appearance is a classic sign of a vertical root fracture (VRF).
⢠This fracture typically occurs in endodontically treated teeth due to excessive force during obturation or post-placement.
⢠Other signs include isolated deep periodontal pockets and persistent sinus tracts.REFERENCE
Cohen S, Hargreaves KM. Pathways of the Pulp – 11th edition, Elsevier, 2015.Incorrect
ANSWER
Vertical root fractureOTHER OPTIONS
⢠Not applicableSYNOPSIS
⢠The radiographic halo appearance is a classic sign of a vertical root fracture (VRF).
⢠This fracture typically occurs in endodontically treated teeth due to excessive force during obturation or post-placement.
⢠Other signs include isolated deep periodontal pockets and persistent sinus tracts.REFERENCE
Cohen S, Hargreaves KM. Pathways of the Pulp – 11th edition, Elsevier, 2015. -
Question 57 of 150
57. Question
A patient presents with a fractured maxillary central incisor. The fracture extends subgingivally and involves the pulp chamber. What is the most significant factor that makes the prognosis poor?
Correct
ANSWER
Subgingival fracture extensionOTHER OPTIONS
⢠Not applicableSYNOPSIS
⢠Subgingival fracture extension compromises the biological width and makes restorability difficult.
⢠It can lead to periodontal complications and poor aesthetics.
⢠Endodontic and restorative procedures may be challenging or impossible without crown lengthening or orthodontic extrusionREFERENCE
Andreasen JO, Andreasen FM. Textbook and Color Atlas of Traumatic Injuries to the Teeth. 4th Edition, Wiley-Blackwell, 2007.Incorrect
ANSWER
Subgingival fracture extensionOTHER OPTIONS
⢠Not applicableSYNOPSIS
⢠Subgingival fracture extension compromises the biological width and makes restorability difficult.
⢠It can lead to periodontal complications and poor aesthetics.
⢠Endodontic and restorative procedures may be challenging or impossible without crown lengthening or orthodontic extrusionREFERENCE
Andreasen JO, Andreasen FM. Textbook and Color Atlas of Traumatic Injuries to the Teeth. 4th Edition, Wiley-Blackwell, 2007. -
Question 58 of 150
58. Question
A patient came for regular dental check up. You found the hairy tongue in the patient. What could be the triggering factor?
Correct
ANSWER
Poor oral hygieneOTHER OPTIONS
Not applicableSYNOPSIS
⢠Hairy tongue is condition of defective desquamation of filiform papillae.
⢠Can be caused by poor oral hygiene, smoking, excessive intake of tea or coffee etc.
⢠Filiform papillae in hairy tongue are more than 15mm in length.
⢠Patient complaints of tickling sensation in soft palate and oropharynx during swallowing.
⢠Tongue has a thick coating in the middle with a greater accentuation towards back.
⢠Treatment includes improved oral hygiene, surgical removal of papilla and controlling precipitating factors.REFERNCE
Shafer’s Textbook of Oral Pathology – 8th Edition Page No 32.Incorrect
ANSWER
Poor oral hygieneOTHER OPTIONS
Not applicableSYNOPSIS
⢠Hairy tongue is condition of defective desquamation of filiform papillae.
⢠Can be caused by poor oral hygiene, smoking, excessive intake of tea or coffee etc.
⢠Filiform papillae in hairy tongue are more than 15mm in length.
⢠Patient complaints of tickling sensation in soft palate and oropharynx during swallowing.
⢠Tongue has a thick coating in the middle with a greater accentuation towards back.
⢠Treatment includes improved oral hygiene, surgical removal of papilla and controlling precipitating factors.REFERNCE
Shafer’s Textbook of Oral Pathology – 8th Edition Page No 32. -
Question 59 of 150
59. Question
A patient came to your clinic with deep infrabony pocket and recession in 46. What is the treatment of choice
Correct
ANSWER
GTR (guided tissue regeneration)OTHER OPTIONS
⢠Open Flap Debridement – Only cleans the defect, does not regenerate lost bone or attachment.
⢠Apically Repositioned Flap – Used for pocket elimination but worsens recession, making it unsuitable.SYNOPSIS
⢠A deep infrabony pocket (defect within bone) suggests vertical bone loss, which requires regeneration rather than just resection.
⢠Recession indicates soft tissue loss, which may need coverage or preservation.
⢠GTR promotes new attachment of periodontal ligament and bone, preventing epithelial down-growth.
⢠Bone grafting enhances bone fill, improving long-term stability.REFERENCE
Carranzaās Clinical Periodontology, 14th EditionIncorrect
ANSWER
GTR (guided tissue regeneration)OTHER OPTIONS
⢠Open Flap Debridement – Only cleans the defect, does not regenerate lost bone or attachment.
⢠Apically Repositioned Flap – Used for pocket elimination but worsens recession, making it unsuitable.SYNOPSIS
⢠A deep infrabony pocket (defect within bone) suggests vertical bone loss, which requires regeneration rather than just resection.
⢠Recession indicates soft tissue loss, which may need coverage or preservation.
⢠GTR promotes new attachment of periodontal ligament and bone, preventing epithelial down-growth.
⢠Bone grafting enhances bone fill, improving long-term stability.REFERENCE
Carranzaās Clinical Periodontology, 14th Edition -
Question 60 of 150
60. Question
A 5 year old child ingested around half bottle of fluoride containing mouthwash. What is the best method to immediately treat acute fluoride toxicity?
Correct
ANSWER
Give milkOTHER OPTIONS
Not applicableSYNOPSIS
⢠Accidental ingestion of excessive amounts of fluoride containing products can lead to acute or chronic toxicity.
⢠Acute toxicity can cause a sudden onset of nausea, abdominal pain, vomiting, diarrhea, cytotoxic effects, hyperkalemia, seizures and multiple organ failure.
⢠Treatment includes inducing vomiting, give 1 percent calcium chloride or tolerated levels of milk orally and inform emergency department.REFERENCE
Pediatric Dentistry Infancy through Adolescence – 6th Edition Page No 292.Incorrect
ANSWER
Give milkOTHER OPTIONS
Not applicableSYNOPSIS
⢠Accidental ingestion of excessive amounts of fluoride containing products can lead to acute or chronic toxicity.
⢠Acute toxicity can cause a sudden onset of nausea, abdominal pain, vomiting, diarrhea, cytotoxic effects, hyperkalemia, seizures and multiple organ failure.
⢠Treatment includes inducing vomiting, give 1 percent calcium chloride or tolerated levels of milk orally and inform emergency department.REFERENCE
Pediatric Dentistry Infancy through Adolescence – 6th Edition Page No 292. -
Question 61 of 150
61. Question
A patient complaints of a feeling of toothache, with swelling of nasal mucosa, watering of eyes, bloodshot eyes and woke him from sleep. On intraoral examination, you couldn’t find any caries or tooth related problems. What can be the most probable diagnosis for this case?
Correct
ANSWER
Cluster headacheOTHER OPTIONS
⢠Migraine – Migraine is characterized by deep, aching, throbbing pain that is usually unilateral with visual aura, anorexia and even vomiting.
⢠Trigeminal neuralgia – Trigeminal neuralgia is associated with trigger zones and usually arises from acts like eating, smiling or even exposure to strong breeze.SYNOPSIS
⢠Sphenopalatine neuralgia or cluster headache is characterized by attacks of severe, strictly unilateral pain in orbital, supraorbital, temporal or in any combination.
⢠There is no trigger zone.
⢠Patients usually described their pain as toothache.
⢠Pain is also associated with swelling of mucosa, severe nasal discharge, watering of eyes, bloodshot eyes and epiphora.
⢠It can also wake up patient from sleep and is referred as alarm clock headache.
⢠Cocainization of ganglion or alcohol injection can reduce pain.REFERENCE
Shafer’s Textbook of Oral Pathology – 8th Edition Page No 573.Incorrect
ANSWER
Cluster headacheOTHER OPTIONS
⢠Migraine – Migraine is characterized by deep, aching, throbbing pain that is usually unilateral with visual aura, anorexia and even vomiting.
⢠Trigeminal neuralgia – Trigeminal neuralgia is associated with trigger zones and usually arises from acts like eating, smiling or even exposure to strong breeze.SYNOPSIS
⢠Sphenopalatine neuralgia or cluster headache is characterized by attacks of severe, strictly unilateral pain in orbital, supraorbital, temporal or in any combination.
⢠There is no trigger zone.
⢠Patients usually described their pain as toothache.
⢠Pain is also associated with swelling of mucosa, severe nasal discharge, watering of eyes, bloodshot eyes and epiphora.
⢠It can also wake up patient from sleep and is referred as alarm clock headache.
⢠Cocainization of ganglion or alcohol injection can reduce pain.REFERENCE
Shafer’s Textbook of Oral Pathology – 8th Edition Page No 573. -
Question 62 of 150
62. Question
A patient who is under benzodiazepine and digoxin for hypotension, presents to your clinic with physician consent for some treatment. While performing the treatment, the patient experienced excessive salivation and yellowish blurred vision. What is the reason for this?
Correct
ANSWER
Digoxin toxicityOTHER OPTIONS
⢠Stroke – In stroke, patient shows vision loss, facial drooping, speech difficulties, weakness, confusion, dizziness etc. Benzodiazepine toxicity – In excess benzodiazepine intake, patient complaints of slurred speech, ataxia, confusion, dizziness.
⢠Heart failure – Heart failure symptoms include chest pain, shortness of breath, swelling of legs, dizziness and sudden fatigue.SYNOPSIS
⢠Digoxin is a drug used commonly for treatment of hypotension.
⢠Digoxin toxicity can cause symptoms like nausea, vomiting, excessive salivation, yellowish – green visual problems, syncope, palpitations, tachycardia, arrhythmias etc.
⢠Kidney dysfunction and low levels of potassium can generally cause digoxin toxicity.REFERENCE
Digoxin Toxicity Article by Earl D Cummings and Henry D Swoboda.Incorrect
ANSWER
Digoxin toxicityOTHER OPTIONS
⢠Stroke – In stroke, patient shows vision loss, facial drooping, speech difficulties, weakness, confusion, dizziness etc. Benzodiazepine toxicity – In excess benzodiazepine intake, patient complaints of slurred speech, ataxia, confusion, dizziness.
⢠Heart failure – Heart failure symptoms include chest pain, shortness of breath, swelling of legs, dizziness and sudden fatigue.SYNOPSIS
⢠Digoxin is a drug used commonly for treatment of hypotension.
⢠Digoxin toxicity can cause symptoms like nausea, vomiting, excessive salivation, yellowish – green visual problems, syncope, palpitations, tachycardia, arrhythmias etc.
⢠Kidney dysfunction and low levels of potassium can generally cause digoxin toxicity.REFERENCE
Digoxin Toxicity Article by Earl D Cummings and Henry D Swoboda. -
Question 63 of 150
63. Question
Which is more flexible among the following?
Correct
ANSWER
ReamerOTHER OPTIONS
Not applicableSYNOPSIS
⢠Reamers and files are by far the most widely used root canal instruments.
⢠Traditionally, this group of instruments has been made from stainless steel
⢠Because of its fewer flutes and triangular cross section, the reamer is a much more flexible instrument than the K-file, H-file and broaches.REFERENCE
Endodontic Instruments by Pocket Dentistry.Incorrect
ANSWER
ReamerOTHER OPTIONS
Not applicableSYNOPSIS
⢠Reamers and files are by far the most widely used root canal instruments.
⢠Traditionally, this group of instruments has been made from stainless steel
⢠Because of its fewer flutes and triangular cross section, the reamer is a much more flexible instrument than the K-file, H-file and broaches.REFERENCE
Endodontic Instruments by Pocket Dentistry. -
Question 64 of 150
64. Question
A 21-year-old male presents with dull, intermittent pain in the right posterior mandible. The pain is characteristically worse at night and is relieved by NSAIDs. On radiographic examination, a small well-circumscribed radiolucent lesion with a central radiopaque nidus surrounded by sclerotic bone is seen near the mandibular body. What is the most likely diagnosis?
Correct
ANSWER
Osteoid osteomaOTHER OPTIONS
⢠Cementoblastoma – Cementoblastoma is a relatively uncommon, benign odontogenic neoplasm of jaws where lesions are generally diagnosed on a routine radiograph with sometimes localized expansion of cortical plates is observed.
⢠Osteoblastoma – Osteoblastoma lacks the characteristic pain and halo sclerotic bone which is associated with osteoid osteoma.
⢠Fibrous dysplasia – Fibrous dysplasia is seen as pain, deformity and swelling of affected jaw and mostly seen in age between 3-15 years with orange peel appearance in radiograph.SYNOPSIS
⢠Osteoid osteoma is a benign tumour of bone seen mainly in young persons below 30 years age.
⢠Condition is associated with sharp, severe pain which is worse at night and is relieved by taking NSAIDs like Aspirin.
⢠Localized swelling of soft tissue over involved area of bone may occur and may be tender.
⢠Radiographically, seen as a small ovoid or round radiolucent area surrounded by rim of sclerotic bone.
⢠Treatment includes surgical removal of lesion.REFERENCE
Shafer’s Textbook of Oral Pathology – 8th Edition Page No 208.Incorrect
ANSWER
Osteoid osteomaOTHER OPTIONS
⢠Cementoblastoma – Cementoblastoma is a relatively uncommon, benign odontogenic neoplasm of jaws where lesions are generally diagnosed on a routine radiograph with sometimes localized expansion of cortical plates is observed.
⢠Osteoblastoma – Osteoblastoma lacks the characteristic pain and halo sclerotic bone which is associated with osteoid osteoma.
⢠Fibrous dysplasia – Fibrous dysplasia is seen as pain, deformity and swelling of affected jaw and mostly seen in age between 3-15 years with orange peel appearance in radiograph.SYNOPSIS
⢠Osteoid osteoma is a benign tumour of bone seen mainly in young persons below 30 years age.
⢠Condition is associated with sharp, severe pain which is worse at night and is relieved by taking NSAIDs like Aspirin.
⢠Localized swelling of soft tissue over involved area of bone may occur and may be tender.
⢠Radiographically, seen as a small ovoid or round radiolucent area surrounded by rim of sclerotic bone.
⢠Treatment includes surgical removal of lesion.REFERENCE
Shafer’s Textbook of Oral Pathology – 8th Edition Page No 208. -
Question 65 of 150
65. Question
What type of pontic design would you give to a patient with a high esthetic demand when preparing teeth 21,22 and 23 for a FPD?
Correct
ANSWER
Modified ridge lap ponticOTHER OPTIONS
⢠Ridge lap – The ridge lap design provides reasonably good esthetics. The large, concave tissue surface of the pontic makes the removal of adherent plaque often quite difficult usually associated with inflammation and ulceration of the soft tissues.
⢠An ovate pontic – Ovate pontics are used when the residual ridge is defective or incompletely healed.
⢠Hygiene pontic – The hygienic pontic is used in the nonappearance zone, particularly for replacing mandibular first molars.SYNOPSIS
⢠The modified ridge lap design is the most common pontic form used in areas of the mouth that are visible during function (maxillary and mandibular anterior teeth and maxillary premolars and first molars).
⢠The modified ridge lap pontic combines the best features of the hygienic and saddle pontic designs, combining esthetics with easy cleaning.
⢠The modified ridge lap design overlaps the residual ridge on the facial (to achieve the appearance of a tooth emerging from the gingiva) but remains clear of the ridge on the lingual.
⢠To enable optimal plaque control, the gingival surface must have no depression or hollow. Rather, it should be as convex as possible from mesial to distal (the greater the convexity, the easier the oral hygiene).
⢠Tissue contact should resemble a letter T whose vertical arm ends at the crest of the ridge.REFERENCE
Rosenstiel, Contemporary Fixed Prosthodontics, pg 521Incorrect
ANSWER
Modified ridge lap ponticOTHER OPTIONS
⢠Ridge lap – The ridge lap design provides reasonably good esthetics. The large, concave tissue surface of the pontic makes the removal of adherent plaque often quite difficult usually associated with inflammation and ulceration of the soft tissues.
⢠An ovate pontic – Ovate pontics are used when the residual ridge is defective or incompletely healed.
⢠Hygiene pontic – The hygienic pontic is used in the nonappearance zone, particularly for replacing mandibular first molars.SYNOPSIS
⢠The modified ridge lap design is the most common pontic form used in areas of the mouth that are visible during function (maxillary and mandibular anterior teeth and maxillary premolars and first molars).
⢠The modified ridge lap pontic combines the best features of the hygienic and saddle pontic designs, combining esthetics with easy cleaning.
⢠The modified ridge lap design overlaps the residual ridge on the facial (to achieve the appearance of a tooth emerging from the gingiva) but remains clear of the ridge on the lingual.
⢠To enable optimal plaque control, the gingival surface must have no depression or hollow. Rather, it should be as convex as possible from mesial to distal (the greater the convexity, the easier the oral hygiene).
⢠Tissue contact should resemble a letter T whose vertical arm ends at the crest of the ridge.REFERENCE
Rosenstiel, Contemporary Fixed Prosthodontics, pg 521 -
Question 66 of 150
66. Question
A dental public health researcher is studying traditional oral hygiene practices. In some communities, miswak (chewing stick from Salvadora persica) is used along with the toothbrush. Miswak contains natural antibacterial agents, fluoride, silica, and resins that aid in plaque control.
Which of the following is the recommended way of using miswak and toothbrush?
Correct
ANSWER
Toothbrush used after meals. Miswak at prayer time and when out of stationOTHER OPTIONS
⢠Miswak and toothbrush must be used together – Not necessary, toothbrush alone is sufficient if available.
⢠Miswak use is not just socioeconomic, but also cultural or religious.
⢠Don’t use the miswak and use the toothbrush instead – WHO recognizes miswak as a useful traditional oral hygiene aid.SYNOPSIS
⢠Toothbrush with fluoridated toothpaste remains the gold standard for mechanical plaque removal and caries prevention.
⢠Use the miswak only when they can not afford to buy the toothbrush and toothpaste – Miswak has cultural, religious, and antibacterial benefits. It can be used as an adjunct, especially at prayer times or when toothbrush or toothpaste are unavailable (e.g., while traveling).REFERENCE
Carranzaās Clinical Periodontology, 13th EditionIncorrect
ANSWER
Toothbrush used after meals. Miswak at prayer time and when out of stationOTHER OPTIONS
⢠Miswak and toothbrush must be used together – Not necessary, toothbrush alone is sufficient if available.
⢠Miswak use is not just socioeconomic, but also cultural or religious.
⢠Don’t use the miswak and use the toothbrush instead – WHO recognizes miswak as a useful traditional oral hygiene aid.SYNOPSIS
⢠Toothbrush with fluoridated toothpaste remains the gold standard for mechanical plaque removal and caries prevention.
⢠Use the miswak only when they can not afford to buy the toothbrush and toothpaste – Miswak has cultural, religious, and antibacterial benefits. It can be used as an adjunct, especially at prayer times or when toothbrush or toothpaste are unavailable (e.g., while traveling).REFERENCE
Carranzaās Clinical Periodontology, 13th Edition -
Question 67 of 150
67. Question
What is the type of gingivitis involving the interdental papilla and may often extends into adjacent gingival margin?
Correct
ANSWER
Papillary gingivitisOTHER OPTIONS
Refer synopsis.SYNOPSIS
⢠Gingivitis is classified depending on the distribution.
⢠Marginal gingivitis involves the gingival margin, and it can include a portion of the contiguous attached gingiva.
⢠Papillary gingivitis involves the interdental papillae and it often extends into the adjacent portion of the gingival margin.
⢠Diffuse gingivitis affects the gingival margin, the attached gingiva and the
interdental papillae.REFERENCE
Gingivitis by Rehan Adhan Radhi.Incorrect
ANSWER
Papillary gingivitisOTHER OPTIONS
Refer synopsis.SYNOPSIS
⢠Gingivitis is classified depending on the distribution.
⢠Marginal gingivitis involves the gingival margin, and it can include a portion of the contiguous attached gingiva.
⢠Papillary gingivitis involves the interdental papillae and it often extends into the adjacent portion of the gingival margin.
⢠Diffuse gingivitis affects the gingival margin, the attached gingiva and the
interdental papillae.REFERENCE
Gingivitis by Rehan Adhan Radhi. -
Question 68 of 150
68. Question
A patient came with a slow growing rubbery painless swelling in front of his ear on right side. Swelling is related to superficial lobe of pleomorphic adenoma. Which tumour can be this most probably?
Correct
ANSWER
Pleomorphic adenomaOTHER OPTIONS
⢠Warthin’s tumour – Warthin’s tumour is the second most frequent benign neoplasm of the salivary glands after pleomorphic adenoma.
⢠Adenoid cystic carcinoma – Adenoid cystic carcinoma is a malignant neoplasm affecting salivary gland manifested as early local pain, facial nerve paralysis, swelling with marked tendency to spread through perineural spaces.
⢠Mucoepidermoid carcinoma – Mucoepidermoid carcinoma is the most common malignant salivary gland tumor, accounting for 10-15 percent of all salivary gland tumors and one-third of all salivary gland malignancies.SYNOPSIS
⢠Pleomorphic adenoma is the most common salivary gland tumor, is also known as benign mixed tumors.
⢠It is the commonest of all salivary gland tumors constituting up to two-thirds of all salivary gland tumors.
⢠Pleomorphic adenoma mostly presents as a solitary mobile slow growing, painless mass, which may be present for many years.
⢠In the parotid gland, signs of facial nerve weakness occur when the tumor is large.
⢠Histology will reveal proliferation of myoepithelial and epithelial cells of the ducts.
⢠Presently pleomorphic adenoma of the parotid gland is treated either with superficial or total parotidectomy or excision.REFERENCE
Shafer’s Textbook of Oral Pathology – 8th Edition Page No 266.Incorrect
ANSWER
Pleomorphic adenomaOTHER OPTIONS
⢠Warthin’s tumour – Warthin’s tumour is the second most frequent benign neoplasm of the salivary glands after pleomorphic adenoma.
⢠Adenoid cystic carcinoma – Adenoid cystic carcinoma is a malignant neoplasm affecting salivary gland manifested as early local pain, facial nerve paralysis, swelling with marked tendency to spread through perineural spaces.
⢠Mucoepidermoid carcinoma – Mucoepidermoid carcinoma is the most common malignant salivary gland tumor, accounting for 10-15 percent of all salivary gland tumors and one-third of all salivary gland malignancies.SYNOPSIS
⢠Pleomorphic adenoma is the most common salivary gland tumor, is also known as benign mixed tumors.
⢠It is the commonest of all salivary gland tumors constituting up to two-thirds of all salivary gland tumors.
⢠Pleomorphic adenoma mostly presents as a solitary mobile slow growing, painless mass, which may be present for many years.
⢠In the parotid gland, signs of facial nerve weakness occur when the tumor is large.
⢠Histology will reveal proliferation of myoepithelial and epithelial cells of the ducts.
⢠Presently pleomorphic adenoma of the parotid gland is treated either with superficial or total parotidectomy or excision.REFERENCE
Shafer’s Textbook of Oral Pathology – 8th Edition Page No 266. -
Question 69 of 150
69. Question
A patient came for orthodontic treatment with SNA 80 degrees, SNB 82 degrees and ANB 0 degree. What type of skeletal malocclusion does he have?
Correct
ANSWER
Class III malocclusionOTHER OPTIONS
Refer synopsis.SYNOPSIS
⢠If SNA is greater than 82, maxilla is considered protruded.
⢠If SNA is less than 82, maxilla is said to be retruded.
⢠If SNB is greater than 80, mandibular prognathism is noted.
⢠If SNB angle less than 80 means mandibular retrognathism.
⢠ANB angle indicates relative position of maxilla to mandible.
⢠ANB greater than 2 means a class 2 tendency.
⢠ANB less than 2 means a class 3 tendency.REFERENCE
Contemporary Orthodontics 6th Edition Page No 175.Incorrect
ANSWER
Class III malocclusionOTHER OPTIONS
Refer synopsis.SYNOPSIS
⢠If SNA is greater than 82, maxilla is considered protruded.
⢠If SNA is less than 82, maxilla is said to be retruded.
⢠If SNB is greater than 80, mandibular prognathism is noted.
⢠If SNB angle less than 80 means mandibular retrognathism.
⢠ANB angle indicates relative position of maxilla to mandible.
⢠ANB greater than 2 means a class 2 tendency.
⢠ANB less than 2 means a class 3 tendency.REFERENCE
Contemporary Orthodontics 6th Edition Page No 175. -
Question 70 of 150
70. Question
A 25 year old comes to the department of prosthodontics with a chief complaint of replacing his missing lower back teeth, which he lost during a road accident. After clinical examination, and diagnosis the prosthodontist advised patient to replace it with implants. What is the ideal bone for placing implant?
Correct
ANSWER
Dense cortical and coarse trabecularOTHER OPTIONS
Refer synopsis.SYNOPSIS
⢠Misch classified bone density into-
⢠D1 – dense cortical bone and seen in anterior mandible.
⢠D2 – thick dense to porous cortical bone crest and coarse trabecular bone within and seen in anterior and posterior mandible and anterior maxilla.
⢠Type 2 bone is the best bone for osseointegration of dental implants.
⢠It provides good cortical anchorage for primary stability, yet has better vascularity than Type 1 bone.
⢠D3 – thin porous cortical bone on crest and fine trabecular bone within and seen in posterior mandible and anterior and posterior maxilla.
⢠D4 – fine trabecular bone as seen in posterior maxilla.
⢠Thus, prosthesis movement will be more limited and the amount of bone density and volume is most favourable in anterior mandible between mental foramens.REFERENCE
Textbook of Prosthodontics, V Rangarajan – Page No 768.Incorrect
ANSWER
Dense cortical and coarse trabecularOTHER OPTIONS
Refer synopsis.SYNOPSIS
⢠Misch classified bone density into-
⢠D1 – dense cortical bone and seen in anterior mandible.
⢠D2 – thick dense to porous cortical bone crest and coarse trabecular bone within and seen in anterior and posterior mandible and anterior maxilla.
⢠Type 2 bone is the best bone for osseointegration of dental implants.
⢠It provides good cortical anchorage for primary stability, yet has better vascularity than Type 1 bone.
⢠D3 – thin porous cortical bone on crest and fine trabecular bone within and seen in posterior mandible and anterior and posterior maxilla.
⢠D4 – fine trabecular bone as seen in posterior maxilla.
⢠Thus, prosthesis movement will be more limited and the amount of bone density and volume is most favourable in anterior mandible between mental foramens.REFERENCE
Textbook of Prosthodontics, V Rangarajan – Page No 768. -
Question 71 of 150
71. Question
A 26 years old female patient came to your clinic for extraction of a grossly decayed tooth. She has some congenital valvular disease and is allergic to penicillin. On the day of procedure you asked patient to take antibiotic prophylaxis around 1 hour before procedure. Which antibiotic will you prescribe?
Correct
ANSWER
Clindamycin 600 mgOTHER OPTIONS
Not applicableSYNOPSIS
⢠Infective endocarditis develops if the patient has conditions like valvular heart disease, congenital heart disease, prosthetic valves, cardiomyopathy or congenital malformations.
⢠Dental extractions, oral prophylaxis, implant placement, biopsies, periodontal treatments, periapical endodontic treatments and any other oral procedures causing intraoral bleeding can lead to infective endocarditis in patients with cardiac issues as mentioned above.
⢠Amoxicillin 2g orally 1 hr before procedure is common antibiotic prophylaxis.
⢠If unable to take orally, ampicillin 2 g IV or IM 30 minutes before procedure is given.
⢠If allergic to pencillin, clindamycin 600 mg or azithromycin or clarithromycin 500 mg can be given orally.
⢠If unable to take orally and allergic to pencillin, clindamycin 600 mg IV or cefazolin 1 g IM or IV within 30 minutes is indicated.REFERENCE
Peterson’s Principles of Oral and Maxillofacial Surgery – 2nd Edition Page No 24.Incorrect
ANSWER
Clindamycin 600 mgOTHER OPTIONS
Not applicableSYNOPSIS
⢠Infective endocarditis develops if the patient has conditions like valvular heart disease, congenital heart disease, prosthetic valves, cardiomyopathy or congenital malformations.
⢠Dental extractions, oral prophylaxis, implant placement, biopsies, periodontal treatments, periapical endodontic treatments and any other oral procedures causing intraoral bleeding can lead to infective endocarditis in patients with cardiac issues as mentioned above.
⢠Amoxicillin 2g orally 1 hr before procedure is common antibiotic prophylaxis.
⢠If unable to take orally, ampicillin 2 g IV or IM 30 minutes before procedure is given.
⢠If allergic to pencillin, clindamycin 600 mg or azithromycin or clarithromycin 500 mg can be given orally.
⢠If unable to take orally and allergic to pencillin, clindamycin 600 mg IV or cefazolin 1 g IM or IV within 30 minutes is indicated.REFERENCE
Peterson’s Principles of Oral and Maxillofacial Surgery – 2nd Edition Page No 24. -
Question 72 of 150
72. Question
What is the function of a periodontal probe?
Correct
ANSWER
Both B and COTHER OPTIONS
⢠Determine the presence of caries – Explorer is used to determine the presence of caries.SYNOPSIS
⢠The periodontal probe is a narrow rounded or flat, blunt ended, graduated instrument.
⢠Because of its blunt end, it can be inserted into the gingival sulcus without causing trauma.
⢠Periodontal probe is used to primarily to measure periodontal probing depth.
⢠It is also used to determine the degree of gingival inflammation, to evaluate furcation lesions and also used to evaluate the extent of tooth mobility.REFERENCE
Periodontal Probe by ScienceDirect.Incorrect
ANSWER
Both B and COTHER OPTIONS
⢠Determine the presence of caries – Explorer is used to determine the presence of caries.SYNOPSIS
⢠The periodontal probe is a narrow rounded or flat, blunt ended, graduated instrument.
⢠Because of its blunt end, it can be inserted into the gingival sulcus without causing trauma.
⢠Periodontal probe is used to primarily to measure periodontal probing depth.
⢠It is also used to determine the degree of gingival inflammation, to evaluate furcation lesions and also used to evaluate the extent of tooth mobility.REFERENCE
Periodontal Probe by ScienceDirect. -
Question 73 of 150
73. Question
What is the best test for tooth with amalgam restoration?
Correct
ANSWER
Cold testOTHER OPTIONS
Not applicableSYNOPSIS
⢠Cold pulp testing has generally high diagnostic accuracy and can be considered the pulp testing method in clinical practice.
⢠Diagnostic accuracy in 86 percent.
⢠Cold pulp test is the simplest and most accurate pulpal sensibility test available to clinicians and hence should be the primary diagnostic tool to assess pulpal status.
⢠Materials used are endo ice, CO2 snow, pencil of ice, ice cold water under rubber dam isolation and ethyl chloride.REFERENCE
Cold Pulp Testing is the Simplest and Most Accurate of all Dental Pulp Sensibility Tests by PMC.Incorrect
ANSWER
Cold testOTHER OPTIONS
Not applicableSYNOPSIS
⢠Cold pulp testing has generally high diagnostic accuracy and can be considered the pulp testing method in clinical practice.
⢠Diagnostic accuracy in 86 percent.
⢠Cold pulp test is the simplest and most accurate pulpal sensibility test available to clinicians and hence should be the primary diagnostic tool to assess pulpal status.
⢠Materials used are endo ice, CO2 snow, pencil of ice, ice cold water under rubber dam isolation and ethyl chloride.REFERENCE
Cold Pulp Testing is the Simplest and Most Accurate of all Dental Pulp Sensibility Tests by PMC. -
Question 74 of 150
74. Question
A 45 year old patient came to your clinic with caries. You noticed gingival recession with respect to tooth 45 and associated root caries. Which restorative material will you used to restore this caries?
Correct
ANSWER
RMGICOTHER OPTIONS
Not applicableSYNOPSIS
⢠In a root caries procedure, lack of dentin and poor isolation may present restoration challenges.
⢠Resin modified glass ionomer cement (RMGIC) is a restorative material that can be used effectively in this situation.
⢠Clinical studies have demonstrated that RMGIC have very good retention over time due itās chemical and mechanical adhesion.
⢠It shows low polymerization shrinkage and is less adversely affected by moisture.
⢠Release fluoride to the dental structure is an important feature of RMGIC that makes it more effective in root caries.REFERENCE
What kind of restoration material is best recommended for root caries, especially where isolation is a problem by CDA Oasis.Incorrect
ANSWER
RMGICOTHER OPTIONS
Not applicableSYNOPSIS
⢠In a root caries procedure, lack of dentin and poor isolation may present restoration challenges.
⢠Resin modified glass ionomer cement (RMGIC) is a restorative material that can be used effectively in this situation.
⢠Clinical studies have demonstrated that RMGIC have very good retention over time due itās chemical and mechanical adhesion.
⢠It shows low polymerization shrinkage and is less adversely affected by moisture.
⢠Release fluoride to the dental structure is an important feature of RMGIC that makes it more effective in root caries.REFERENCE
What kind of restoration material is best recommended for root caries, especially where isolation is a problem by CDA Oasis. -
Question 75 of 150
75. Question
A 60 year old male patient came for extraction of a tooth with grade 2 mobility. Patient is on warfarin medication since 2 years. On the day before extraction her INR was 3.5. What will you do?
Correct
ANSWER
Continue taking warfarinOTHER OPTIONS
Not applicableSYNOPSIS
⢠Dentist should consult with the patientās physician to determine the level of anticoagulation being maintained with warfarin therapy.
⢠If invasive procedures or minor oral surgery are planned and the patientās INR is between 2.0 and 3.5, no adjustment in warfarin dosage is indicated.
⢠If the INR is greater than 3.5, the dentist should request that the dosage be reduced to allow the INR to fall in the range of 2.0 to 3.5.
⢠If major oral surgery is planned and the patientās INR is between 3.0 and 3.5, the dentist may request that the dosage be reduced to allow the INR to fall in the range of 2.0 to 3.0.
⢠If the dosage of warfarin is reduced by the patientās physician it will take 3 to 5 days for the desired reduction to occur.
⢠The reduction should be confirmed by INR before the dental or surgical procedure.REFERENCE
Little and Falace’s Dental Management of Medically Compromised Patients – 9th Edition Page No 450.Incorrect
ANSWER
Continue taking warfarinOTHER OPTIONS
Not applicableSYNOPSIS
⢠Dentist should consult with the patientās physician to determine the level of anticoagulation being maintained with warfarin therapy.
⢠If invasive procedures or minor oral surgery are planned and the patientās INR is between 2.0 and 3.5, no adjustment in warfarin dosage is indicated.
⢠If the INR is greater than 3.5, the dentist should request that the dosage be reduced to allow the INR to fall in the range of 2.0 to 3.5.
⢠If major oral surgery is planned and the patientās INR is between 3.0 and 3.5, the dentist may request that the dosage be reduced to allow the INR to fall in the range of 2.0 to 3.0.
⢠If the dosage of warfarin is reduced by the patientās physician it will take 3 to 5 days for the desired reduction to occur.
⢠The reduction should be confirmed by INR before the dental or surgical procedure.REFERENCE
Little and Falace’s Dental Management of Medically Compromised Patients – 9th Edition Page No 450. -
Question 76 of 150
76. Question
A 22 year old female patient came with complaint of weakness and atrophic papillae on tongue. On complete blood count test, patient had MCH less than 80. What type of anemia does she have?
Correct
ANSWER
Iron deficiency anemiaOTHER OPTIONS
⢠Thalassemia – Thalassemia shows microcytic normochromic anemia with reduced MCV and MCH and normal MCHC.
⢠Vit B12 deficiency anemia and Folate deficiency anemia – Vit B12 deficiency and folate deficiency anemia are associated with macrocytic normochromic and increasef MCV and MCH and a normal MCHC.SYNOPSIS
⢠Iron deficiency anemia is a manifestation of severe iron deficiency and is most common anemia in the world.
⢠Developed by nutritional deficiency, impaired absorption, chronic inflammation etc.
⢠Reduced MCV, MCH and MCHC with microcytic hypochromic RBCs are found in smear.
⢠Angular cheilitis, atrophic glossitis, oesophageal webs, dysphagia are seen orally.
⢠Patient may experience fatigue, leg cramps, craving for ice or cold vegetables, intolerance to cold, poor mental performance etc.
⢠Treated with iron supplements.REFERENCE
Shafer’s Textbook of Oral Pathology – 8th Edition Page No 501.Incorrect
ANSWER
Iron deficiency anemiaOTHER OPTIONS
⢠Thalassemia – Thalassemia shows microcytic normochromic anemia with reduced MCV and MCH and normal MCHC.
⢠Vit B12 deficiency anemia and Folate deficiency anemia – Vit B12 deficiency and folate deficiency anemia are associated with macrocytic normochromic and increasef MCV and MCH and a normal MCHC.SYNOPSIS
⢠Iron deficiency anemia is a manifestation of severe iron deficiency and is most common anemia in the world.
⢠Developed by nutritional deficiency, impaired absorption, chronic inflammation etc.
⢠Reduced MCV, MCH and MCHC with microcytic hypochromic RBCs are found in smear.
⢠Angular cheilitis, atrophic glossitis, oesophageal webs, dysphagia are seen orally.
⢠Patient may experience fatigue, leg cramps, craving for ice or cold vegetables, intolerance to cold, poor mental performance etc.
⢠Treated with iron supplements.REFERENCE
Shafer’s Textbook of Oral Pathology – 8th Edition Page No 501. -
Question 77 of 150
77. Question
What will happen to ANB value in a skeletal class II patient?
Correct
ANSWER
IncreasesOTHER OPTIONS
Refer synopsis.SYNOPSIS
⢠ANB angle indicates relative position of maxilla to mandible.
⢠ANB greater than 2 means a class 2 tendancy.
⢠ANB less than 2 means a class 3 tendancy.
⢠If SNA is greater than 82, maxilla is considered protruded.
⢠If SNA is less than 82, maxilla is said to be retruded.
⢠If SNB is greater than 80, mandibular prognathism is noted.
⢠If SNB angle less than 80 means mandibular retrognathism.REFERENCE
Contemporary Orthodontics 6th Edition Page No 175.Incorrect
ANSWER
IncreasesOTHER OPTIONS
Refer synopsis.SYNOPSIS
⢠ANB angle indicates relative position of maxilla to mandible.
⢠ANB greater than 2 means a class 2 tendancy.
⢠ANB less than 2 means a class 3 tendancy.
⢠If SNA is greater than 82, maxilla is considered protruded.
⢠If SNA is less than 82, maxilla is said to be retruded.
⢠If SNB is greater than 80, mandibular prognathism is noted.
⢠If SNB angle less than 80 means mandibular retrognathism.REFERENCE
Contemporary Orthodontics 6th Edition Page No 175. -
Question 78 of 150
78. Question
Which agent is used for acid etching of tooth surface?
Correct
ANSWER
37 percent phosphoric acidOTHER OPTIONS
Not applicableSYNOPSIS
⢠Acid etching is done using 37 percent phosphoric acid.
⢠It aids and enhances the formation of resin bond to teeth.
⢠On acid etching, enamel rods are dissolved to depths of 10-100 micrometers.
⢠This increases surface area, increases surface energy to enhance wetting and creates porous enamel rod ends for resin penetration.REFERENCE
Phillip’s Science of Dental Materials – 12th Edition Page No 300.Incorrect
ANSWER
37 percent phosphoric acidOTHER OPTIONS
Not applicableSYNOPSIS
⢠Acid etching is done using 37 percent phosphoric acid.
⢠It aids and enhances the formation of resin bond to teeth.
⢠On acid etching, enamel rods are dissolved to depths of 10-100 micrometers.
⢠This increases surface area, increases surface energy to enhance wetting and creates porous enamel rod ends for resin penetration.REFERENCE
Phillip’s Science of Dental Materials – 12th Edition Page No 300. -
Question 79 of 150
79. Question
Which drug among the following can cause lichenoid reaction?
Correct
ANSWER
AntimalarialOTHER OPTIONS
Not applicableSYNOPSIS
⢠Lichenoid reaction is similar to lichen planus but associated with an underlying cause.
⢠Drugs like antimalarials, NSAIDS, antihypertensives, antiretrovirals, dental restorative materials like amalgam, gold, chromium, cobalt, epoxy resins etc can trigger lichenoid reactions.
⢠Seen as greyish white streak or even ulcerations.
⢠Lesions are usually unilateral.
⢠Treatment is to identify and eliminate trigger factors.REFERENCE
Shafer’s Textbook of Oral Pathology – 8th Edition Page No 548.Incorrect
ANSWER
AntimalarialOTHER OPTIONS
Not applicableSYNOPSIS
⢠Lichenoid reaction is similar to lichen planus but associated with an underlying cause.
⢠Drugs like antimalarials, NSAIDS, antihypertensives, antiretrovirals, dental restorative materials like amalgam, gold, chromium, cobalt, epoxy resins etc can trigger lichenoid reactions.
⢠Seen as greyish white streak or even ulcerations.
⢠Lesions are usually unilateral.
⢠Treatment is to identify and eliminate trigger factors.REFERENCE
Shafer’s Textbook of Oral Pathology – 8th Edition Page No 548. -
Question 80 of 150
80. Question
What can possibly occur as a side effect of calcium hydroxide pulp therapy in primary teeth?
Correct
ANSWER
Internal root resorptionOTHER OPTIONS
Not applicableSYNOPSIS
⢠Calcium hydroxide is commonly used endodontics due to its antimicrobial effect and potential to stimulate mineralized repair of pulp and periapical tissues.
⢠But calcium hydroxide is less used in primary teeth because it can cause chronic pulpal inflammation and internal root resorption.
⢠Chronic inflammation influence macrophages to fuse to form odontoblasts which starts resorption.REFERENCE
Article on Calcium Hydroxide Induced Resorption of Deciduous teeth – Possible Explanation.Incorrect
ANSWER
Internal root resorptionOTHER OPTIONS
Not applicableSYNOPSIS
⢠Calcium hydroxide is commonly used endodontics due to its antimicrobial effect and potential to stimulate mineralized repair of pulp and periapical tissues.
⢠But calcium hydroxide is less used in primary teeth because it can cause chronic pulpal inflammation and internal root resorption.
⢠Chronic inflammation influence macrophages to fuse to form odontoblasts which starts resorption.REFERENCE
Article on Calcium Hydroxide Induced Resorption of Deciduous teeth – Possible Explanation. -
Question 81 of 150
81. Question
What is amount of force required to produce tipping in orthodontic movements?
Correct
ANSWER
50 gmOTHER OPTIONS
Not applicableSYNOPSIS
⢠Tipping is the simplest type of tooth movement where a single force is applied to the crown, which results in movement of crown in the direction of force and root in opposite direction.
⢠It is simplest among tooth movements.
⢠Controlled tipping occurs when tooth tips about a centre of rotation at its apex.
⢠Uncontrolled tipping is when tooth movement occurs about a centre of rotation apical and very close to centre of resistance.
⢠A 50 gm force directed lingually and perpendicular to the long axis of the tooth produced 0.17152 N per mm2 compressive stress at the cervical margin on the lingual side and tensile stresses 0.024357 N per mm2 at the cervical margin on the labial surface.REFERENCE
Orthodontic Force Distribution – A Three dimensional Finite Element Analysis by Hemanth M and Siddharth D Lodaya.Incorrect
ANSWER
50 gmOTHER OPTIONS
Not applicableSYNOPSIS
⢠Tipping is the simplest type of tooth movement where a single force is applied to the crown, which results in movement of crown in the direction of force and root in opposite direction.
⢠It is simplest among tooth movements.
⢠Controlled tipping occurs when tooth tips about a centre of rotation at its apex.
⢠Uncontrolled tipping is when tooth movement occurs about a centre of rotation apical and very close to centre of resistance.
⢠A 50 gm force directed lingually and perpendicular to the long axis of the tooth produced 0.17152 N per mm2 compressive stress at the cervical margin on the lingual side and tensile stresses 0.024357 N per mm2 at the cervical margin on the labial surface.REFERENCE
Orthodontic Force Distribution – A Three dimensional Finite Element Analysis by Hemanth M and Siddharth D Lodaya. -
Question 82 of 150
82. Question
A 23 year old male patient had an accident and his upper right lateral incisor got extruded. The tooth was repositioned within 30 minutes. How long should be the splinting done?
Correct
ANSWER
2 weeksOTHER OPTIONS
Not applicableSYNOPSIS
⢠Extrusive luxation is a type of traumatic injury mainly caused by forces with oblique direction, characterized by the partial or total separation of periodontal attachments.
⢠Following injury, the loosening and axial displacement of the tooth are expected.
⢠Before the accident your tooth was held in place by fibers and cells called the periodontal ligament.
⢠These fibers were torn apart and many of the cells were damaged as the tooth was torn from its location in the bone.
⢠When extruded teeth are repositioned they are quite likely to be retained for a lifetime.
⢠The longer the tooth has been extruded without repositioning, the more difficult repositioning becomes because a blood clot forms that makes repositioning less successful.
⢠Reposition the extruded tooth and stabilize it with a flexible splint for up to 2 weeks.
⢠By 48 hours it is unlikely the tooth can be repositioned to its original location.
⢠Examination at 2 weeks, 3 months, 6 months, 12 months are done.
⢠Patient should be informed of the signs and symptoms of pulp necrosis and RCT should be initiated then.REFERENCE
The Extruded Permanent Tooth – Information for dentists by The Toronto Dental Trauma Research Group.Incorrect
ANSWER
2 weeksOTHER OPTIONS
Not applicableSYNOPSIS
⢠Extrusive luxation is a type of traumatic injury mainly caused by forces with oblique direction, characterized by the partial or total separation of periodontal attachments.
⢠Following injury, the loosening and axial displacement of the tooth are expected.
⢠Before the accident your tooth was held in place by fibers and cells called the periodontal ligament.
⢠These fibers were torn apart and many of the cells were damaged as the tooth was torn from its location in the bone.
⢠When extruded teeth are repositioned they are quite likely to be retained for a lifetime.
⢠The longer the tooth has been extruded without repositioning, the more difficult repositioning becomes because a blood clot forms that makes repositioning less successful.
⢠Reposition the extruded tooth and stabilize it with a flexible splint for up to 2 weeks.
⢠By 48 hours it is unlikely the tooth can be repositioned to its original location.
⢠Examination at 2 weeks, 3 months, 6 months, 12 months are done.
⢠Patient should be informed of the signs and symptoms of pulp necrosis and RCT should be initiated then.REFERENCE
The Extruded Permanent Tooth – Information for dentists by The Toronto Dental Trauma Research Group. -
Question 83 of 150
83. Question
What type of lesion is given in the picture?
Correct
ANSWER
PustuleOTHER OPTIONS
⢠Papule – Papules are solid lesions raised above the skin surface that are smaller than 1 cm in diameter.
⢠Vesicle – Vesicle is an elevated blister containing clear fluid that are under 1 cm in diameter.
⢠Macules are well circumscribed flat lesions.SYNOPSIS
⢠Pustule – Pustule is raised lesion containing purulent material.
⢠Size – usually less than5 mm.
⢠Color – yellowish or whitish (due to pus).
⢠Surrounding area – often erythematous.
⢠Course – pustules rupture easily, leaving behind an ulcer covered with a yellow crust.
⢠Common Sites in the Oral Cavity
– Buccal mucosa
– Gingiva
– Lips
– Tongue (less common)REFERENCE
Oral and Maxillofacial Pathology, Neville – 4th Edition Page No 50.Incorrect
ANSWER
PustuleOTHER OPTIONS
⢠Papule – Papules are solid lesions raised above the skin surface that are smaller than 1 cm in diameter.
⢠Vesicle – Vesicle is an elevated blister containing clear fluid that are under 1 cm in diameter.
⢠Macules are well circumscribed flat lesions.SYNOPSIS
⢠Pustule – Pustule is raised lesion containing purulent material.
⢠Size – usually less than5 mm.
⢠Color – yellowish or whitish (due to pus).
⢠Surrounding area – often erythematous.
⢠Course – pustules rupture easily, leaving behind an ulcer covered with a yellow crust.
⢠Common Sites in the Oral Cavity
– Buccal mucosa
– Gingiva
– Lips
– Tongue (less common)REFERENCE
Oral and Maxillofacial Pathology, Neville – 4th Edition Page No 50. -
Question 84 of 150
84. Question
Where is the centre of resistance of a tooth with severe bone loss?
Correct
ANSWER
Closer to the apexOTHER OPTIONS
Not applicableSYNOPSIS
⢠As bone support around tooth decreases, the centre of resistance of tooth also changes.
⢠With the change in positioning of the center of resistance, which becomes more apical as the alveolar bone height is reduced, there is a change in the moment to force ratio.
⢠When there is alveolar bone loss, the center of resistance is apically displaced proportionally to the amount of bone height present.
⢠Therefore, it is suggested to bond the orthodontic accessories as cervically as possible, provided it does not interfere with hygiene procedures.REFERENCE
Orthodontic Treatment of Periodontal Patients – Challenges and Solutions, from Planning to Retention by PMC.Incorrect
ANSWER
Closer to the apexOTHER OPTIONS
Not applicableSYNOPSIS
⢠As bone support around tooth decreases, the centre of resistance of tooth also changes.
⢠With the change in positioning of the center of resistance, which becomes more apical as the alveolar bone height is reduced, there is a change in the moment to force ratio.
⢠When there is alveolar bone loss, the center of resistance is apically displaced proportionally to the amount of bone height present.
⢠Therefore, it is suggested to bond the orthodontic accessories as cervically as possible, provided it does not interfere with hygiene procedures.REFERENCE
Orthodontic Treatment of Periodontal Patients – Challenges and Solutions, from Planning to Retention by PMC. -
Question 85 of 150
85. Question
A child was brought to your clinic with yellow fluid filled vesicles on his lips. Some vesicles have ruptured to form painful ulcers. He had fever, pain on swallowing and lymphadenopathy. His mouth was painful with inflamed and erythematous gingiva. What condition is he suffering from?
Correct
ANSWER
Herpes simplex infectionOTHER OPTIONS
⢠Herpes zoster – Herpes zoster infection exhibits fever, malaise, linear vesicular lesions skin or mucosa supplied by affected nerve.
⢠Herpangina – Herpangina exhibits sore throat, low grade fever, headache, vomiting and abdominal pain with crops of ulcers on hard and soft palate, pharyngeal wall, buccal mucosa, tongue and anterior faucial pillars.
⢠Hand, foot and mouth disease – Hand, foot and mouth disease is seen as low grade fever, lymphadenopathy, nausea, diarrhea, vomiting, pharyngitis and vesicular lesions on hand, feet and mouth.SYNOPSIS
⢠Herpes simplex is an acute infectious disease of viral origin.
⢠Common in children and young adults.
⢠Clinically characterized by fever, irritability, headache, pain upon swallowing and regional lymphadenopathy.
⢠Mouth becomes painful with gingival erythema, edema and inflammation.
⢠Lips, tongue, buccal mucosa, palate, pharynx and tonsils may show yellowish fluid filled vesicles which may eventually rupture to form shallow, ragged painful ulcers covered by grey membrane surrounded by erythematous halo.
⢠They heal within 7-14 days and leave no scar.
⢠Antiviral drugs can be used for treatment.REFERENCE
Shafer’s Textbook of Oral Pathology – 8th Edition Page No 325.Incorrect
ANSWER
Herpes simplex infectionOTHER OPTIONS
⢠Herpes zoster – Herpes zoster infection exhibits fever, malaise, linear vesicular lesions skin or mucosa supplied by affected nerve.
⢠Herpangina – Herpangina exhibits sore throat, low grade fever, headache, vomiting and abdominal pain with crops of ulcers on hard and soft palate, pharyngeal wall, buccal mucosa, tongue and anterior faucial pillars.
⢠Hand, foot and mouth disease – Hand, foot and mouth disease is seen as low grade fever, lymphadenopathy, nausea, diarrhea, vomiting, pharyngitis and vesicular lesions on hand, feet and mouth.SYNOPSIS
⢠Herpes simplex is an acute infectious disease of viral origin.
⢠Common in children and young adults.
⢠Clinically characterized by fever, irritability, headache, pain upon swallowing and regional lymphadenopathy.
⢠Mouth becomes painful with gingival erythema, edema and inflammation.
⢠Lips, tongue, buccal mucosa, palate, pharynx and tonsils may show yellowish fluid filled vesicles which may eventually rupture to form shallow, ragged painful ulcers covered by grey membrane surrounded by erythematous halo.
⢠They heal within 7-14 days and leave no scar.
⢠Antiviral drugs can be used for treatment.REFERENCE
Shafer’s Textbook of Oral Pathology – 8th Edition Page No 325. -
Question 86 of 150
86. Question
A 42-year-old patient has undergone flap surgery for periodontitis. During the postoperative phase, the periodontist instructs him on a brushing method that avoids trauma to the healing gingiva while still allowing plaque removal, especially around the surgical site. Which brushing technique is recommended after periodontal surgery?
Correct
ANSWER
Charter techniqueOTHER OPTIONS
⢠Bass technique – Sulcular penetration so contraindicated in immediate post-surgical phase (risk of trauma)
⢠Modified Stillman – More suited for patients with gingival recession, not for surgical sites.SYNOPSIS
⢠In Charter technique bristles placed at a 45° angle directed coronally (toward occlusal surface), with gentle vibratory strokes.
⢠This avoids bristle penetration into the gingival sulcus or surgical wound, while keeping plaque away from the gingival margin.REFERENCE
Carranzaās Clinical Periodontology, 13th EditionIncorrect
ANSWER
Charter techniqueOTHER OPTIONS
⢠Bass technique – Sulcular penetration so contraindicated in immediate post-surgical phase (risk of trauma)
⢠Modified Stillman – More suited for patients with gingival recession, not for surgical sites.SYNOPSIS
⢠In Charter technique bristles placed at a 45° angle directed coronally (toward occlusal surface), with gentle vibratory strokes.
⢠This avoids bristle penetration into the gingival sulcus or surgical wound, while keeping plaque away from the gingival margin.REFERENCE
Carranzaās Clinical Periodontology, 13th Edition -
Question 87 of 150
87. Question
Which is among the following is more common site of mandibular fracture in adults?
Correct
ANSWER
AngleOTHER OPTIONS
Refer synopsis.SYNOPSIS
⢠Mandibular fractures occurs mainly due to trauma.
⢠The most common sites for the mandible to fracture are the condyle, the angle and the symphysis.
⢠Condylar fracture is the most common site of fracture which accounts for around 29.1 percent.
⢠Angle fracture is seen in around 24.5 percent people followed by symphysis fracture seen in around 22 percent individuals.
⢠Body fracture is seen in 16 percent, alveolar fracture in 3.1 percent and coronoid fracture is seen in 1.3 percent individuals.
⢠1.7 percent people might show ramus fracture.REFERENCE
Contemporary Oral and Maxillofacial Surgery St. Louis, Mosby, 2013 – 6th Edition.Incorrect
ANSWER
AngleOTHER OPTIONS
Refer synopsis.SYNOPSIS
⢠Mandibular fractures occurs mainly due to trauma.
⢠The most common sites for the mandible to fracture are the condyle, the angle and the symphysis.
⢠Condylar fracture is the most common site of fracture which accounts for around 29.1 percent.
⢠Angle fracture is seen in around 24.5 percent people followed by symphysis fracture seen in around 22 percent individuals.
⢠Body fracture is seen in 16 percent, alveolar fracture in 3.1 percent and coronoid fracture is seen in 1.3 percent individuals.
⢠1.7 percent people might show ramus fracture.REFERENCE
Contemporary Oral and Maxillofacial Surgery St. Louis, Mosby, 2013 – 6th Edition. -
Question 88 of 150
88. Question
Which among the following is best used for confirmatory diagnosis of vertical root fracture?
Correct
ANSWER
Exploratory surgeryOTHER OPTIONS
Not applicableSYNOPSIS
⢠When clinical and radiographic evaluations are equivocal, exploratory surgery may be indicated.
⢠Vertical root fractures are a significant cause of failure and may be difficult to diagnose in the early stages.
⢠The identification of a VRF may be challenging, as patients are often reluctant to undergo the surgical exploration that is required to permit direct visualization of the root surface, as the fracture may be masked by extraācoronal restorations or located subgingivally.
⢠An early stage VRF may share similar pathological signs to postātreatment endodontic disease leading to greater uncertainty in reaching an accurate diagnosis.
⢠CBCT can be also preferred.REFERENCE
Present Status and Future Directions – Vertical Root Fractures in Root Filled Teeth by PMC.Incorrect
ANSWER
Exploratory surgeryOTHER OPTIONS
Not applicableSYNOPSIS
⢠When clinical and radiographic evaluations are equivocal, exploratory surgery may be indicated.
⢠Vertical root fractures are a significant cause of failure and may be difficult to diagnose in the early stages.
⢠The identification of a VRF may be challenging, as patients are often reluctant to undergo the surgical exploration that is required to permit direct visualization of the root surface, as the fracture may be masked by extraācoronal restorations or located subgingivally.
⢠An early stage VRF may share similar pathological signs to postātreatment endodontic disease leading to greater uncertainty in reaching an accurate diagnosis.
⢠CBCT can be also preferred.REFERENCE
Present Status and Future Directions – Vertical Root Fractures in Root Filled Teeth by PMC. -
Question 89 of 150
89. Question
What is the area of insertion in Gow-Gates injection technique?
Correct
ANSWER
Mucous membrane distal to second maxillary molar just below mesiopalatal cuspOTHER OPTIONS
⢠Mucous membrane taking pterygomandibular raphae and coronoid notch as landmarks – Inferior alveolar nerve block is given in mucous membrane taking pterygomandibular raphae and coronoid notch as landmarks.
⢠Adjacent to maxillary tuberosity at the height of mucogingival junction next to maxillary third molar – Vazirani akinosi is administered adjacent to maxillary tuberosity at the height of mucogingival junction next to maxillary third molar.
⢠Mucobuccal fold above maxillary second molar – Posterior superior alveolar nerve block is administered in mucobuccal fold above maxillary second molar.SYNOPSIS
⢠Mandibular nerve block or Gow Gates technique provides sensory anesthesia to virtually the entire distribution of mandibular nerve.
⢠It anesthetizes inferior alveolar, mental, incisive, lingual, mylohyoid, auriculotemporal and buccal nerves.
⢠Area of insertion is mucous membrane on mesial of mandibular ramus, mucosa distal to maxillary second molar by placement of needle tip just below the mesiopalatal cusp of maxillary second molar.
⢠Landmarks includes center of external auditory meatus and corner of mouth extraorally and intraorally mesiopalatal cusp of upper 2nd molar and soft tissues distal to 2nd molar.REFERENCE
Handbook of Local Anesthesia, Stanley F Malamed – 7th Edition Page No 237.Incorrect
ANSWER
Mucous membrane distal to second maxillary molar just below mesiopalatal cuspOTHER OPTIONS
⢠Mucous membrane taking pterygomandibular raphae and coronoid notch as landmarks – Inferior alveolar nerve block is given in mucous membrane taking pterygomandibular raphae and coronoid notch as landmarks.
⢠Adjacent to maxillary tuberosity at the height of mucogingival junction next to maxillary third molar – Vazirani akinosi is administered adjacent to maxillary tuberosity at the height of mucogingival junction next to maxillary third molar.
⢠Mucobuccal fold above maxillary second molar – Posterior superior alveolar nerve block is administered in mucobuccal fold above maxillary second molar.SYNOPSIS
⢠Mandibular nerve block or Gow Gates technique provides sensory anesthesia to virtually the entire distribution of mandibular nerve.
⢠It anesthetizes inferior alveolar, mental, incisive, lingual, mylohyoid, auriculotemporal and buccal nerves.
⢠Area of insertion is mucous membrane on mesial of mandibular ramus, mucosa distal to maxillary second molar by placement of needle tip just below the mesiopalatal cusp of maxillary second molar.
⢠Landmarks includes center of external auditory meatus and corner of mouth extraorally and intraorally mesiopalatal cusp of upper 2nd molar and soft tissues distal to 2nd molar.REFERENCE
Handbook of Local Anesthesia, Stanley F Malamed – 7th Edition Page No 237. -
Question 90 of 150
90. Question
A patient came with a pain in upper front tooth. Patient gives a history of trauma before a few months. Clinically no findings noted. On radiographic examination, a well defined radiolucency seen in the middle third of the root canal. What is your diagnosis?
Correct
ANSWER
Internal resorptionOTHER OPTIONS
⢠Dens invaginatus – Dens invaginatus is a developmental variation which arises as an invagination in surface of tooth crown before calcification has occurred.
⢠Pulp stones – Pulp stones are foci of calcification in the pulp of tooth that are seen as radiopaque structures in pulp.
⢠Replacement resorption – Replacement resorption is a continuous process whereby the tooth is gradually replaced by bone.SYNOPSIS
⢠Internal resorption may develop from trauma, pulpotomy or crown preparation.
⢠Inflammed pulp is tissue involved in resorbing root structure.
⢠Usually asymptomatic, but can be manifested as pink tooth.
⢠Radiographically seen as a well defined, sharp and smooth radiolucency involving root canal.
⢠Immediate removal of inflamed tissue and completion of endodontic treatment can stop internal resorption.REFERENCE
Grossman’s Endodontic Practise – 13th Edition Page No 139.Incorrect
ANSWER
Internal resorptionOTHER OPTIONS
⢠Dens invaginatus – Dens invaginatus is a developmental variation which arises as an invagination in surface of tooth crown before calcification has occurred.
⢠Pulp stones – Pulp stones are foci of calcification in the pulp of tooth that are seen as radiopaque structures in pulp.
⢠Replacement resorption – Replacement resorption is a continuous process whereby the tooth is gradually replaced by bone.SYNOPSIS
⢠Internal resorption may develop from trauma, pulpotomy or crown preparation.
⢠Inflammed pulp is tissue involved in resorbing root structure.
⢠Usually asymptomatic, but can be manifested as pink tooth.
⢠Radiographically seen as a well defined, sharp and smooth radiolucency involving root canal.
⢠Immediate removal of inflamed tissue and completion of endodontic treatment can stop internal resorption.REFERENCE
Grossman’s Endodontic Practise – 13th Edition Page No 139. -
Question 91 of 150
91. Question
Which condition is associated with Bence jones protein in urine and multiple punched out radiolucencies in radiograph?
Correct
ANSWER
Multiple myelomaOTHER OPTIONS
⢠Multiple sclerosis – Multiple sclerosis is autoimmune demylinating disease of central nervous system.
⢠Osteosarcoma – Osteosarcoma is malignancy of bone that shows sunburst appearance radiographically.
⢠Paget’s disease – Paget’s disease shows cotton wool appearance radiographically.SYNOPSIS
⢠Multiple myeloma is malignancy of plasma cells.
⢠It results from mutation of plasma cells.
⢠Findings include bone pain, anemia, hypercalcemia and infections.
⢠Orally manifested as pain, swelling, expansion of jaw, numbness and mobility of teeth.
⢠Radiographically it shows numerous punched-out radiolucencies in affected bones likd ribs, skull, jaw and end of long bones.
⢠Diffuse destructive lesions of bone may also occur.
⢠Bence jones protein in urine is an important feature.
⢠Treatment includes chemotherapy, bisphosphonate therapy etc.REFERENCE
Shafer’s Textbook of Oral Pathology – 8th Edition Page No 241.Incorrect
ANSWER
Multiple myelomaOTHER OPTIONS
⢠Multiple sclerosis – Multiple sclerosis is autoimmune demylinating disease of central nervous system.
⢠Osteosarcoma – Osteosarcoma is malignancy of bone that shows sunburst appearance radiographically.
⢠Paget’s disease – Paget’s disease shows cotton wool appearance radiographically.SYNOPSIS
⢠Multiple myeloma is malignancy of plasma cells.
⢠It results from mutation of plasma cells.
⢠Findings include bone pain, anemia, hypercalcemia and infections.
⢠Orally manifested as pain, swelling, expansion of jaw, numbness and mobility of teeth.
⢠Radiographically it shows numerous punched-out radiolucencies in affected bones likd ribs, skull, jaw and end of long bones.
⢠Diffuse destructive lesions of bone may also occur.
⢠Bence jones protein in urine is an important feature.
⢠Treatment includes chemotherapy, bisphosphonate therapy etc.REFERENCE
Shafer’s Textbook of Oral Pathology – 8th Edition Page No 241. -
Question 92 of 150
92. Question
A patient came with severe lingering pain in tooth 34. On examination you noticed deep caries with respect to 34 and tenderness on percussion is positive. What is the pulp and periapical status?
Correct
ANSWER
Symptomatic irreversible pulpitis and symptomatic apical periodontitisOTHER OPTIONS
Not applicableSYNOPSIS
⢠Symptomatic irreversible pulpitis is a pulpal inflammation irreversible in nature, which is usually caused by deep caries or restorations.
⢠It is clinically manifested primarly as spontaneous pain.
⢠Pain will be sharp, severe, exaggerated, continous, lingering and nocturnal type.
⢠Radiographs show no periradicular changes.
⢠In acute or symptomatic apical periodontitis, tenderness on percussion will be present.
⢠RCT or extraction are the treatment of choice.REFERENCE
Grossman’s Endodontic Practice – 13th Edition Page No 101.Incorrect
ANSWER
Symptomatic irreversible pulpitis and symptomatic apical periodontitisOTHER OPTIONS
Not applicableSYNOPSIS
⢠Symptomatic irreversible pulpitis is a pulpal inflammation irreversible in nature, which is usually caused by deep caries or restorations.
⢠It is clinically manifested primarly as spontaneous pain.
⢠Pain will be sharp, severe, exaggerated, continous, lingering and nocturnal type.
⢠Radiographs show no periradicular changes.
⢠In acute or symptomatic apical periodontitis, tenderness on percussion will be present.
⢠RCT or extraction are the treatment of choice.REFERENCE
Grossman’s Endodontic Practice – 13th Edition Page No 101. -
Question 93 of 150
93. Question
A patient came to your clinic with a mobile tooth with respect to 36. On examination, you noticed severe vertical bone loss, grade III mobility and grade III furcation involvement around 36. What treatment will you suggest?
Correct
ANSWER
ExtractionOTHER OPTIONS
Not applicableSYNOPSIS
⢠Among the various periodontal prognosis systems, the one most widely used was proposed by McGuire and Nunn in 1996.
⢠It consists of five categories that include good, fair, poor, questionable and hopeless.
⢠In poor prognosis, it includes 50 percent attachment loss and grade II furcation.
⢠In questionable prognosis, more than 50 percent attachment loss and grade III furcation, poor crown root ratio, class II or more mobility.
⢠Hopeless includes severe attachment loss.
⢠Extraction may be preferred in this case.REFERENCE
Strategies for Periodontal Risk Assessment and Prognosis by Decisions in Dentistry.Incorrect
ANSWER
ExtractionOTHER OPTIONS
Not applicableSYNOPSIS
⢠Among the various periodontal prognosis systems, the one most widely used was proposed by McGuire and Nunn in 1996.
⢠It consists of five categories that include good, fair, poor, questionable and hopeless.
⢠In poor prognosis, it includes 50 percent attachment loss and grade II furcation.
⢠In questionable prognosis, more than 50 percent attachment loss and grade III furcation, poor crown root ratio, class II or more mobility.
⢠Hopeless includes severe attachment loss.
⢠Extraction may be preferred in this case.REFERENCE
Strategies for Periodontal Risk Assessment and Prognosis by Decisions in Dentistry. -
Question 94 of 150
94. Question
A 65 year old female patient came to your clinic with upper and lower edentulous arches for fabrication of CD. On examination you noticed severely resorbed lower arch. What type of impression will you prefer in severely resorbed arch?
Correct
ANSWER
AdmixOTHER OPTIONS
Not applicableSYNOPSIS
⢠The ‘admix impression’ is a definitive secondary impression technique used in the management of severely resorbed mandibular ridges covered with atrophic mucosa.
⢠In cases exhibiting high degree of resorption, more the denture covers the basal seat area, the better the stability of the prosthesis.
⢠To gain as much coverage of the basal seat as possible, an admixed technique is advocated.
⢠It involves making of a primary impression by mixing impression compound and green stick compound in the ratio of 3 by 7.
⢠This technique is quite useful in poor ridge cases.REFERENCE
Impression techniques for the Resorbed Mandibular arch – A Guide to Increased Guide by ResearchGate.Incorrect
ANSWER
AdmixOTHER OPTIONS
Not applicableSYNOPSIS
⢠The ‘admix impression’ is a definitive secondary impression technique used in the management of severely resorbed mandibular ridges covered with atrophic mucosa.
⢠In cases exhibiting high degree of resorption, more the denture covers the basal seat area, the better the stability of the prosthesis.
⢠To gain as much coverage of the basal seat as possible, an admixed technique is advocated.
⢠It involves making of a primary impression by mixing impression compound and green stick compound in the ratio of 3 by 7.
⢠This technique is quite useful in poor ridge cases.REFERENCE
Impression techniques for the Resorbed Mandibular arch – A Guide to Increased Guide by ResearchGate. -
Question 95 of 150
95. Question
A patient came for RCT of tooth 13. Which nerve block will you do in this case?
Correct
ANSWER
ASAOTHER OPTIONS
Not applicableSYNOPSIS
⢠The anterior superior alveolar, or infraorbital, nerve block, is a useful technique for achieving anesthesia of the maxillary central and lateral incisors and canine as well as the surrounding soft tissue on the buccal aspect.
⢠In case of RCT, pulpal anesthesia is required and achieved by ASA nerve block alone.
⢠Nasopalatine nerve block is indicated when palatal soft tissue anesthesia is also needed as in cases like minor surgical procedures.REFERENCE
Handbook of Local Anesthesia, Stanley F Malamed – 7th Edition Page No 197.Incorrect
ANSWER
ASAOTHER OPTIONS
Not applicableSYNOPSIS
⢠The anterior superior alveolar, or infraorbital, nerve block, is a useful technique for achieving anesthesia of the maxillary central and lateral incisors and canine as well as the surrounding soft tissue on the buccal aspect.
⢠In case of RCT, pulpal anesthesia is required and achieved by ASA nerve block alone.
⢠Nasopalatine nerve block is indicated when palatal soft tissue anesthesia is also needed as in cases like minor surgical procedures.REFERENCE
Handbook of Local Anesthesia, Stanley F Malamed – 7th Edition Page No 197. -
Question 96 of 150
96. Question
A 48 year old woman presented with pain and bleeding of gums. Clinical examination showed generalized probing depths of 5-8 mm, generalized clinical attachment loss of 5 mm, bleeding of 30 percent. Radiographic analysis showed 49 percent horizontal bone loss. What is the stage of periodontitis?
Correct
ANSWER
Generalized stage III, grade COTHER OPTIONS
Not applicableSYNOPSIS
⢠Localized means less than 30 percent teeth are involved.
⢠Generalized means more than 30 percent teeth are involved.
⢠Stage 1 Periodontitis – In stage 1 periodontitis, clinical attachment loss will be 1-2mm and probing depth tends to be less than or equal to 4mm with horizontal bone loss.
⢠Stage 2 Periodontitis – Stage 2 periodontitis is characterized by a CAL of 3-4 mm, probing depth less than or equal to 5mm and mostly horizontal bone loss.
⢠Stage 3 Periodontitis – In stage 3 periodontitis, clinical attachment loss will be greater than or equal to 5mm and probing depth tends to be greater than or equal to 6 mm with vertical bone loss.
⢠Stage 4 Periodontitis – In stage 4 periodontitis, clinical attachment loss will be greater than or equal to 5mm with severe mobility, teeth loss and masticatory dysfunction.
⢠Grade A is if ratio of percentage of bone loss to age is less than 0.25.
⢠Grade B is if ratio of percentage of bone loss to age is between 0.25-1.
⢠Grade C is if ratio of percentage of bone loss to age is more than 1.REFERENCE
Staging and Grading Periodontitis by AAP.Incorrect
ANSWER
Generalized stage III, grade COTHER OPTIONS
Not applicableSYNOPSIS
⢠Localized means less than 30 percent teeth are involved.
⢠Generalized means more than 30 percent teeth are involved.
⢠Stage 1 Periodontitis – In stage 1 periodontitis, clinical attachment loss will be 1-2mm and probing depth tends to be less than or equal to 4mm with horizontal bone loss.
⢠Stage 2 Periodontitis – Stage 2 periodontitis is characterized by a CAL of 3-4 mm, probing depth less than or equal to 5mm and mostly horizontal bone loss.
⢠Stage 3 Periodontitis – In stage 3 periodontitis, clinical attachment loss will be greater than or equal to 5mm and probing depth tends to be greater than or equal to 6 mm with vertical bone loss.
⢠Stage 4 Periodontitis – In stage 4 periodontitis, clinical attachment loss will be greater than or equal to 5mm with severe mobility, teeth loss and masticatory dysfunction.
⢠Grade A is if ratio of percentage of bone loss to age is less than 0.25.
⢠Grade B is if ratio of percentage of bone loss to age is between 0.25-1.
⢠Grade C is if ratio of percentage of bone loss to age is more than 1.REFERENCE
Staging and Grading Periodontitis by AAP. -
Question 97 of 150
97. Question
Each of the following is correct except?
Correct
ANSWER
Bad breath originating from the gastrointestinal tract is quite commonOTHER OPTIONS
⢠Refer SynopsisSYNOPSIS
⢠Bad breath appears to be largely bacterial in origin – The primary cause of halitosis is volatile sulfur compounds (VSCs) produced by anaerobic bacteria in the oral cavity (mainly the tongue dorsum, periodontal pockets, and throat).
⢠Gastrointestinal causes are uncommon and account for less than 1 percent of halitosis cases.
⢠Self-perceptions of bad breath appear to be unreliable. Studies show many patients who believe they have halitosis actually donāt (pseudo-halitosis), while some with halitosis are unaware.REFERENCE
Carranzaās Clinical Periodontology, 13th EditionIncorrect
ANSWER
Bad breath originating from the gastrointestinal tract is quite commonOTHER OPTIONS
⢠Refer SynopsisSYNOPSIS
⢠Bad breath appears to be largely bacterial in origin – The primary cause of halitosis is volatile sulfur compounds (VSCs) produced by anaerobic bacteria in the oral cavity (mainly the tongue dorsum, periodontal pockets, and throat).
⢠Gastrointestinal causes are uncommon and account for less than 1 percent of halitosis cases.
⢠Self-perceptions of bad breath appear to be unreliable. Studies show many patients who believe they have halitosis actually donāt (pseudo-halitosis), while some with halitosis are unaware.REFERENCE
Carranzaās Clinical Periodontology, 13th Edition -
Question 98 of 150
98. Question
Which probe is given in the picture?
Correct
ANSWER
William probeOTHER OPTIONS
⢠Marquis color coded probe – Marquis probe has calibrations at 3,6,9,12 mm sections.
⢠UNC 15 probe – UNC 15 probe is 15mm long with millimeter markings and color coding at 5,10 and 15mm.
⢠WHO probe – WHO probe has 0.5mm ball at tip and millimeter markings at 3.5, 8.5 and 11.5 mm and color coding from 3.5-5.5 mm.SYNOPSIS
⢠William Probe was introduced by Charles H M Williams.
⢠It is stainless steel probe with 1mm diameter and 13 mm length with a blunt tip end.
⢠All marking from 1 to 10 mm except 4 and 6 are missing, which minimizes confusion during reading markings.
⢠It is intended to measure depth of periodontal pockets.REFERENCE
Article on Complete Guide – Periodontal instruments and their Uses.Incorrect
ANSWER
William probeOTHER OPTIONS
⢠Marquis color coded probe – Marquis probe has calibrations at 3,6,9,12 mm sections.
⢠UNC 15 probe – UNC 15 probe is 15mm long with millimeter markings and color coding at 5,10 and 15mm.
⢠WHO probe – WHO probe has 0.5mm ball at tip and millimeter markings at 3.5, 8.5 and 11.5 mm and color coding from 3.5-5.5 mm.SYNOPSIS
⢠William Probe was introduced by Charles H M Williams.
⢠It is stainless steel probe with 1mm diameter and 13 mm length with a blunt tip end.
⢠All marking from 1 to 10 mm except 4 and 6 are missing, which minimizes confusion during reading markings.
⢠It is intended to measure depth of periodontal pockets.REFERENCE
Article on Complete Guide – Periodontal instruments and their Uses. -
Question 99 of 150
99. Question
Which of the following are important factors that must be evaluated before performing a laterally repositioned flap?
Correct
ANSWER
All of the aboveOTHER OPTIONS
⢠NilSYNOPSIS
⢠Before performing a laterally repositioned flap (laterally positioned flap, LRP) for root coverage or gingival augmentation, the following factors must be evaluated
⢠Presence of bone on the facial surface of the donor tooth – Adequate bone support is crucial to ensure flap survival and prevent recession.
⢠Thickness of the gingiva at the donor site – Thicker gingiva enhances vascular supply and flap stability, leading to better healing.
⢠Width of attached gingiva at the donor site – A wider band of attached gingiva ensures sufficient donor tissue for successful flap mobilization.REFERENCE
Lindhe 6th edition chapter 39
Carranza 11th edition chapter 63Incorrect
ANSWER
All of the aboveOTHER OPTIONS
⢠NilSYNOPSIS
⢠Before performing a laterally repositioned flap (laterally positioned flap, LRP) for root coverage or gingival augmentation, the following factors must be evaluated
⢠Presence of bone on the facial surface of the donor tooth – Adequate bone support is crucial to ensure flap survival and prevent recession.
⢠Thickness of the gingiva at the donor site – Thicker gingiva enhances vascular supply and flap stability, leading to better healing.
⢠Width of attached gingiva at the donor site – A wider band of attached gingiva ensures sufficient donor tissue for successful flap mobilization.REFERENCE
Lindhe 6th edition chapter 39
Carranza 11th edition chapter 63 -
Question 100 of 150
100. Question
A 5 year old child was brought to your clinic for pulpectomy of tooth 85. Patient has high caries index. You completed pulpectomy. What is the most appropriate definite restoration to avoid future fracture of the remaining few surfaces of tooth left?
Correct
ANSWER
Stainless steel crownOTHER OPTIONS
Not applicableSYNOPSIS
⢠Stainless steel crowns are indicated after pulpotomy or pulpectomy specially for primary first molar.
⢠Used when three or more surfaces need restoration.
⢠Used as an abutment for fixed appliances.
⢠SCCs are also used in high caries index patient.
⢠Strong consideration should be given to the use of stainless steel crowns in children who require general anesthesia.REFERENCE
The Use of Stainless Steel Crowns by AAPD.Incorrect
ANSWER
Stainless steel crownOTHER OPTIONS
Not applicableSYNOPSIS
⢠Stainless steel crowns are indicated after pulpotomy or pulpectomy specially for primary first molar.
⢠Used when three or more surfaces need restoration.
⢠Used as an abutment for fixed appliances.
⢠SCCs are also used in high caries index patient.
⢠Strong consideration should be given to the use of stainless steel crowns in children who require general anesthesia.REFERENCE
The Use of Stainless Steel Crowns by AAPD. -
Question 101 of 150
101. Question
A 4 year old child came with deep dental caries in tooth 84. During carious removal you accidently exposed pulp. What treatment will you prefer?
Correct
ANSWER
PulpotomyOTHER OPTIONS
Not applicableSYNOPSIS
⢠When the carious process has reached the pulp or in incidences of direct pulpal exposure during excavation of a carious lesion, the pulpotomy procedure is indicated and is the treatment of choice.
⢠Pulpotomy procedure is based on the rationale that the radicular pulp tissue is healthy or is capable of healing after surgical amputation of the affected or infected coronal pulp.
⢠The presence of any signs or symptoms of inflammation extending beyond the coronal pulp is a contraindication for a pulpotomy.
⢠Thus, a pulpotomy is contraindicated when there is any swelling (of pulpal origin), fistula, pathologic mobility, pathologic external root resorption, internal root resorption, periapical or interradicular radiolucency, pulp calcifications, or excessive bleeding from the amputated radicular stumps.REFERENCE
Pediatric Dentistry Infancy through Adolescence – 6th Edition Page No 340.Incorrect
ANSWER
PulpotomyOTHER OPTIONS
Not applicableSYNOPSIS
⢠When the carious process has reached the pulp or in incidences of direct pulpal exposure during excavation of a carious lesion, the pulpotomy procedure is indicated and is the treatment of choice.
⢠Pulpotomy procedure is based on the rationale that the radicular pulp tissue is healthy or is capable of healing after surgical amputation of the affected or infected coronal pulp.
⢠The presence of any signs or symptoms of inflammation extending beyond the coronal pulp is a contraindication for a pulpotomy.
⢠Thus, a pulpotomy is contraindicated when there is any swelling (of pulpal origin), fistula, pathologic mobility, pathologic external root resorption, internal root resorption, periapical or interradicular radiolucency, pulp calcifications, or excessive bleeding from the amputated radicular stumps.REFERENCE
Pediatric Dentistry Infancy through Adolescence – 6th Edition Page No 340. -
Question 102 of 150
102. Question
You are performing a 5 year old follow up on a 37 year old patient with an implant. When comparing radiograph, you noticed of almost 1 mm bone loss since the placement of implant. Which of the following is indicated?
Correct
ANSWER
Implant is doing wellOTHER OPTIONS
Not applicableSYNOPSIS
⢠The rate of crestal bone loss in the region can be used to determine the long term success of dental implant.
⢠A crucial success factor was suggested as crestal bone loss of less than 1.5 mm in the first year following loading.
⢠Less than 0.2 mm annually after that is also considered normal.REFERENCE
Article on Comparative Evaluation of Crestal Bone Level around Pre and Post-Loaded Endosseous Implants.Incorrect
ANSWER
Implant is doing wellOTHER OPTIONS
Not applicableSYNOPSIS
⢠The rate of crestal bone loss in the region can be used to determine the long term success of dental implant.
⢠A crucial success factor was suggested as crestal bone loss of less than 1.5 mm in the first year following loading.
⢠Less than 0.2 mm annually after that is also considered normal.REFERENCE
Article on Comparative Evaluation of Crestal Bone Level around Pre and Post-Loaded Endosseous Implants. -
Question 103 of 150
103. Question
Which among the following is a main disadvantage of automated probe?
Correct
ANSWER
Less tactile sensitivityOTHER OPTIONS
Refer synopsis.SYNOPSIS
⢠Automated probes like florida probe and interprobe measures probing depth electronically and transferred automatically to computer.
⢠These probes have more reproducibility, less variability and does not require any assistance to record data.
⢠But these probes underestimate probing depth due to their less tactile sensitivity.REFERENCE
Newman and Carranza’s Clinical Periodontology – 13th Edition Page No 388.Incorrect
ANSWER
Less tactile sensitivityOTHER OPTIONS
Refer synopsis.SYNOPSIS
⢠Automated probes like florida probe and interprobe measures probing depth electronically and transferred automatically to computer.
⢠These probes have more reproducibility, less variability and does not require any assistance to record data.
⢠But these probes underestimate probing depth due to their less tactile sensitivity.REFERENCE
Newman and Carranza’s Clinical Periodontology – 13th Edition Page No 388. -
Question 104 of 150
104. Question
What is the most observed tissue response in patient when the oral hygiene instructions are followed by him correctly?
Correct
ANSWER
Less bleeding on probingOTHER OPTIONS
⢠Increased attachment level – Would suggest improvement, but requires long-term monitoring. Not as immediate or sensitive as BOP for short-term evaluation.
⢠Decreased plaque score – Reflects improved oral hygiene. Helpful, but not as directly correlated with tissue health as BOP.SYNOPSIS
⢠BOP is an early and sensitive sign of inflammation.
⢠Its absence is a strong predictor of periodontal stability.
⢠According to Lang & Tonetti (2003) and Carranzaās Clinical Periodontology, absence of BOP is a key indicator of periodontal health after therapy.
⢠Highly reproducible and easily measurable in clinical practice.REFERENCE
Carranzaās Clinical Periodontology, 13th EditionIncorrect
ANSWER
Less bleeding on probingOTHER OPTIONS
⢠Increased attachment level – Would suggest improvement, but requires long-term monitoring. Not as immediate or sensitive as BOP for short-term evaluation.
⢠Decreased plaque score – Reflects improved oral hygiene. Helpful, but not as directly correlated with tissue health as BOP.SYNOPSIS
⢠BOP is an early and sensitive sign of inflammation.
⢠Its absence is a strong predictor of periodontal stability.
⢠According to Lang & Tonetti (2003) and Carranzaās Clinical Periodontology, absence of BOP is a key indicator of periodontal health after therapy.
⢠Highly reproducible and easily measurable in clinical practice.REFERENCE
Carranzaās Clinical Periodontology, 13th Edition -
Question 105 of 150
105. Question
A 5 year old child weighing 20 kg have fever. Doctor prescribed acetaminophen 160 mg per 5ml syrup. What is the maximum ml of syrup to consume per day?
Correct
ANSWER
9 mlOTHER OPTIONS
Not applicableSYNOPSIS
⢠Acetaminophen in children is given to a dose of 10-15 mg per kg per dose.
⢠In a child of 20 kg, maximum dose of acetaminophen that can be given is 20 multiplied with 15 and that is 300 mg per dose.
⢠In a commercially available syrup 160 mg of acetaminophen is present in 5 ml.
⢠So, at least 9 ml can be consumed per dose to maximum by a child of 20kg.REFERENCE
Pediatric Acetaminophen Dosage By Medscape.Incorrect
ANSWER
9 mlOTHER OPTIONS
Not applicableSYNOPSIS
⢠Acetaminophen in children is given to a dose of 10-15 mg per kg per dose.
⢠In a child of 20 kg, maximum dose of acetaminophen that can be given is 20 multiplied with 15 and that is 300 mg per dose.
⢠In a commercially available syrup 160 mg of acetaminophen is present in 5 ml.
⢠So, at least 9 ml can be consumed per dose to maximum by a child of 20kg.REFERENCE
Pediatric Acetaminophen Dosage By Medscape. -
Question 106 of 150
106. Question
A patient came to your clinic. After explaining about the treatment plan, the patient gave you permission to start the procedure. This comes under?
Correct
ANSWER
ConsentOTHER OPTIONS
⢠Justice – Justice is principle concerned with providing individuals or groups with what is owed, dued and deserved.
⢠Paternalism – Paternalism means the health care professional acts as a parent and makes decisions for the patient on the basis of what the professional believes is in the best interest of the patient.SYNOPSIS
⢠Informed consent is the basis for every treatment you propose to and perform on patients.
⢠Dentists must obtain informed consent from each patient or from the patientās legal guardian or decision-maker.
⢠State laws impact whether consent can be verbal or written.
⢠For common simple procedures such as an evaluation or prophylaxis for a healthy individual, the act of sitting in the dental chair may be interpreted as implied or waived consent.REFERENCE
Informed Consent or Refusal by ADA.Incorrect
ANSWER
ConsentOTHER OPTIONS
⢠Justice – Justice is principle concerned with providing individuals or groups with what is owed, dued and deserved.
⢠Paternalism – Paternalism means the health care professional acts as a parent and makes decisions for the patient on the basis of what the professional believes is in the best interest of the patient.SYNOPSIS
⢠Informed consent is the basis for every treatment you propose to and perform on patients.
⢠Dentists must obtain informed consent from each patient or from the patientās legal guardian or decision-maker.
⢠State laws impact whether consent can be verbal or written.
⢠For common simple procedures such as an evaluation or prophylaxis for a healthy individual, the act of sitting in the dental chair may be interpreted as implied or waived consent.REFERENCE
Informed Consent or Refusal by ADA. -
Question 107 of 150
107. Question
A 8 year old child was brought to your clinic with class 2 anterior open bite. Which among the following is best indicated in such cases?
Correct
ANSWER
High pull headgearOTHER OPTIONS
⢠Reverse pull headgear or Face mask – Reverse pull headgear or Face mask is used in class III cases with maxillary hypoplasia.
⢠Cervical headgear – Cervical headgear is used in correcting class II especially in distalizing maxilla and maxillary molars.SYNOPSIS
⢠High pull headgear derive anchorage from parietal region of cranium.
⢠They produce vertically directed force on maxilla and maxillary dentition.
⢠These can be used to produce intrusive forces on anterior region of maxilla thereby producing counter clockwise moment of the maxilla.
⢠This is beneficial in correction of vertical maxillary excess and gummy smile.
⢠Intrusive forces on posterior aspect of maxilla can be of benefit in anterior open bite patients as it intrudes the maxillary molars and therefore produces a clockwise moment of the maxilla.REFERENCE
Orthodontics The Art and Science – 6th Edition Page No 456.Incorrect
ANSWER
High pull headgearOTHER OPTIONS
⢠Reverse pull headgear or Face mask – Reverse pull headgear or Face mask is used in class III cases with maxillary hypoplasia.
⢠Cervical headgear – Cervical headgear is used in correcting class II especially in distalizing maxilla and maxillary molars.SYNOPSIS
⢠High pull headgear derive anchorage from parietal region of cranium.
⢠They produce vertically directed force on maxilla and maxillary dentition.
⢠These can be used to produce intrusive forces on anterior region of maxilla thereby producing counter clockwise moment of the maxilla.
⢠This is beneficial in correction of vertical maxillary excess and gummy smile.
⢠Intrusive forces on posterior aspect of maxilla can be of benefit in anterior open bite patients as it intrudes the maxillary molars and therefore produces a clockwise moment of the maxilla.REFERENCE
Orthodontics The Art and Science – 6th Edition Page No 456. -
Question 108 of 150
108. Question
What is the concept of leeway space?
Correct
ANSWER
Combined mesiodistal width of permanent canines and premolars is less than that of primary canines and molarsOTHER OPTIONS
Not applicableSYNOPSIS
⢠Second transitional period during mixed dentition stage is characterized by replacement of deciduos molar and canines by premolars and permanent cuspids respectively.
⢠Combined mesiodistal width of permanent canines and premolars is less than that of the deciduous molars and canines and this surplus is called leeway space of Nance.
⢠Amount of leeway space is greater in mandibular arch than maxillary arch.
⢠It is about 1.7 mm on each side or arch (3.4 mm total) in mandible.
⢠It is about 0.9 mm on each side or arch (1.8 mm total) in maxilla.REFERENCE
Orthodontics The Art and Science – 6th Edition Page No 58.Incorrect
ANSWER
Combined mesiodistal width of permanent canines and premolars is less than that of primary canines and molarsOTHER OPTIONS
Not applicableSYNOPSIS
⢠Second transitional period during mixed dentition stage is characterized by replacement of deciduos molar and canines by premolars and permanent cuspids respectively.
⢠Combined mesiodistal width of permanent canines and premolars is less than that of the deciduous molars and canines and this surplus is called leeway space of Nance.
⢠Amount of leeway space is greater in mandibular arch than maxillary arch.
⢠It is about 1.7 mm on each side or arch (3.4 mm total) in mandible.
⢠It is about 0.9 mm on each side or arch (1.8 mm total) in maxilla.REFERENCE
Orthodontics The Art and Science – 6th Edition Page No 58. -
Question 109 of 150
109. Question
What is the mechanism of action of chlorhexidine?
Correct
ANSWER
Disrupts membrane integrityOTHER OPTIONS
Not applicableSYNOPSIS
⢠Chlorhexidine is a broad spectrum biocide effective against gram positive bacteria, gram-negative bacteria and fungi.
⢠Chlorhexidine inactivates microorganisms with a broader spectrum than other antimicrobials (e.g. antibiotics) and has a quicker kill rate than other antimicrobials (e.g. povidone-iodine).
⢠It has both bacteriostatic (inhibits bacterial growth) and bactericidal (kills bacteria) mechanisms of action, depending on its concentration.
⢠Chlorhexidine kills by disrupting the cell membrane.
⢠Upon application in vitro, chlorhexidine can kill nearly 100 percent of gram positive and gram negative bacteria within 30 seconds.
⢠Chlorhexidine is a positively charged molecule that binds to the negatively charged sites on the cell wall, it destabilizes the cell wall and interferes with osmosis.REFERENCE
Chlorhexidine in Dentistry – Pharmacology, Uses and Adverse effects by PMC.Incorrect
ANSWER
Disrupts membrane integrityOTHER OPTIONS
Not applicableSYNOPSIS
⢠Chlorhexidine is a broad spectrum biocide effective against gram positive bacteria, gram-negative bacteria and fungi.
⢠Chlorhexidine inactivates microorganisms with a broader spectrum than other antimicrobials (e.g. antibiotics) and has a quicker kill rate than other antimicrobials (e.g. povidone-iodine).
⢠It has both bacteriostatic (inhibits bacterial growth) and bactericidal (kills bacteria) mechanisms of action, depending on its concentration.
⢠Chlorhexidine kills by disrupting the cell membrane.
⢠Upon application in vitro, chlorhexidine can kill nearly 100 percent of gram positive and gram negative bacteria within 30 seconds.
⢠Chlorhexidine is a positively charged molecule that binds to the negatively charged sites on the cell wall, it destabilizes the cell wall and interferes with osmosis.REFERENCE
Chlorhexidine in Dentistry – Pharmacology, Uses and Adverse effects by PMC. -
Question 110 of 150
110. Question
A 4 years old patient visits your hospital after bicycle accident. In clinical examination you found the primary tooth intruded into the permanent follicle. What is the effect it will cause to permanent tooth?
Correct
ANSWER
Turner’s hypoplasiaOTHER OPTIONS
⢠Dens in dente – A condition resulting from invagination of the inner enamel epithelium producing the appearance of a tooth within a tooth.
⢠Dens evagenatus – Describes an outfolding of the enamel organ that results in an extra cusp, usually in the central groove or ridge of posterior teeth and in the cingulum area of the anterior teeth, sometimes called a talon cusp.
⢠Ankylosis – Fusion of tooth to boneSYNOPSIS
⢠Trauma to the primary teeth may progress to cellulitis, and may result in aborted development or enamel hypoplasia of the succedaneous tooth.
⢠Enamel hypoplasia or hypomineralization can be generalized throughout the dentition or it can be localized.
⢠Enamel development, or amelogenesis, is an exquisitely regulated process at the molecular level but can be disrupted by many environmental factors, such as fever, infection, trauma, changes in oxygen saturation, antibiotics, and many other factors.
⢠Trauma leads to hypomineralization to marked enamel hypoplasia, which manifests as blue-gray to yellow-brown color.
⢠The altered tooth is called Turner’s tooth.REFERENCE
Pediatric dentistry Arthur.J.Nowak PageIncorrect
ANSWER
Turner’s hypoplasiaOTHER OPTIONS
⢠Dens in dente – A condition resulting from invagination of the inner enamel epithelium producing the appearance of a tooth within a tooth.
⢠Dens evagenatus – Describes an outfolding of the enamel organ that results in an extra cusp, usually in the central groove or ridge of posterior teeth and in the cingulum area of the anterior teeth, sometimes called a talon cusp.
⢠Ankylosis – Fusion of tooth to boneSYNOPSIS
⢠Trauma to the primary teeth may progress to cellulitis, and may result in aborted development or enamel hypoplasia of the succedaneous tooth.
⢠Enamel hypoplasia or hypomineralization can be generalized throughout the dentition or it can be localized.
⢠Enamel development, or amelogenesis, is an exquisitely regulated process at the molecular level but can be disrupted by many environmental factors, such as fever, infection, trauma, changes in oxygen saturation, antibiotics, and many other factors.
⢠Trauma leads to hypomineralization to marked enamel hypoplasia, which manifests as blue-gray to yellow-brown color.
⢠The altered tooth is called Turner’s tooth.REFERENCE
Pediatric dentistry Arthur.J.Nowak Page -
Question 111 of 150
111. Question
A 7 year old child came with unerupted upper central incisors and gives a history of primary centrals were exfoliated 1 year back. On radiographic examination, dentist noticed unerupted mesiodens between unerupted central incisors. What treatment will you prefer?
Correct
ANSWER
Extract supernumerary and wait for central incisors to eruptOTHER OPTIONS
Not applicableSYNOPSIS
⢠Unerupted maxillary incisors may result secondary to supernumerary teeth.
⢠Early surgical extraction of a mesiodens or mesiodens, ideally before 7 years of age.
⢠In order to promote eruption and proper alignment of adjacent teeth, it is recommended to extract mesiodens in the early mixed dentition, which may reduce the need for orthodontic treatment.
⢠It might take six months to three years for an unerupted tooth to erupt after removal of the mesiodens.
⢠If treatment is postponed after this age, more complex surgical and orthodontic treatment may be necessary.
⢠The type and position of the unerupted tooth, the space available in the dental arch, in addition to the stage of root development may influence how long it takes for an impacted tooth to erupt after surgical removal of the mesiodens.REFERENCE
Diagnosis and Management of Supernumerary (Mesiodens) – A Review of the Literature by PMC.Incorrect
ANSWER
Extract supernumerary and wait for central incisors to eruptOTHER OPTIONS
Not applicableSYNOPSIS
⢠Unerupted maxillary incisors may result secondary to supernumerary teeth.
⢠Early surgical extraction of a mesiodens or mesiodens, ideally before 7 years of age.
⢠In order to promote eruption and proper alignment of adjacent teeth, it is recommended to extract mesiodens in the early mixed dentition, which may reduce the need for orthodontic treatment.
⢠It might take six months to three years for an unerupted tooth to erupt after removal of the mesiodens.
⢠If treatment is postponed after this age, more complex surgical and orthodontic treatment may be necessary.
⢠The type and position of the unerupted tooth, the space available in the dental arch, in addition to the stage of root development may influence how long it takes for an impacted tooth to erupt after surgical removal of the mesiodens.REFERENCE
Diagnosis and Management of Supernumerary (Mesiodens) – A Review of the Literature by PMC. -
Question 112 of 150
112. Question
On routine examination of a child’s oral cavity, you noticed some bulge on the lingual surface of on mandibular central incisor. What is the condition?
Correct
ANSWER
Talon cuspOTHER OPTIONS
⢠Fusion – Fusion is union of two normally separate tooth germs.
⢠Dens in dente – Dens in dente or dens invaginatus is a developmental variation which is thought to arise as a result of invagination in surface of tooth crown.
⢠Enamel pearls – Ectopic formation of enamel in form of globule on root surface.SYNOPSIS
⢠Talon cusp or eagle’s talon projects lingually from cingulum areas of maxillary or mandibular permanent incisors.
⢠It can poses problems such as esthetics, caries control and occlusal accommodation.
⢠It there is occlusal interference, it should be removed but exposure of the pulp horn, necessitating endodontic therapy.REFERENCE
Shafer’s Textbook of Oral Pathology – 8th Edition Page No 41.Incorrect
ANSWER
Talon cuspOTHER OPTIONS
⢠Fusion – Fusion is union of two normally separate tooth germs.
⢠Dens in dente – Dens in dente or dens invaginatus is a developmental variation which is thought to arise as a result of invagination in surface of tooth crown.
⢠Enamel pearls – Ectopic formation of enamel in form of globule on root surface.SYNOPSIS
⢠Talon cusp or eagle’s talon projects lingually from cingulum areas of maxillary or mandibular permanent incisors.
⢠It can poses problems such as esthetics, caries control and occlusal accommodation.
⢠It there is occlusal interference, it should be removed but exposure of the pulp horn, necessitating endodontic therapy.REFERENCE
Shafer’s Textbook of Oral Pathology – 8th Edition Page No 41. -
Question 113 of 150
113. Question
A 50 year old female patient came for extraction of a mobile tooth. You successfully extracted the tooth under LA. Which among the following teeth require more lingual force during extraction?
Correct
ANSWER
Mandibular molarOTHER OPTIONS
Not applicableSYNOPSIS
⢠Initially the movements are gentle with buccal and lingual pressure.
⢠Buccolingual pressure but since the lingual alveolar bone is very thin compared to the buccal alveolar bone, therefore, the force that mobilizes the tooth must be more in the lingual direction.
⢠After the tooth is mobilized, force is gradually increased and the final extraction movement is buccal.REFERENCE
Principles of Exodontia by Dr Hamid Hammad Enezei.Incorrect
ANSWER
Mandibular molarOTHER OPTIONS
Not applicableSYNOPSIS
⢠Initially the movements are gentle with buccal and lingual pressure.
⢠Buccolingual pressure but since the lingual alveolar bone is very thin compared to the buccal alveolar bone, therefore, the force that mobilizes the tooth must be more in the lingual direction.
⢠After the tooth is mobilized, force is gradually increased and the final extraction movement is buccal.REFERENCE
Principles of Exodontia by Dr Hamid Hammad Enezei. -
Question 114 of 150
114. Question
Crater in the interdental area indicate which of the following?
Correct
ANSWER
2 Wall bone defectOTHER OPTIONS
⢠3 Wall bone defectĀ – The three-wall vertical defect was originally called an infrabony defect. This defect appears most frequently on the mesial aspects of second and third maxillary and mandibular molars.
⢠One-wall bone defect – The one-wall vertical defect is also called a hemiseptum.
⢠Combined wall defect – The number of walls in the apical portion of the defect may be greater than that in its occlusal portion, in which case the term combined osseous defect is used.SYNOPSIS
⢠A crater in the interdental area is a common type of intrabony defect where the buccal and lingual/palatal walls remain intact, but the mesial and distal walls are lost.
⢠This creates a 2-wall defect (buccal and lingual walls are present, while mesial and distal walls are missing).
⢠Craters are the most common osseous defects seen in periodontal disease, especially in posterior teeth.REFERENCE
Periobasics 1st edition chapter 35Incorrect
ANSWER
2 Wall bone defectOTHER OPTIONS
⢠3 Wall bone defectĀ – The three-wall vertical defect was originally called an infrabony defect. This defect appears most frequently on the mesial aspects of second and third maxillary and mandibular molars.
⢠One-wall bone defect – The one-wall vertical defect is also called a hemiseptum.
⢠Combined wall defect – The number of walls in the apical portion of the defect may be greater than that in its occlusal portion, in which case the term combined osseous defect is used.SYNOPSIS
⢠A crater in the interdental area is a common type of intrabony defect where the buccal and lingual/palatal walls remain intact, but the mesial and distal walls are lost.
⢠This creates a 2-wall defect (buccal and lingual walls are present, while mesial and distal walls are missing).
⢠Craters are the most common osseous defects seen in periodontal disease, especially in posterior teeth.REFERENCE
Periobasics 1st edition chapter 35 -
Question 115 of 150
115. Question
Which type of local anesthetic will you recommend for a pregnant woman who came to your clinic?
Correct
ANSWER
LignocaineOTHER OPTIONS
⢠NILSYNOPSIS
⢠The second trimester is weeks 14 through 27 in terms of gestational age.
⢠The risk of the teratogenic effects of drugs is lower during this period than during the first trimester.
⢠Elective dental treatment has been reported to be relatively safe during this period.
⢠Moreover, as the extent of physiological changes that occur during the second trimester is not too considerable, anesthesia is safer to perform in pregnant women for non-obstetric surgery in the second trimester than in the first or third trimester.
⢠Lidocaine with adrenaline is considered safe in pregnancy.
⢠Vasoconstrictors, mostly epinephrine, are added to lidocaine to reduce the absorption of the local anesthetic, reduce toxicity, and increase the analgesic effects.
⢠Vasoconstriction induced by epinephrine delays the absorption of local anesthetics by the mother, allowing the absorption of lidocaine to gradually occur in the maternal systemic circulation, while also allowing blood levels of lidocaine to gradually increase.
⢠The local anesthetic is transferred to the fetus slowly, and its margin of safety is also increased.REFERENCE
Use of local anesthetics for dental treatment during pregnancy, safety for parturient – Journal of Dental Anesthesia and Pain MedicineIncorrect
ANSWER
LignocaineOTHER OPTIONS
⢠NILSYNOPSIS
⢠The second trimester is weeks 14 through 27 in terms of gestational age.
⢠The risk of the teratogenic effects of drugs is lower during this period than during the first trimester.
⢠Elective dental treatment has been reported to be relatively safe during this period.
⢠Moreover, as the extent of physiological changes that occur during the second trimester is not too considerable, anesthesia is safer to perform in pregnant women for non-obstetric surgery in the second trimester than in the first or third trimester.
⢠Lidocaine with adrenaline is considered safe in pregnancy.
⢠Vasoconstrictors, mostly epinephrine, are added to lidocaine to reduce the absorption of the local anesthetic, reduce toxicity, and increase the analgesic effects.
⢠Vasoconstriction induced by epinephrine delays the absorption of local anesthetics by the mother, allowing the absorption of lidocaine to gradually occur in the maternal systemic circulation, while also allowing blood levels of lidocaine to gradually increase.
⢠The local anesthetic is transferred to the fetus slowly, and its margin of safety is also increased.REFERENCE
Use of local anesthetics for dental treatment during pregnancy, safety for parturient – Journal of Dental Anesthesia and Pain Medicine -
Question 116 of 150
116. Question
A 1 year old child came to your dental hospital along with parents. The child has upper central incisor in his mouth. When will be the root formation complete for that tooth?
Correct
ANSWER
After 6 monthsOTHER OPTIONS
⢠NilSYNOPSIS
⢠Primary maxillary incisor hard tissue formation begins at 4 months in utero.
⢠Enamel is completed at 1.5 months after birth.
⢠Eruption at 7 months.
⢠Root completed at 1.5 years or 18 months.REFERENCE
Pediatric dentistry Arthur.J.Nowak Page 196Incorrect
ANSWER
After 6 monthsOTHER OPTIONS
⢠NilSYNOPSIS
⢠Primary maxillary incisor hard tissue formation begins at 4 months in utero.
⢠Enamel is completed at 1.5 months after birth.
⢠Eruption at 7 months.
⢠Root completed at 1.5 years or 18 months.REFERENCE
Pediatric dentistry Arthur.J.Nowak Page 196 -
Question 117 of 150
117. Question
1.Increasing mobility
2. Pathologic migration
3. Inflamed gingiva
4. Alveolar bone loss.
Which of these are associated with secondary occlusal trauma?Correct
ANSWER
1, 2, 4 (Increasing mobility, Pathologic migration, Alveolar bone loss)OTHER OPTIONS
⢠Inflamed gingiva – While inflammation is common in periodontitis, it is not a direct sign of secondary occlusal trauma; it is primarily related to plaque-induced gingivitis or periodontitis.SYNOPSIS
⢠Secondary occlusal trauma occurs when excessive occlusal forces are applied to teeth that already have a compromised periodontium due to pre-existing bone loss from periodontitis.
⢠Increasing mobility – Due to loss of periodontal support.
Pathologic migration – Shifting of teeth due to loss of bone and occlusal forces.
Alveolar bone loss – Already present due to periodontitis, worsened by trauma.REFERENCE
Lindhe J, Lang NP. Clinical Periodontology and Implant Dentistry, 6th ed., Wiley-Blackwell.Incorrect
ANSWER
1, 2, 4 (Increasing mobility, Pathologic migration, Alveolar bone loss)OTHER OPTIONS
⢠Inflamed gingiva – While inflammation is common in periodontitis, it is not a direct sign of secondary occlusal trauma; it is primarily related to plaque-induced gingivitis or periodontitis.SYNOPSIS
⢠Secondary occlusal trauma occurs when excessive occlusal forces are applied to teeth that already have a compromised periodontium due to pre-existing bone loss from periodontitis.
⢠Increasing mobility – Due to loss of periodontal support.
Pathologic migration – Shifting of teeth due to loss of bone and occlusal forces.
Alveolar bone loss – Already present due to periodontitis, worsened by trauma.REFERENCE
Lindhe J, Lang NP. Clinical Periodontology and Implant Dentistry, 6th ed., Wiley-Blackwell. -
Question 118 of 150
118. Question
Which among the following is the largest cusp?
Correct
ANSWER
Mesiolingual cusp of 1st upper molarOTHER OPTIONS
Not applicableSYNOPSIS
⢠Maxillary first molar is the largest tooth in the maxillary arch.
⢠It has four well-developed functioning cusps.
⢠The four cusps are mesiobuccal, distobuccal, mesiolingual and distolingual cusps.
⢠A supplemental cusp is called the cusp or tubercle of carabelli.
⢠The mesiolingual cusp is the largest cusp, it is followed in point of size by mesiobuccal, distolingual, distobuccal and fifth cusps.
⢠Outline of occlusal aspect is somewhat rhomboidal.REFERENCE
Wheeler’s Dental Anatomy, Physiology and Occlusion – First South Asian Edition.Incorrect
ANSWER
Mesiolingual cusp of 1st upper molarOTHER OPTIONS
Not applicableSYNOPSIS
⢠Maxillary first molar is the largest tooth in the maxillary arch.
⢠It has four well-developed functioning cusps.
⢠The four cusps are mesiobuccal, distobuccal, mesiolingual and distolingual cusps.
⢠A supplemental cusp is called the cusp or tubercle of carabelli.
⢠The mesiolingual cusp is the largest cusp, it is followed in point of size by mesiobuccal, distolingual, distobuccal and fifth cusps.
⢠Outline of occlusal aspect is somewhat rhomboidal.REFERENCE
Wheeler’s Dental Anatomy, Physiology and Occlusion – First South Asian Edition. -
Question 119 of 150
119. Question
Which among the following may be a cause of bad breath?
Correct
ANSWER
All of the aboveOTHER OPTIONS
⢠NilSYNOPSIS
⢠Bad breath is caused by odor-producing bacteria that grow in the mouth.
⢠When you don’t brush and floss regularly, bacteria accumulate on the bits of food left in your mouth and between your teeth.
⢠The sulfur compounds released by these bacteria make your breath smell.
⢠Many other diseases and illnesses can cause bad breath, including Seasonal allergies, Respiratory tract infections such as pneumonia or bronchitis, Long-term, (chronic) sinus infections, Postnasal drip, Diabetes, Chronic acid reflux.REFERENCE
Dental Health and Bad Breath. By Alfred D. Wyatt.Incorrect
ANSWER
All of the aboveOTHER OPTIONS
⢠NilSYNOPSIS
⢠Bad breath is caused by odor-producing bacteria that grow in the mouth.
⢠When you don’t brush and floss regularly, bacteria accumulate on the bits of food left in your mouth and between your teeth.
⢠The sulfur compounds released by these bacteria make your breath smell.
⢠Many other diseases and illnesses can cause bad breath, including Seasonal allergies, Respiratory tract infections such as pneumonia or bronchitis, Long-term, (chronic) sinus infections, Postnasal drip, Diabetes, Chronic acid reflux.REFERENCE
Dental Health and Bad Breath. By Alfred D. Wyatt. -
Question 120 of 150
120. Question
During embryological development, the given condition results from failure of fusion between which of the following structures?
Correct
ANSWER
Maxillary process and medial nasal processOTHER OPTIONS
⢠Lateral nasal process and medial nasal process – failure leads to nasal defects, not cleft lip.
⢠Maxillary and lateral nasal process – forms side of nose, not lip.
⢠Right and left medial nasal processes – failure causes median cleft lip.SYNOPSIS
⢠Unilateral cleft lip forms when the maxillary process fails to fuse with the medial nasal process on one side.REFERENCE
Langmanās Medical Embryology, 14th ed.Incorrect
ANSWER
Maxillary process and medial nasal processOTHER OPTIONS
⢠Lateral nasal process and medial nasal process – failure leads to nasal defects, not cleft lip.
⢠Maxillary and lateral nasal process – forms side of nose, not lip.
⢠Right and left medial nasal processes – failure causes median cleft lip.SYNOPSIS
⢠Unilateral cleft lip forms when the maxillary process fails to fuse with the medial nasal process on one side.REFERENCE
Langmanās Medical Embryology, 14th ed. -
Question 121 of 150
121. Question
A patient came with pain after 3 days after extraction with pain in the area of extraction. On examination, you realized that the patient has dry socket. What treatment will you prefer?
Correct
ANSWER
Irrigate with mouthrinse and pack with eugenol dressingOTHER OPTIONS
Not applicableSYNOPSIS
⢠Alveolar osteitis or dry socket is a severely painful complication arising between one and three days of extraction.
⢠Patient will experience severe, debilitating constant pain that continue through the night.
⢠Clinically an empty socket with exposed bone is seen.
⢠Adjacent gingiva tends to be red, inflamed and edematous.
⢠Range of treatments for a dry socket include treatments directed locally to the socket, including irrigation of the socket with a 0.12-0.2 percent chlorhexidine rinse and instructing in home use of a syringe for irrigation, placement of a self eliminating dressing such as Alvogyl(containing eugenol, butamben and iodoform), placement of an obtundent dressing such as zinc oxide, eugenol and lidocaine gel, or a combination of these therapies and where appropriate, the prescription of systemic antibiotics.REFERENCE
The Management of Alveolar Osteitis by ResearchGate.Incorrect
ANSWER
Irrigate with mouthrinse and pack with eugenol dressingOTHER OPTIONS
Not applicableSYNOPSIS
⢠Alveolar osteitis or dry socket is a severely painful complication arising between one and three days of extraction.
⢠Patient will experience severe, debilitating constant pain that continue through the night.
⢠Clinically an empty socket with exposed bone is seen.
⢠Adjacent gingiva tends to be red, inflamed and edematous.
⢠Range of treatments for a dry socket include treatments directed locally to the socket, including irrigation of the socket with a 0.12-0.2 percent chlorhexidine rinse and instructing in home use of a syringe for irrigation, placement of a self eliminating dressing such as Alvogyl(containing eugenol, butamben and iodoform), placement of an obtundent dressing such as zinc oxide, eugenol and lidocaine gel, or a combination of these therapies and where appropriate, the prescription of systemic antibiotics.REFERENCE
The Management of Alveolar Osteitis by ResearchGate. -
Question 122 of 150
122. Question
What will be the cause if a patient came with sensitivity after placing composite?
Correct
ANSWER
Polymerization shrinkageOTHER OPTION
⢠NILSYNOPSIS
⢠Post-operative sensitivity in teeth after doing a composite restoration is caused by residual stress buildup due to polymerization shrinkage resulting in de-bonding of the restoration ensuing in an enamel crack, microleakage at the margins of the restoration and secondary caries results in postoperative sensitivity.REFERENCE
Immediate Post-Operative Sensitivity After Composite Resin Restoration ā A Review of Treatment Protocol Monu SurvasheIncorrect
ANSWER
Polymerization shrinkageOTHER OPTION
⢠NILSYNOPSIS
⢠Post-operative sensitivity in teeth after doing a composite restoration is caused by residual stress buildup due to polymerization shrinkage resulting in de-bonding of the restoration ensuing in an enamel crack, microleakage at the margins of the restoration and secondary caries results in postoperative sensitivity.REFERENCE
Immediate Post-Operative Sensitivity After Composite Resin Restoration ā A Review of Treatment Protocol Monu Survashe -
Question 123 of 150
123. Question
Behaviour modification is based on the principle of
Correct
ANSWER
Social learning theoryOTHER OPTIONS
⢠NilSYNOPSIS
⢠Behaviour shaping is a form of behaviour modification technique based on principles of Social learning
⢠It is the procedure that slowly develops desired behaviour
⢠The behaviour shaping techniques are
– Desensitisation(TSD)
– Modelling
– Contingency ManagementREFERENCE
Textbook of Pedodontics by Shobha TandonIncorrect
ANSWER
Social learning theoryOTHER OPTIONS
⢠NilSYNOPSIS
⢠Behaviour shaping is a form of behaviour modification technique based on principles of Social learning
⢠It is the procedure that slowly develops desired behaviour
⢠The behaviour shaping techniques are
– Desensitisation(TSD)
– Modelling
– Contingency ManagementREFERENCE
Textbook of Pedodontics by Shobha Tandon -
Question 124 of 150
124. Question
What should be the minimum distance between mandibular nerve and apex of implant(in mm)?
Correct
ANSWER
2 mmOTHER OPTIONS
Not applicableSYNOPSIS
⢠A safety distance of 2 mm has been widely accepted as the standard measurement to prevent implant related nerve injuries.
⢠Implant placement is the second most common cause for IAN damage, following the removal of impacted wisdom teeth.
⢠New studies says that if one can avoid thermal, pressure, and traumatic damage to the IAN, then implants can be placed closer than 2 mm to the Inferior Alveolar Canal.REFERENCE
A New Concept of Safety Distance to Place Implants in the Area of the Inferior Alveolar Canal to Avoid Neurosensory Disturbance.Incorrect
ANSWER
2 mmOTHER OPTIONS
Not applicableSYNOPSIS
⢠A safety distance of 2 mm has been widely accepted as the standard measurement to prevent implant related nerve injuries.
⢠Implant placement is the second most common cause for IAN damage, following the removal of impacted wisdom teeth.
⢠New studies says that if one can avoid thermal, pressure, and traumatic damage to the IAN, then implants can be placed closer than 2 mm to the Inferior Alveolar Canal.REFERENCE
A New Concept of Safety Distance to Place Implants in the Area of the Inferior Alveolar Canal to Avoid Neurosensory Disturbance. -
Question 125 of 150
125. Question
You were presented with a patient who complains of spontaneous radiating pain of an amalgam-filled tooth. On examination, it showed sensitivity to cold and lingering pain. Identify the diagnosis.
Correct
ANSWER
Irreversible pulpitisOTHER OPTIONS
⢠Apical periodontitis – Apical periodontitis (AP) is an acute or chronic inflammatory lesion around the apex of a tooth root, most commonly caused by bacterial invasion of the pulp of the tooth.
⢠Acute apical abscess – It is the most common form of dental abscess and is caused by infection of the root canal of the tooth. It is usually localized intraorally, but in some cases the apical abscess may spread and result in severe complications or even mortality.
⢠Other options donot applySYNOPSIS
⢠In the early stages of irreversible pulpitis, a paroxysm of pain may be caused by the following- sudden temperature changes, particularly cold- sweet or acid foodstuffs- and pressure from packing food into a cavity or suction exerted by the tongue or cheek.
⢠Symptomatic irreversible pulpitis exhibits pain usually caused by a hot or cold stimulus, or pain that occurs spontaneously.
⢠The pain persists for several minutes to hours, lingering after removal of the thermal stimulus.
⢠The patient may describe the pain as sharp, piercing, or shooting, and it is generally severe.
⢠It may be intermittent or continuous, depending on the degree of pulpal involvement and on whether it is related to an external stimulus.
⢠The patient may also complain of postural pain, i.e., change of position (bending over or lying down) exacerbates the pain. This is due to the increase in intrapulpal pressure when the patient changes position from a standing posture to a supine (lying down) posture.
⢠The patient may also have pain referred to adjacent teeth, to the temple or sinuses when an upper posterior tooth is involved, or to the ear when a lower posterior tooth is affected.
⢠Patients are often kept awake at night by the pain (nocturnal pain), which continues to be intolerable despite all their efforts at analgesia.REFERENCE
Grossman s Endodontic Practice-14th.edIncorrect
ANSWER
Irreversible pulpitisOTHER OPTIONS
⢠Apical periodontitis – Apical periodontitis (AP) is an acute or chronic inflammatory lesion around the apex of a tooth root, most commonly caused by bacterial invasion of the pulp of the tooth.
⢠Acute apical abscess – It is the most common form of dental abscess and is caused by infection of the root canal of the tooth. It is usually localized intraorally, but in some cases the apical abscess may spread and result in severe complications or even mortality.
⢠Other options donot applySYNOPSIS
⢠In the early stages of irreversible pulpitis, a paroxysm of pain may be caused by the following- sudden temperature changes, particularly cold- sweet or acid foodstuffs- and pressure from packing food into a cavity or suction exerted by the tongue or cheek.
⢠Symptomatic irreversible pulpitis exhibits pain usually caused by a hot or cold stimulus, or pain that occurs spontaneously.
⢠The pain persists for several minutes to hours, lingering after removal of the thermal stimulus.
⢠The patient may describe the pain as sharp, piercing, or shooting, and it is generally severe.
⢠It may be intermittent or continuous, depending on the degree of pulpal involvement and on whether it is related to an external stimulus.
⢠The patient may also complain of postural pain, i.e., change of position (bending over or lying down) exacerbates the pain. This is due to the increase in intrapulpal pressure when the patient changes position from a standing posture to a supine (lying down) posture.
⢠The patient may also have pain referred to adjacent teeth, to the temple or sinuses when an upper posterior tooth is involved, or to the ear when a lower posterior tooth is affected.
⢠Patients are often kept awake at night by the pain (nocturnal pain), which continues to be intolerable despite all their efforts at analgesia.REFERENCE
Grossman s Endodontic Practice-14th.ed -
Question 126 of 150
126. Question
What will be the size of pontic design of an FPD?
Correct
ANSWER
Smaller than missing tooth buccolinguallyOTHER OPTIONS
⢠Wider buccolingually – Wider pontics are difficult to cleanse and decrease chewing efficiency.SYNOPSIS
⢠Reducing the buccolingual width of the pontic by as much as 30 percent has been suggested as a way to lessen occlusal forces on, and thus the loading of, abutment teeth.
⢠Critical analysis reveals that forces are lessened only when chewing food of uniform consistency and that a mere 12 percent increase in chewing efficiency can be expected from a one-third reduction of pontic width.
⢠Decreasing the buccolingual width leads to a decrease in interferences in eccentric movements.
⢠Narrowing the occlusal table may actually impede or even preclude the development of a harmonious and stable occlusal relationship.
⢠Like a malposed tooth, it may cause difficulties in plaque control and may not provide proper cheek support. For these reasons, pontics with normal occlusal widths (at least on the occlusal third) are generally recommended.REFERENCE
Rosenstiel, Contemporary Fixed Prosthodontics, pg 527Incorrect
ANSWER
Smaller than missing tooth buccolinguallyOTHER OPTIONS
⢠Wider buccolingually – Wider pontics are difficult to cleanse and decrease chewing efficiency.SYNOPSIS
⢠Reducing the buccolingual width of the pontic by as much as 30 percent has been suggested as a way to lessen occlusal forces on, and thus the loading of, abutment teeth.
⢠Critical analysis reveals that forces are lessened only when chewing food of uniform consistency and that a mere 12 percent increase in chewing efficiency can be expected from a one-third reduction of pontic width.
⢠Decreasing the buccolingual width leads to a decrease in interferences in eccentric movements.
⢠Narrowing the occlusal table may actually impede or even preclude the development of a harmonious and stable occlusal relationship.
⢠Like a malposed tooth, it may cause difficulties in plaque control and may not provide proper cheek support. For these reasons, pontics with normal occlusal widths (at least on the occlusal third) are generally recommended.REFERENCE
Rosenstiel, Contemporary Fixed Prosthodontics, pg 527 -
Question 127 of 150
127. Question
What is the ideal angle between rest and minor connector?
Correct
ANSWER
Less than 90 degreesOTHER OPTIONS
⢠More than 90 degrees – An angle greater than 90 degrees fails to transmit occlusal forces along the supporting vertical axis of the abutment tooth. This also permits slippage of the prosthesis away from the abutment, which can result in orthodontic-like forces being applied to an inclined plane on the abutment, with possible tooth movement.SYNOPSIS
⢠The angle formed by the occlusal rest and the vertical minor connector from which it originates should be less than 90 degrees.
⢠Only in this way can the occlusal forces be directed along the long axis of the abutment tooth.REFERENCE
Chapter 6 Rests and Rest Seats. Pocket dentistry.Incorrect
ANSWER
Less than 90 degreesOTHER OPTIONS
⢠More than 90 degrees – An angle greater than 90 degrees fails to transmit occlusal forces along the supporting vertical axis of the abutment tooth. This also permits slippage of the prosthesis away from the abutment, which can result in orthodontic-like forces being applied to an inclined plane on the abutment, with possible tooth movement.SYNOPSIS
⢠The angle formed by the occlusal rest and the vertical minor connector from which it originates should be less than 90 degrees.
⢠Only in this way can the occlusal forces be directed along the long axis of the abutment tooth.REFERENCE
Chapter 6 Rests and Rest Seats. Pocket dentistry. -
Question 128 of 150
128. Question
A child has a congenital defect that presents with micrognathia, glossoptosis and cleft palate. What is the condition?
Correct
ANSWER
Pierre Robin syndromeOTHER OPTIONS
⢠Papillone leferve syndrome – Papillone leferve syndrome is characterized by the development of dry scaly patches of skin and periodontitis.
⢠Melkersson-Rosenthal syndrome – Melkersson-Rosenthal syndrome is a rare neurological disorder characterized by recurring facial paralysis, swelling of the face and lips and the development of folds and furrows in the tongue.
⢠Van der Woude syndrome – Van der Woude syndrome is a genetic form of cleft lip and palate.SYNOPSIS
⢠Pierre Robin syndrome is a congenital defect that presents with a classic triad of micrognathia or retrognathia, glossoptosis and cleft palate.
⢠Affected neonates suffer airway obstruction amd feeding difficulties.
⢠Primary clinical defect is presence of small mandible with an obtuse gonial angle and posteriorly located condyle or the jaw size is normal but the mandible is retropositioned with respect to maxilla.
⢠Patient will have a bird face.
⢠Tongue tends to fall back causing airway obstruction.
⢠Otitis media, hearing loss, nasal deformities, labrinthitis with equilibrium disturbance, dental and philtral malformations are also seen.
⢠Multidisciplinary approach is used for treatment.REFERENCE
Shafer’s Textbook of Oral Pathology – 8th Edition Page No 7.Incorrect
ANSWER
Pierre Robin syndromeOTHER OPTIONS
⢠Papillone leferve syndrome – Papillone leferve syndrome is characterized by the development of dry scaly patches of skin and periodontitis.
⢠Melkersson-Rosenthal syndrome – Melkersson-Rosenthal syndrome is a rare neurological disorder characterized by recurring facial paralysis, swelling of the face and lips and the development of folds and furrows in the tongue.
⢠Van der Woude syndrome – Van der Woude syndrome is a genetic form of cleft lip and palate.SYNOPSIS
⢠Pierre Robin syndrome is a congenital defect that presents with a classic triad of micrognathia or retrognathia, glossoptosis and cleft palate.
⢠Affected neonates suffer airway obstruction amd feeding difficulties.
⢠Primary clinical defect is presence of small mandible with an obtuse gonial angle and posteriorly located condyle or the jaw size is normal but the mandible is retropositioned with respect to maxilla.
⢠Patient will have a bird face.
⢠Tongue tends to fall back causing airway obstruction.
⢠Otitis media, hearing loss, nasal deformities, labrinthitis with equilibrium disturbance, dental and philtral malformations are also seen.
⢠Multidisciplinary approach is used for treatment.REFERENCE
Shafer’s Textbook of Oral Pathology – 8th Edition Page No 7. -
Question 129 of 150
129. Question
What is resonance frequency analysis used for?
Correct
ANSWER
Check implant stabilityOTHER OPTIONS
Not applicableSYNOPSIS
⢠One of the direct methods for evaluating osteointegration is the resonance frequency analysis (RFA) that provides valuable clinical objective data of implant stability.
⢠The technique of resonant frequency analysis is noninvasive and nondestructive, essentially a test of the stability for the dental implant.
⢠It is equivalent in terms of the direction and the type of application of fixed lateral forces to the implant and the measurement of the implant displacement.REFERENCE
Influence of Resonance Frequency Analysis (RFA) Measurements for Successful Osseointegration of Dental Implants During the Healing Period and Its Impact on Implant Assessed by Osstell Mentor Device by PMC.Incorrect
ANSWER
Check implant stabilityOTHER OPTIONS
Not applicableSYNOPSIS
⢠One of the direct methods for evaluating osteointegration is the resonance frequency analysis (RFA) that provides valuable clinical objective data of implant stability.
⢠The technique of resonant frequency analysis is noninvasive and nondestructive, essentially a test of the stability for the dental implant.
⢠It is equivalent in terms of the direction and the type of application of fixed lateral forces to the implant and the measurement of the implant displacement.REFERENCE
Influence of Resonance Frequency Analysis (RFA) Measurements for Successful Osseointegration of Dental Implants During the Healing Period and Its Impact on Implant Assessed by Osstell Mentor Device by PMC. -
Question 130 of 150
130. Question
A 35-year-old patient presents with flushing and sweating on one side of the face after eating certain foods. The patient reports that this has been occurring for several months. On examination, there is no evidence of infection or other abnormalities. On taking history, patient reported to have a surgical procedure done in parotid area. Given these symptoms, what is the most likely diagnosis?
Correct
ANSWER
Frey’s syndromeOTHER OPTIONS
⢠Bell’s palsy – Bell’s Palsy is characterized by sudden weakness or paralysis of the facial muscles on one side of the face due to a dysfunction of the facial nerve, but it does not cause flushing and sweating.
⢠Sjogren syndrome – Sjƶgren’s Syndrome is an autoimmune disorder that primarily affects the glands that produce tears and saliva, leading to dry eyes and mouth, but it does not cause the flushing and sweating described.
⢠Trigeminal neuralgia – Trigeminal Neuralgia is a chronic pain condition affecting the trigeminal nerve, causing intense facial pain, but it does not cause flushing and sweating.SYNOPSIS
⢠Frey’s Syndrome, also known as gustatory sweating, is characterized by flushing and sweating on one side of the face, usually in the region of the parotid gland, after eating or thinking about certain foods.
⢠This condition often occurs after surgery or trauma to the parotid gland or the surrounding area, leading to aberrant nerve regeneration.
⢠For patients with more severe and bothersome symptoms, there are several options for treatment.
⢠Medical treatments include
– Topical anticholinergic ointments (scopolamine, glycopyrrolate)
– Topical antiperspirants (deodorant)
– Topical α agonist (clonidine)
– Botulinum toxin injections
⢠Botulinum toxin appears to be the easiest and safest method. It provides the longest period of symptom relief with the lowest complications. However, none of these treatments allow a definitive cure; relief is only temporary.
⢠For permanent treatment, reconstructive surgery is the only option.ĀREFERENCE
Shafer’s Textbook of Oral PathologyIncorrect
ANSWER
Frey’s syndromeOTHER OPTIONS
⢠Bell’s palsy – Bell’s Palsy is characterized by sudden weakness or paralysis of the facial muscles on one side of the face due to a dysfunction of the facial nerve, but it does not cause flushing and sweating.
⢠Sjogren syndrome – Sjƶgren’s Syndrome is an autoimmune disorder that primarily affects the glands that produce tears and saliva, leading to dry eyes and mouth, but it does not cause the flushing and sweating described.
⢠Trigeminal neuralgia – Trigeminal Neuralgia is a chronic pain condition affecting the trigeminal nerve, causing intense facial pain, but it does not cause flushing and sweating.SYNOPSIS
⢠Frey’s Syndrome, also known as gustatory sweating, is characterized by flushing and sweating on one side of the face, usually in the region of the parotid gland, after eating or thinking about certain foods.
⢠This condition often occurs after surgery or trauma to the parotid gland or the surrounding area, leading to aberrant nerve regeneration.
⢠For patients with more severe and bothersome symptoms, there are several options for treatment.
⢠Medical treatments include
– Topical anticholinergic ointments (scopolamine, glycopyrrolate)
– Topical antiperspirants (deodorant)
– Topical α agonist (clonidine)
– Botulinum toxin injections
⢠Botulinum toxin appears to be the easiest and safest method. It provides the longest period of symptom relief with the lowest complications. However, none of these treatments allow a definitive cure; relief is only temporary.
⢠For permanent treatment, reconstructive surgery is the only option.ĀREFERENCE
Shafer’s Textbook of Oral Pathology -
Question 131 of 150
131. Question
A 50-year-old diabetic patient presents with trismus three days after the extraction of a third molar. The patient reports difficulty opening the mouth, pain, and swelling in the extraction area. On examination, there is no evidence of infection or abscess. What is the most appropriate initial management for this patient?
Correct
ANSWER
Recommend warm saline rinses and jaw exercisesOTHER OPTIONS
⢠Prescribe antibiotics and perform incision and drainage – Prescribing antibiotics and performing incision and drainage is unnecessary if there is no evidence of infection or abscess.
⢠Refer to a specialist for surgical intervention – Referral to a specialist for surgical intervention is not indicated as the first line of treatment unless conservative measures fail or if there are signs of a serious underlying condition.
⢠Prescribe systemic corticosteroids to reduce inflammation – Systemic corticosteroids can help reduce inflammation but are not the first choice due to potential complications, especially in a diabetic patient where corticosteroids can affect blood glucose levels.SYNOPSIS
⢠Trismus, or difficulty in opening the mouth, is often due to muscle spasm or inflammation after third molar extraction. In a diabetic patient, careful management is required to prevent complications such as infection.
⢠Warm saline rinses and jaw exercises are appropriate initial management to reduce inflammation, promote healing, and help relieve muscle spasm.
⢠This approach encourages gentle movement and helps in reducing trismus without introducing the risk of infection.REFERENCE
Managing Trismus Post extraction – Journal of Oral and Maxillofacial SurgeryIncorrect
ANSWER
Recommend warm saline rinses and jaw exercisesOTHER OPTIONS
⢠Prescribe antibiotics and perform incision and drainage – Prescribing antibiotics and performing incision and drainage is unnecessary if there is no evidence of infection or abscess.
⢠Refer to a specialist for surgical intervention – Referral to a specialist for surgical intervention is not indicated as the first line of treatment unless conservative measures fail or if there are signs of a serious underlying condition.
⢠Prescribe systemic corticosteroids to reduce inflammation – Systemic corticosteroids can help reduce inflammation but are not the first choice due to potential complications, especially in a diabetic patient where corticosteroids can affect blood glucose levels.SYNOPSIS
⢠Trismus, or difficulty in opening the mouth, is often due to muscle spasm or inflammation after third molar extraction. In a diabetic patient, careful management is required to prevent complications such as infection.
⢠Warm saline rinses and jaw exercises are appropriate initial management to reduce inflammation, promote healing, and help relieve muscle spasm.
⢠This approach encourages gentle movement and helps in reducing trismus without introducing the risk of infection.REFERENCE
Managing Trismus Post extraction – Journal of Oral and Maxillofacial Surgery -
Question 132 of 150
132. Question
A 28-year-old male presents to the emergency department with severe facial trauma following a high-speed motor vehicle accident. The patient exhibits midface flattening, bilateral periorbital ecchymosis, and mobility of the midface relative to the cranial base. CT scans confirm a Le Fort III fracture. What is the most appropriate initial management for this patient?
Correct
ANSWER
Airway management and stabilization, followed by surgical interventionOTHER OPTIONS
⢠Immediate open reduction and internal fixation – Immediate open reduction and internal fixation (ORIF) without first ensuring airway management can be dangerous if the patient’s airway is compromised.
⢠Placement of intermaxillary fixation (IMF) and observation – Placement of intermaxillary fixation (IMF) and observation may not address the complexity and instability of a Le Fort III fracture and is not sufficient for definitive management.
⢠Antibiotic therapy and analgesics only – Antibiotic therapy and analgesics only do not address the critical need for surgical repair and stabilization of the facial fractures.SYNOPSIS
⢠In the management of Le Fort III fractures, which involve a complete craniofacial disjunction, the most critical initial step is to ensure the patient’s airway is secure and that the patient is hemodynamically stable.
⢠This is crucial because these fractures can significantly impact the airway and breathing due to the displacement of facial structures.
⢠Airway management and stabilization – Ensuring a clear airway and addressing any immediate life-threatening conditions take precedence. This might involve intubation or tracheostomy if necessary.
⢠Once the airway is secured and the patient is stabilized, surgical intervention (such as open reduction and internal fixation) is necessary to repair the fractures and restore the structural integrity and function of the facial skeleton.REFERENCE
Le Fort III fractures – An approach to resuscitation and management – Annals of Medicine and SurgeryIncorrect
ANSWER
Airway management and stabilization, followed by surgical interventionOTHER OPTIONS
⢠Immediate open reduction and internal fixation – Immediate open reduction and internal fixation (ORIF) without first ensuring airway management can be dangerous if the patient’s airway is compromised.
⢠Placement of intermaxillary fixation (IMF) and observation – Placement of intermaxillary fixation (IMF) and observation may not address the complexity and instability of a Le Fort III fracture and is not sufficient for definitive management.
⢠Antibiotic therapy and analgesics only – Antibiotic therapy and analgesics only do not address the critical need for surgical repair and stabilization of the facial fractures.SYNOPSIS
⢠In the management of Le Fort III fractures, which involve a complete craniofacial disjunction, the most critical initial step is to ensure the patient’s airway is secure and that the patient is hemodynamically stable.
⢠This is crucial because these fractures can significantly impact the airway and breathing due to the displacement of facial structures.
⢠Airway management and stabilization – Ensuring a clear airway and addressing any immediate life-threatening conditions take precedence. This might involve intubation or tracheostomy if necessary.
⢠Once the airway is secured and the patient is stabilized, surgical intervention (such as open reduction and internal fixation) is necessary to repair the fractures and restore the structural integrity and function of the facial skeleton.REFERENCE
Le Fort III fractures – An approach to resuscitation and management – Annals of Medicine and Surgery -
Question 133 of 150
133. Question
A 14-year-old patient presents with a Class II malocclusion characterized by a retrognathic mandible and significant overjet. The orthodontist decides to use a twin block appliance as part of the treatment plan. What is the most appropriate management strategy for using a twin block appliance in this patient?
Correct
ANSWER
Full-time wear with regular adjustments and monitoringOTHER OPTIONS
⢠Immediate full-time wear with no adjustments – Immediate full-time wear with no adjustments does not allow for necessary monitoring and adjustments, which are crucial for successful treatment.
⢠Part-time wear during the night only for the first month – Part-time wear during the night only for the first month is insufficient to achieve the desired results, as full-time wear is needed to promote consistent mandibular growth.SYNOPSIS
⢠The twin block appliance is an effective orthodontic device used to correct Class II malocclusion by encouraging mandibular growth and reducing overjet. The most appropriate management strategy involves
⢠Full-time wear – Twin block appliances should be worn full-time, including during eating, to maximize their effectiveness in stimulating mandibular growth and correcting malocclusion.
⢠Regular adjustments and monitoring – Regular follow-up appointments are essential to monitor progress, make necessary adjustments to the appliance, and ensure proper fit and function. These adjustments help guide the treatment process and address any issues that arise.REFERENCE
Case Report Treatment of Skeletal Class II Malocclusion with Twin-block – IOSR Journal of Dental and Medical SciencesIncorrect
ANSWER
Full-time wear with regular adjustments and monitoringOTHER OPTIONS
⢠Immediate full-time wear with no adjustments – Immediate full-time wear with no adjustments does not allow for necessary monitoring and adjustments, which are crucial for successful treatment.
⢠Part-time wear during the night only for the first month – Part-time wear during the night only for the first month is insufficient to achieve the desired results, as full-time wear is needed to promote consistent mandibular growth.SYNOPSIS
⢠The twin block appliance is an effective orthodontic device used to correct Class II malocclusion by encouraging mandibular growth and reducing overjet. The most appropriate management strategy involves
⢠Full-time wear – Twin block appliances should be worn full-time, including during eating, to maximize their effectiveness in stimulating mandibular growth and correcting malocclusion.
⢠Regular adjustments and monitoring – Regular follow-up appointments are essential to monitor progress, make necessary adjustments to the appliance, and ensure proper fit and function. These adjustments help guide the treatment process and address any issues that arise.REFERENCE
Case Report Treatment of Skeletal Class II Malocclusion with Twin-block – IOSR Journal of Dental and Medical Sciences -
Question 134 of 150
134. Question
Why do the spread of infection is more dangerous in children than adult?
Correct
ANSWER
Marrow spaces are wideOTHER OPTIONS
⢠Not applicableSYNOPSIS
⢠Because of wider marrow spaces in the child, an odontogenic infection can rapidly spread through the bone, possibly damaging the erupting teeth.
⢠Most odontogenic infections in the child are not serious and can be easily managed by removing the source of infection with pulp therapy or removing the involved tooth.
⢠Although uncommon, serious complications can occur when the infection spreads beyond the dentition, including cellulitis, cavernous sinus thrombosis, brain abscess, temporary blindness, airway obstruction, and mediastinal spread of infection.
⢠Classic signs and symptoms of infection include redness, pain, swelling, and local and systemic temperature increases.REFERENCE
Pediatric Dentistry Arthur.J.Nowak Page 405Incorrect
ANSWER
Marrow spaces are wideOTHER OPTIONS
⢠Not applicableSYNOPSIS
⢠Because of wider marrow spaces in the child, an odontogenic infection can rapidly spread through the bone, possibly damaging the erupting teeth.
⢠Most odontogenic infections in the child are not serious and can be easily managed by removing the source of infection with pulp therapy or removing the involved tooth.
⢠Although uncommon, serious complications can occur when the infection spreads beyond the dentition, including cellulitis, cavernous sinus thrombosis, brain abscess, temporary blindness, airway obstruction, and mediastinal spread of infection.
⢠Classic signs and symptoms of infection include redness, pain, swelling, and local and systemic temperature increases.REFERENCE
Pediatric Dentistry Arthur.J.Nowak Page 405 -
Question 135 of 150
135. Question
A child who had a pulpotomy comes next day with his parent and complained of a painful ulcer over the lower lip which was absent on the day of treatment. What would be the diagnosis?
Correct
ANSWER
Masticatory traumaOTHER OPTIONS
⢠Allergic reaction – Allergic reactions are not usually manifested as single ulcer and it shows similar features to aphthous ulcers.
⢠Aphthous ulcer – Aphthous ulcer or canker sores develop on lining mucosa.
⢠Herpes – In herpes, lesions develop mostly on keratinized mucosa or it can be seen as blisters around mouth and lips.SYNOPSIS
⢠Masticatory trauma develops usually following inferior alveolar nerve block.
⢠IANB results in large, heavy feeling of numbness where pediatric patient tends to squeeze or bite the anesthetized area causing laceration of lips, cheek or tongue.
⢠It can be prevented by timely warning.REFERENCE
Fundamentals of Pediatric Dentistry – 3rd Edition.Incorrect
ANSWER
Masticatory traumaOTHER OPTIONS
⢠Allergic reaction – Allergic reactions are not usually manifested as single ulcer and it shows similar features to aphthous ulcers.
⢠Aphthous ulcer – Aphthous ulcer or canker sores develop on lining mucosa.
⢠Herpes – In herpes, lesions develop mostly on keratinized mucosa or it can be seen as blisters around mouth and lips.SYNOPSIS
⢠Masticatory trauma develops usually following inferior alveolar nerve block.
⢠IANB results in large, heavy feeling of numbness where pediatric patient tends to squeeze or bite the anesthetized area causing laceration of lips, cheek or tongue.
⢠It can be prevented by timely warning.REFERENCE
Fundamentals of Pediatric Dentistry – 3rd Edition. -
Question 136 of 150
136. Question
How long will it take for remineralization of accidentally wrong etched tooth?
Correct
ANSWER
Few hoursOTHER OPTIONS
Not applicableSYNOPSIS
⢠A complete stabilisation or hardening of surface softened enamel after 24 hours remineralisation in artificial saliva was demonstrated.
⢠After 24 hours, a compact and homogeneous mineralized layer at the thickness of about 100 nm formed with similar structure to enamel.REFERENCE
Effect of Time on the Remineralisationof Enamel by Synthetic Saliva after Citric Acid erosion by ResearchGate.Incorrect
ANSWER
Few hoursOTHER OPTIONS
Not applicableSYNOPSIS
⢠A complete stabilisation or hardening of surface softened enamel after 24 hours remineralisation in artificial saliva was demonstrated.
⢠After 24 hours, a compact and homogeneous mineralized layer at the thickness of about 100 nm formed with similar structure to enamel.REFERENCE
Effect of Time on the Remineralisationof Enamel by Synthetic Saliva after Citric Acid erosion by ResearchGate. -
Question 137 of 150
137. Question
Which of the following correctly differentiates a craze line from a crack in dentistry?
Correct
ANSWER
Craze lines are superficial and affect only the enamel, while cracks extend into the dentin and potentially the pulp.OTHER OPTIONS
⢠Not applicableSYNOPSIS
⢠Craze lines are superficial lines that appear on the enamel and typically do not extend into the dentin. ⢠They are usually asymptomatic and do not require treatment unless for aesthetic reasons.
⢠Cracks extend deeper into the tooth structure, reaching the dentin and potentially the pulp, and can lead to symptoms such as pain, sensitivity, and even tooth fracture.
⢠Cracks often require more extensive treatment, including restorative procedures or, in severe cases, root canal therapy or extraction.
⢠Therefore, the correct differentiation is that craze lines are superficial and affect only the enamel, whereas cracks extend into the dentin and potentially the pulp.REFERENCE
Philips Science of Dental materialsIncorrect
ANSWER
Craze lines are superficial and affect only the enamel, while cracks extend into the dentin and potentially the pulp.OTHER OPTIONS
⢠Not applicableSYNOPSIS
⢠Craze lines are superficial lines that appear on the enamel and typically do not extend into the dentin. ⢠They are usually asymptomatic and do not require treatment unless for aesthetic reasons.
⢠Cracks extend deeper into the tooth structure, reaching the dentin and potentially the pulp, and can lead to symptoms such as pain, sensitivity, and even tooth fracture.
⢠Cracks often require more extensive treatment, including restorative procedures or, in severe cases, root canal therapy or extraction.
⢠Therefore, the correct differentiation is that craze lines are superficial and affect only the enamel, whereas cracks extend into the dentin and potentially the pulp.REFERENCE
Philips Science of Dental materials -
Question 138 of 150
138. Question
Which is the muscle present in relation to capsule of TMJ?
Correct
ANSWER
Lateral pterygoidOTHER OPTIONS
Not applicableSYNOPSIS
⢠TMJ is a synovial, condylar and hinge-type joint which involves fibrocartilaginous surfaces and an articular disc which divides the joint into two cavities.
⢠Capsule is a fibrous membrane that surrounds the joint and attaches to the articular eminence, the articular disc and the neck of the mandibular condyle.
⢠The articular disc is a fibrous extension of the capsule that runs between the two articular surfaces of the temporomandibular joint.
⢠Anterior disc attaches to the joint capsule and the superior head of the lateral pterygoid.
⢠TMJ is closely associated with lateral pterygoid muscle.REFERENCE
TMJ Anatomy by Physiopedia.Incorrect
ANSWER
Lateral pterygoidOTHER OPTIONS
Not applicableSYNOPSIS
⢠TMJ is a synovial, condylar and hinge-type joint which involves fibrocartilaginous surfaces and an articular disc which divides the joint into two cavities.
⢠Capsule is a fibrous membrane that surrounds the joint and attaches to the articular eminence, the articular disc and the neck of the mandibular condyle.
⢠The articular disc is a fibrous extension of the capsule that runs between the two articular surfaces of the temporomandibular joint.
⢠Anterior disc attaches to the joint capsule and the superior head of the lateral pterygoid.
⢠TMJ is closely associated with lateral pterygoid muscle.REFERENCE
TMJ Anatomy by Physiopedia. -
Question 139 of 150
139. Question
A 55-year-old patient reports experiencing chest pain that typically occurs with exertion and disappears after resting for a few minutes. This pattern of pain is most characteristic of which type of angina?
Correct
ANSWER
Stable AnginaOTHER OPTIONS
⢠Unstable Angina – Unstable Angina is characterized by chest pain that occurs at rest, is more severe or prolonged, and may not be relieved by rest or nitroglycerin, indicating a higher risk for myocardial infarction.
⢠Prinzmetalās Angina – Prinzmetalās Angina (or Variant Angina) is caused by coronary artery spasm and can occur at rest, often in cycles, and might not always be relieved by rest alone.
⢠Variant Angina – Variant Angina is a type of Prinzmetal’s Angina where the pain is related to coronary artery spasm and may not necessarily follow the pattern of exertion and relief with restSYNOPSIS
⢠Stable Angina is characterized by chest pain that occurs predictably with physical exertion or emotional stress and is relieved by rest or nitroglycerin.
⢠The pain typically resolves within a few minutes after the activity or stress is removed.REFERENCE
Medical Emergencies in Dental Office – Stanley F MalamedIncorrect
ANSWER
Stable AnginaOTHER OPTIONS
⢠Unstable Angina – Unstable Angina is characterized by chest pain that occurs at rest, is more severe or prolonged, and may not be relieved by rest or nitroglycerin, indicating a higher risk for myocardial infarction.
⢠Prinzmetalās Angina – Prinzmetalās Angina (or Variant Angina) is caused by coronary artery spasm and can occur at rest, often in cycles, and might not always be relieved by rest alone.
⢠Variant Angina – Variant Angina is a type of Prinzmetal’s Angina where the pain is related to coronary artery spasm and may not necessarily follow the pattern of exertion and relief with restSYNOPSIS
⢠Stable Angina is characterized by chest pain that occurs predictably with physical exertion or emotional stress and is relieved by rest or nitroglycerin.
⢠The pain typically resolves within a few minutes after the activity or stress is removed.REFERENCE
Medical Emergencies in Dental Office – Stanley F Malamed -
Question 140 of 150
140. Question
A patient came to your clinic for teeth replacement for missing 45 and 46. Patient wants a RPD. While planning RPD where will you give indirect retainer?
Correct
ANSWER
No indirect retainerOTHER OPTIONS
Not applicableSYNOPSIS
⢠Indirect retainer is component of partial removable dental prosthesis that assists the direct retainer in preventing displacement of distal extension of denture base by functioning through lever action on the opposite side of fulcrum line when denture base moves away from tissues in pure rotation around fulcrum line.
⢠The greater the distance between fulcrum line and indirect retainer, more effective it will be.
⢠In class III edentulous areas, indirect retainer is not necessary as there are no rotational forces.REFERENCE
Textbook of Prosthodontics, V Rangarajan – Page No 368.Incorrect
ANSWER
No indirect retainerOTHER OPTIONS
Not applicableSYNOPSIS
⢠Indirect retainer is component of partial removable dental prosthesis that assists the direct retainer in preventing displacement of distal extension of denture base by functioning through lever action on the opposite side of fulcrum line when denture base moves away from tissues in pure rotation around fulcrum line.
⢠The greater the distance between fulcrum line and indirect retainer, more effective it will be.
⢠In class III edentulous areas, indirect retainer is not necessary as there are no rotational forces.REFERENCE
Textbook of Prosthodontics, V Rangarajan – Page No 368. -
Question 141 of 150
141. Question
Which space lies between myelohyoid and platysma?
Correct
ANSWER
SubmentalOTHER OPTIONS
⢠Submandibular – Submandibular space is located beneath the mandible and above the mylohyoid muscle, but it extends laterally rather than being confined to the midline.
⢠Sublingual – Sublingual space is situated above the mylohyoid muscle and below the tongue, within the oral cavity.
⢠Buccal – Buccal space lies in the cheek area, lateral to the buccinator muscle and is not related to the mylohyoid muscle.SYNOPSIS
⢠The submental space is located between the mylohyoid muscle superiorly, the platysma muscle inferiorly, under the chin in the midline.
⢠The space coincides with the anatomic region termed the submental triangle, part of the anterior triangle of the neck.REFERENCE
Submental space – e-AnatomyIncorrect
ANSWER
SubmentalOTHER OPTIONS
⢠Submandibular – Submandibular space is located beneath the mandible and above the mylohyoid muscle, but it extends laterally rather than being confined to the midline.
⢠Sublingual – Sublingual space is situated above the mylohyoid muscle and below the tongue, within the oral cavity.
⢠Buccal – Buccal space lies in the cheek area, lateral to the buccinator muscle and is not related to the mylohyoid muscle.SYNOPSIS
⢠The submental space is located between the mylohyoid muscle superiorly, the platysma muscle inferiorly, under the chin in the midline.
⢠The space coincides with the anatomic region termed the submental triangle, part of the anterior triangle of the neck.REFERENCE
Submental space – e-Anatomy -
Question 142 of 150
142. Question
Which is the minimum depth required for a container used for the sterilization of dental instruments to ensure adequate steam penetration and sterilization?
Correct
ANSWER
4 inchesOTHER OPTIONS
⢠Not applicableSYNOPSIS
⢠For effective steam sterilization of dental instruments, the container used must allow adequate steam penetration to reach all surfaces of the instruments.
⢠A minimum depth of 4 inches is generally recommended to ensure that there is enough space for the steam to circulate around and fully penetrate the load.
⢠This helps to achieve thorough sterilization by allowing proper steam contact with all surfaces of the instruments.REFERENCE
Autoclave Safety and Operation – UNIVERSITY OF ILLINOIS URBANA-CHAMPAIGNIncorrect
ANSWER
4 inchesOTHER OPTIONS
⢠Not applicableSYNOPSIS
⢠For effective steam sterilization of dental instruments, the container used must allow adequate steam penetration to reach all surfaces of the instruments.
⢠A minimum depth of 4 inches is generally recommended to ensure that there is enough space for the steam to circulate around and fully penetrate the load.
⢠This helps to achieve thorough sterilization by allowing proper steam contact with all surfaces of the instruments.REFERENCE
Autoclave Safety and Operation – UNIVERSITY OF ILLINOIS URBANA-CHAMPAIGN -
Question 143 of 150
143. Question
A 50-year-old patient presents with a swollen, red tongue and reports difficulty in swallowing (dysphagia). The patient’s medical history reveals a vegan diet for the past 10 years. What is the most likely diagnosis for these symptoms?
Correct
ANSWER
Vit B12 deficiencyOTHER OPTIONS
⢠Geographic tongue – Geographic Tongue is characterized by map-like, red patches on the tongue with white borders, but it typically does not cause significant swelling or difficulty in swallowing.
⢠Candidiasis – Oral Thrush (candidiasis) presents as white, creamy plaques that can be scraped off, often accompanied by soreness, but not typically a swollen, red tongue with dysphagia.
⢠Herpetic gingivostomatitis – Herpetic Stomatitis is caused by herpes simplex virus and presents with multiple painful vesicles and ulcers in the oral cavity, not specifically with glossitis and dysphagia.SYNOPSIS
⢠Glossitis (inflammation of the tongue) accompanied by dysphagia can be indicative of a nutritional deficiency, particularly Vitamin B12 deficiency.
⢠This condition is common in individuals following a strict vegan diet, as Vitamin B12 is primarily found in animal products.REFERENCE
Shafer’s Textbook of Oral PathologyIncorrect
ANSWER
Vit B12 deficiencyOTHER OPTIONS
⢠Geographic tongue – Geographic Tongue is characterized by map-like, red patches on the tongue with white borders, but it typically does not cause significant swelling or difficulty in swallowing.
⢠Candidiasis – Oral Thrush (candidiasis) presents as white, creamy plaques that can be scraped off, often accompanied by soreness, but not typically a swollen, red tongue with dysphagia.
⢠Herpetic gingivostomatitis – Herpetic Stomatitis is caused by herpes simplex virus and presents with multiple painful vesicles and ulcers in the oral cavity, not specifically with glossitis and dysphagia.SYNOPSIS
⢠Glossitis (inflammation of the tongue) accompanied by dysphagia can be indicative of a nutritional deficiency, particularly Vitamin B12 deficiency.
⢠This condition is common in individuals following a strict vegan diet, as Vitamin B12 is primarily found in animal products.REFERENCE
Shafer’s Textbook of Oral Pathology -
Question 144 of 150
144. Question
A patient requires a dental implant in the posterior maxilla, but the maxillary sinus is pneumatized, resulting in insufficient bone height. Which procedure is commonly performed to allow for successful implant placement in this scenario?
Correct
ANSWER
Sinus liftOTHER OPTIONS
⢠Bone grafting – Bone grafting is a broader term that includes various techniques to increase bone volume, but in the context of a pneumatized maxillary sinus, it typically involves a sinus lift.
⢠Ridge expansion – Ridge expansion involves widening the alveolar ridge but is not specifically aimed at addressing issues with the maxillary sinus.
⢠Osteotomy – Osteotomy refers to the surgical cutting of bone and can be part of various procedures but is not specific to addressing sinus pneumatization.SYNOPSIS
⢠A sinus lift (or sinus augmentation) is a surgical procedure specifically designed to address the issue of a pneumatized maxillary sinus that results in insufficient bone height for implant placement.
⢠The procedure involves lifting the sinus membrane and placing a bone graft into the space created between the membrane and the floor of the sinus. This provides additional bone height and volume to securely place a dental implant.REFERENCE
Contemporary Implant Dentistry – Carl E MischIncorrect
ANSWER
Sinus liftOTHER OPTIONS
⢠Bone grafting – Bone grafting is a broader term that includes various techniques to increase bone volume, but in the context of a pneumatized maxillary sinus, it typically involves a sinus lift.
⢠Ridge expansion – Ridge expansion involves widening the alveolar ridge but is not specifically aimed at addressing issues with the maxillary sinus.
⢠Osteotomy – Osteotomy refers to the surgical cutting of bone and can be part of various procedures but is not specific to addressing sinus pneumatization.SYNOPSIS
⢠A sinus lift (or sinus augmentation) is a surgical procedure specifically designed to address the issue of a pneumatized maxillary sinus that results in insufficient bone height for implant placement.
⢠The procedure involves lifting the sinus membrane and placing a bone graft into the space created between the membrane and the floor of the sinus. This provides additional bone height and volume to securely place a dental implant.REFERENCE
Contemporary Implant Dentistry – Carl E Misch -
Question 145 of 150
145. Question
Ridge mapping is a technique used in dental implantology for which purpose?
Correct
ANSWER
Assessing the width of the alveolar ridgeOTHER OPTIONS
⢠Determining the density of the alveolar bone – Determining the density of the alveolar bone is typically achieved through imaging techniques such as CBCT scans, not ridge mapping.
⢠Measuring the thickness of the soft tissue – Measuring the thickness of the soft tissue can be done using a periodontal probe, but it is not the primary purpose of ridge mapping.
⢠Evaluating the height of the maxillary sinus – Evaluating the height of the maxillary sinus is relevant in sinus lift procedures and is typically assessed through radiographs.SYNOPSIS
⢠Ridge mapping is a technique used in dental implantology to assess the width of the alveolar ridge.
⢠This method involves using a caliper or a ridge mapping device to measure the thickness of the alveolar bone at various points.
⢠Accurate measurements are essential for planning the placement of dental implants, ensuring that there is sufficient bone width to support the implant.REFERENCE
Contemporary Implant Dentistry – Carl E MischIncorrect
ANSWER
Assessing the width of the alveolar ridgeOTHER OPTIONS
⢠Determining the density of the alveolar bone – Determining the density of the alveolar bone is typically achieved through imaging techniques such as CBCT scans, not ridge mapping.
⢠Measuring the thickness of the soft tissue – Measuring the thickness of the soft tissue can be done using a periodontal probe, but it is not the primary purpose of ridge mapping.
⢠Evaluating the height of the maxillary sinus – Evaluating the height of the maxillary sinus is relevant in sinus lift procedures and is typically assessed through radiographs.SYNOPSIS
⢠Ridge mapping is a technique used in dental implantology to assess the width of the alveolar ridge.
⢠This method involves using a caliper or a ridge mapping device to measure the thickness of the alveolar bone at various points.
⢠Accurate measurements are essential for planning the placement of dental implants, ensuring that there is sufficient bone width to support the implant.REFERENCE
Contemporary Implant Dentistry – Carl E Misch -
Question 146 of 150
146. Question
A 12-year-old patient presents with discolored teeth and enamel irregularities. On examination, you observe yellow-brown discoloration, thin enamel, and rough surfaces on all permanent teeth which are smaller than usual size. The patient has a positive family history of similar dental issues. What is the most likely diagnosis?
Correct
ANSWER
Amelogenesis ImperfectaOTHER OPTIONS
⢠Dentinogenesis imperfecta – Dentinogenesis imperfecta is a condition characterized by teeth that are translucent and discolored (most often blue-grey or yellow-brown in color).
⢠Hypocalcification – Enamel hypocalcification is the presence of white, brown or yellow stains on teeth and opaqueness on the tooth enamel.
⢠Fluorosis – Fluorosis isĀ a condition that results in white or brown speckles on the teeth.SYNOPSIS
⢠AmelogenesisĀ imperfecta refers toĀ a group of rare, inherited disorders characterized by abnormal enamel formation.Ā
⢠The term is typically restricted to those disorders of enamel development not associated with other abnormalities of the body.
⢠The main types are
– Hypoplastic (type I)
– Hypomaturation (type II)
– Hypocalcified (type III) and
– Hypomaturation or hypoplasia or taurodontism (type IV).
⢠Amelogenesis imperfecta usually is characterized by smaller than normal teeth, the color of which may range from white to yellow-brown, and teeth that appear to be mottled or spotted. The enamel is thinner than normal with areas that are clearly less dense (hypomineralized) and pitted.REFERENCE
Shafer’s text book of Oral PathologyIncorrect
ANSWER
Amelogenesis ImperfectaOTHER OPTIONS
⢠Dentinogenesis imperfecta – Dentinogenesis imperfecta is a condition characterized by teeth that are translucent and discolored (most often blue-grey or yellow-brown in color).
⢠Hypocalcification – Enamel hypocalcification is the presence of white, brown or yellow stains on teeth and opaqueness on the tooth enamel.
⢠Fluorosis – Fluorosis isĀ a condition that results in white or brown speckles on the teeth.SYNOPSIS
⢠AmelogenesisĀ imperfecta refers toĀ a group of rare, inherited disorders characterized by abnormal enamel formation.Ā
⢠The term is typically restricted to those disorders of enamel development not associated with other abnormalities of the body.
⢠The main types are
– Hypoplastic (type I)
– Hypomaturation (type II)
– Hypocalcified (type III) and
– Hypomaturation or hypoplasia or taurodontism (type IV).
⢠Amelogenesis imperfecta usually is characterized by smaller than normal teeth, the color of which may range from white to yellow-brown, and teeth that appear to be mottled or spotted. The enamel is thinner than normal with areas that are clearly less dense (hypomineralized) and pitted.REFERENCE
Shafer’s text book of Oral Pathology -
Question 147 of 150
147. Question
A patient presents with a carious tooth and rapidly progressing submandibular swelling, difficulty in swallowing, and elevation of the tongue. The patient is febrile and appears acutely ill. What term is commonly used to describe this severe, potentially life-threatening infection involving the submandibular and sublingual spaces?
Correct
ANSWER
Ludwig’s anginaOTHER OPTIONS
⢠Parotitis – Parotitis is the inflammation of the parotid gland. The typical symptoms of parotitis include enlargement of the parotid gland on one or both sides of the face, swelling, redness, tender to touch, and pain, especially while chewing and swallowing food.
⢠Pericoronitis – Pericoronitis is an intraoral inflammatory process due to infection of the gingival tissue surrounding or overlying an erupting or partially erupted tooth.
⢠Sialolithiasis – Sialolithiasis is a benign condition involving the formation of stones within the ducts of the major salivary glands. Clinical diagnosis of sialolithiasis can be challenging as patients may not be symptomatic unless the stone obstructs the salivary ducts, termed sialadenitis.SYNOPSIS
⢠Ludwig angina is a type of bacterial infection that occurs in the floor of the mouth, under the tongue.
⢠It often develops after an infection of the roots of the teeth (such as tooth abscess) or a mouth injury.Ā
⢠This rare type of cellulitis can spread quickly to your tongue and neck.
⢠Swelling can occur which cuts off breathing. This is a life-threatening emergency. Most people recover with antibiotics and surgery to drain the abscess. Rarely, the condition is fatal.REFERENCE
Incorrect
ANSWER
Ludwig’s anginaOTHER OPTIONS
⢠Parotitis – Parotitis is the inflammation of the parotid gland. The typical symptoms of parotitis include enlargement of the parotid gland on one or both sides of the face, swelling, redness, tender to touch, and pain, especially while chewing and swallowing food.
⢠Pericoronitis – Pericoronitis is an intraoral inflammatory process due to infection of the gingival tissue surrounding or overlying an erupting or partially erupted tooth.
⢠Sialolithiasis – Sialolithiasis is a benign condition involving the formation of stones within the ducts of the major salivary glands. Clinical diagnosis of sialolithiasis can be challenging as patients may not be symptomatic unless the stone obstructs the salivary ducts, termed sialadenitis.SYNOPSIS
⢠Ludwig angina is a type of bacterial infection that occurs in the floor of the mouth, under the tongue.
⢠It often develops after an infection of the roots of the teeth (such as tooth abscess) or a mouth injury.Ā
⢠This rare type of cellulitis can spread quickly to your tongue and neck.
⢠Swelling can occur which cuts off breathing. This is a life-threatening emergency. Most people recover with antibiotics and surgery to drain the abscess. Rarely, the condition is fatal.REFERENCE
-
Question 148 of 150
148. Question
A patient is in your dental clinic and wishes to have a tooth extraction of 21. On examination the tooth is restorable. What should the dentist do with the patient’s choice?
Correct
ANSWER
Consent and extract the toothOTHER OPTIONS
Not applicableSYNOPSIS
⢠Autonomy is an ethical principle which includes self governance.
⢠It includes patient’s privacy and safeguarding the patient’s privacy.
⢠It includes involving patient in treatment decisions with due consideration being given to patient needs, desires and abilities.
⢠Informing the patient of disease or defective dental concerns is first step.
⢠A written consent should be signed from patient before any treatment.REFERENCE
Ethics in Dentistry – Principles and Values.Incorrect
ANSWER
Consent and extract the toothOTHER OPTIONS
Not applicableSYNOPSIS
⢠Autonomy is an ethical principle which includes self governance.
⢠It includes patient’s privacy and safeguarding the patient’s privacy.
⢠It includes involving patient in treatment decisions with due consideration being given to patient needs, desires and abilities.
⢠Informing the patient of disease or defective dental concerns is first step.
⢠A written consent should be signed from patient before any treatment.REFERENCE
Ethics in Dentistry – Principles and Values. -
Question 149 of 150
149. Question
What is the concentration of fluoride in commonly available dentrifrices?
Correct
ANSWER
1500 ppmOTHER OPTIONS
Not applicableSYNOPSIS
⢠Fluoride has long been used to prevent decay, through a variety of different methods including toothpaste, water, milk, mouthrinses, tooth gels and varnish.
⢠Regular toothbrushing is recommended to prevent decay and other oral diseases, and toothbrushing for 2 minutes twice daily with a fluoride toothpaste is generally recommended.
⢠Typical strength of regular or family toothpaste is around 1000 to 1500 parts per million (ppm) fluoride.
⢠There is no minimum fluoride concentration, but the maximum permissible fluoride concentration for a toothpaste varies according to age and country.
⢠Toothpaste should contain at least 1000 ppm of fluoride and no more than 1500 ppm of fluoride.REFERENCE
Fluoride Toothpaste by WHO.Incorrect
ANSWER
1500 ppmOTHER OPTIONS
Not applicableSYNOPSIS
⢠Fluoride has long been used to prevent decay, through a variety of different methods including toothpaste, water, milk, mouthrinses, tooth gels and varnish.
⢠Regular toothbrushing is recommended to prevent decay and other oral diseases, and toothbrushing for 2 minutes twice daily with a fluoride toothpaste is generally recommended.
⢠Typical strength of regular or family toothpaste is around 1000 to 1500 parts per million (ppm) fluoride.
⢠There is no minimum fluoride concentration, but the maximum permissible fluoride concentration for a toothpaste varies according to age and country.
⢠Toothpaste should contain at least 1000 ppm of fluoride and no more than 1500 ppm of fluoride.REFERENCE
Fluoride Toothpaste by WHO. -
Question 150 of 150
150. Question
A 50 year old patient came with a pocket of 5-7 mm that persists even after scaling in 46 area. A 4 mm horizontal bone loss is also noted. Which treatment will you prefer?
Correct
ANSWER
Flap surgeryOTHER OPTIONS
Not applicableSYNOPSIS
⢠In moderate periodontitis cases, there is actual bone loss and the pockets may be 5-7mm in depth.
⢠Scaling and root planing will not predictably remove all the calculus from these deeper pockets, because of limited and difficult access in reaching the bottom of the pocket.
⢠Flap surgery is needed so the periodontist can gain access to clean the root.
⢠The ideal flap surgery, pocket elimination surgery, is achieved when the periodontist surgically removes the pocket by repositioning the gum down to the new bone level.
⢠Any irregularities or pitting of the bone that was caused by the infection is first corrected, and the gum is sutured tightly down to the re-contoured bone.
⢠Pocket elimination allows the patient to access and remove the bacterial plaque daily with brushing and flossing.REFERENCE
Treating Periodontal Disease by PerioPartner.Incorrect
ANSWER
Flap surgeryOTHER OPTIONS
Not applicableSYNOPSIS
⢠In moderate periodontitis cases, there is actual bone loss and the pockets may be 5-7mm in depth.
⢠Scaling and root planing will not predictably remove all the calculus from these deeper pockets, because of limited and difficult access in reaching the bottom of the pocket.
⢠Flap surgery is needed so the periodontist can gain access to clean the root.
⢠The ideal flap surgery, pocket elimination surgery, is achieved when the periodontist surgically removes the pocket by repositioning the gum down to the new bone level.
⢠Any irregularities or pitting of the bone that was caused by the infection is first corrected, and the gum is sutured tightly down to the re-contoured bone.
⢠Pocket elimination allows the patient to access and remove the bacterial plaque daily with brushing and flossing.REFERENCE
Treating Periodontal Disease by PerioPartner.
Last Week Performance Board
Leaderboard: EVERGREEN TEST SEP - 2
Pos. | Name | Entered on | Points | Result |
---|---|---|---|---|
Table is loading | ||||
No data available | ||||

Ready to book for this Sunday?
The test will be announced 1 week prior so that you will not miss it.